Re: Entanglement

2022-08-06 Thread Alan Grayson
Why are you so sure Einstein was wrong about locality? If worm holes exist, 
distant appearing particles might not be so distant. AG

On Friday, July 29, 2022 at 9:40:46 AM UTC-6 Lawrence Crowell wrote:

> No, Einstein was wrong on this.
>
> LC
>
> On Thursday, July 28, 2022 at 9:31:17 AM UTC-5 johnk...@gmail.com wrote:
>
>> On Thu, Jul 28, 2022 at 7:58 AM Alan Grayson  wrote:
>>
>> *> Maybe Einstein wasn't wrong about locality, only about geometry. AG*
>>>
>>
>> My hunch, and it's only a hunch, is that Einstein was right about 
>> locality because if things really were non-local then I don't see how we 
>> could understand anything until we understood everything. But as far as 
>> I know although Einstein found the wormhole solution that was lurking in 
>> his General Relativity equations he never made a connection between that 
>> and entanglement or locality, it was others who did that many decades after 
>> his death.  
>> John K ClarkSee what's on my new list at  Extropolis 
>> 
>> oah
>>
>>
>>

-- 
You received this message because you are subscribed to the Google Groups 
"Everything List" group.
To unsubscribe from this group and stop receiving emails from it, send an email 
to everything-list+unsubscr...@googlegroups.com.
To view this discussion on the web visit 
https://groups.google.com/d/msgid/everything-list/d527d4f3-11fb-407d-9e89-4848784aad64n%40googlegroups.com.


Re: Entanglement

2022-07-29 Thread Lawrence Crowell
No, Einstein was wrong on this.

LC

On Thursday, July 28, 2022 at 9:31:17 AM UTC-5 johnk...@gmail.com wrote:

> On Thu, Jul 28, 2022 at 7:58 AM Alan Grayson  wrote:
>
> *> Maybe Einstein wasn't wrong about locality, only about geometry. AG*
>>
>
> My hunch, and it's only a hunch, is that Einstein was right about locality 
> because if things really were non-local then I don't see how we could 
> understand anything until we understood everything. But as far as I know 
> although Einstein found the wormhole solution that was lurking in his 
> General Relativity equations he never made a connection between that and 
> entanglement or locality, it was others who did that many decades after his 
> death.  
> John K ClarkSee what's on my new list at  Extropolis 
> 
> oah
>
>
>

-- 
You received this message because you are subscribed to the Google Groups 
"Everything List" group.
To unsubscribe from this group and stop receiving emails from it, send an email 
to everything-list+unsubscr...@googlegroups.com.
To view this discussion on the web visit 
https://groups.google.com/d/msgid/everything-list/3d4f2715-0e38-4cff-9b6f-aee99a45f27an%40googlegroups.com.


Re: Entanglement

2022-07-29 Thread Lawrence Crowell
Two entangled black holes can teleport states. I may be wrong, but I think 
there is a way of experimentally showing this with a black hole analogue.

LC

On Wednesday, July 27, 2022 at 7:46:50 AM UTC-5 johnk...@gmail.com wrote:

> On Wed, Jul 27, 2022 at 8:20 AM Alan Grayson  wrote:
>
> *> When it's claimed that "entanglement is the glue that connects space 
>> with time", what does this mean? AG*
>
>  
> If entanglement does connect space with time nobody knows exactly how it 
> works, although there are theories, one recent one involves Einstein–Rosen 
> wormholes of unlimited length but submicroscopic diameter. There is zero 
> experimental evidence this theory is correct, and no evidence it is 
> incorrect either. All we know for sure is if 2 particles are entangled and 
> you make a change in one of them then you can instantaneously change the 
> other particle even if it is an enormous distance away, but you can't use 
> that effect to make a device that can communicate faster than light.
>
> John K ClarkSee what's on my new list at  Extropolis 
> 
> 2jr
>
>
>>

-- 
You received this message because you are subscribed to the Google Groups 
"Everything List" group.
To unsubscribe from this group and stop receiving emails from it, send an email 
to everything-list+unsubscr...@googlegroups.com.
To view this discussion on the web visit 
https://groups.google.com/d/msgid/everything-list/c942521a-dc8a-4120-8c04-43639c4f3358n%40googlegroups.com.


Re: Entanglement

2022-07-28 Thread John Clark
On Thu, Jul 28, 2022 at 7:58 AM Alan Grayson  wrote:

*> Maybe Einstein wasn't wrong about locality, only about geometry. AG*
>

My hunch, and it's only a hunch, is that Einstein was right about locality
because if things really were non-local then I don't see how we could
understand anything until we understood everything. But as far as I know
although Einstein found the wormhole solution that was lurking in his
General Relativity equations he never made a connection between that and
entanglement or locality, it was others who did that many decades after his
death.
John K ClarkSee what's on my new list at  Extropolis

oah

-- 
You received this message because you are subscribed to the Google Groups 
"Everything List" group.
To unsubscribe from this group and stop receiving emails from it, send an email 
to everything-list+unsubscr...@googlegroups.com.
To view this discussion on the web visit 
https://groups.google.com/d/msgid/everything-list/CAJPayv0p1K_%2B8B0Ux6DvEanhJLauedr0SPMdMTbkG4gWoTRpyw%40mail.gmail.com.


Re: Entanglement

2022-07-28 Thread Alan Grayson
Maybe Einstein wasn't wrong about locality, only about geometry. AG

On Wednesday, July 27, 2022 at 6:46:50 AM UTC-6 johnk...@gmail.com wrote:

> On Wed, Jul 27, 2022 at 8:20 AM Alan Grayson  wrote:
>
> *> When it's claimed that "entanglement is the glue that connects space 
>> with time", what does this mean? AG*
>
>  
> If entanglement does connect space with time nobody knows exactly how it 
> works, although there are theories, one recent one involves Einstein–Rosen 
> wormholes of unlimited length but submicroscopic diameter. There is zero 
> experimental evidence this theory is correct, and no evidence it is 
> incorrect either. All we know for sure is if 2 particles are entangled and 
> you make a change in one of them then you can instantaneously change the 
> other particle even if it is an enormous distance away, but you can't use 
> that effect to make a device that can communicate faster than light.
>
> John K ClarkSee what's on my new list at  Extropolis 
> 
> 2jr
>
>
>>

-- 
You received this message because you are subscribed to the Google Groups 
"Everything List" group.
To unsubscribe from this group and stop receiving emails from it, send an email 
to everything-list+unsubscr...@googlegroups.com.
To view this discussion on the web visit 
https://groups.google.com/d/msgid/everything-list/bdb16dd8-9598-47e2-8484-23ddd53627c0n%40googlegroups.com.


Re: Entanglement

2022-07-27 Thread John Clark
On Wed, Jul 27, 2022 at 8:20 AM Alan Grayson  wrote:

*> When it's claimed that "entanglement is the glue that connects space
> with time", what does this mean? AG*


If entanglement does connect space with time nobody knows exactly how it
works, although there are theories, one recent one involves Einstein–Rosen
wormholes of unlimited length but submicroscopic diameter. There is zero
experimental evidence this theory is correct, and no evidence it is
incorrect either. All we know for sure is if 2 particles are entangled and
you make a change in one of them then you can instantaneously change the
other particle even if it is an enormous distance away, but you can't use
that effect to make a device that can communicate faster than light.

John K ClarkSee what's on my new list at  Extropolis

2jr


>

-- 
You received this message because you are subscribed to the Google Groups 
"Everything List" group.
To unsubscribe from this group and stop receiving emails from it, send an email 
to everything-list+unsubscr...@googlegroups.com.
To view this discussion on the web visit 
https://groups.google.com/d/msgid/everything-list/CAJPayv1jE1JQxfPHguLBfRPFv7biUr%3DkYZ5oXZHan%2BLacxjTcQ%40mail.gmail.com.


Re: Entanglement and Superposition Are Equivalent Concepts in Any Physical Theory

2022-05-08 Thread Lawrence Crowell
Superposition and entanglement are different manifestations of a quantum 
phase. 

LC

On Tuesday, May 3, 2022 at 3:32:33 PM UTC-5 leeu...@gmail.com wrote:

> May be of interest:
>
> Entanglement and Superposition Are Equivalent Concepts in Any Physical 
> Theory
>
> ABSTRACT
>
> We prove that given any two general probabilistic theories (GPTs) the 
> following are equivalent: (i) each theory is nonclassical, meaning that 
> neither of their state spaces is a simplex; (ii) each theory satisfies a 
> strong notion of incompatibility equivalent to the existence of 
> “superpositions”; and (iii) the two theories are entangleable, in the sense 
> that their composite exhibits either entangled states or entangled 
> measurements. Intuitively, in the post-quantum GPT setting, a superposition 
> is a set of two binary ensembles of states that are unambiguously 
> distinguishable if the ensemble is revealed before the measurement has 
> occurred, but not if it is revealed after. This notion is important because 
> we show that, just like in quantum theory, superposition in the form of 
> strong incompatibility is sufficient to realize the Bennett-Brassard 1984 
> protocol for secret key distribution.
>
>
> https://journals.aps.org/prl/abstract/10.1103/PhysRevLett.128.160402
>
> Free access preprint:
> https://arxiv.org/abs/2109.04446
>
> Dirk
>
>

-- 
You received this message because you are subscribed to the Google Groups 
"Everything List" group.
To unsubscribe from this group and stop receiving emails from it, send an email 
to everything-list+unsubscr...@googlegroups.com.
To view this discussion on the web visit 
https://groups.google.com/d/msgid/everything-list/3c85097c-ca09-4108-bdf4-775892a15092n%40googlegroups.com.


Re: Entanglement Between Photons that have Never Coexisted

2019-07-18 Thread 'Cosmin Visan' via Everything List
Consciousness (and so, reality) is all about meaning reported to context. 
And meanings and contexts can be anything, they don't have to be spatial or 
temporal.

-- 
You received this message because you are subscribed to the Google Groups 
"Everything List" group.
To unsubscribe from this group and stop receiving emails from it, send an email 
to everything-list+unsubscr...@googlegroups.com.
To view this discussion on the web visit 
https://groups.google.com/d/msgid/everything-list/9bd621fb-7091-4c66-ba12-bcb67b61d8a6%40googlegroups.com.


Re: Entanglement Between Photons that have Never Coexisted

2019-07-08 Thread Philip Thrift


On Sunday, July 7, 2019 at 7:15:17 PM UTC-5, Brent wrote:
>
>
>
> On 7/7/2019 6:25 AM, Philip Thrift wrote:
>
>
> But the point is this topic was discussed for 20 years, beginning in the 
> group started by Victor Stenger (Timeless Reality). Physics does not rule 
> backward (or downward, for that matter) causation in or out, and someone 
> who tells you absolutely it is ruled out has no understanding of physics.
>
>
> In general it is ruled out by definition, as in classical mechanics: The 
> past and future are all determined by any sufficient set of conditions, so 
> when we give past conditions we say they cause the future condition and 
> when we give the future condition we say it is caused by the past.
>
> Brent
>
>


*Entangled Time*

https://www.sciencealert.com/if-you-thought-quantum-mechanics-was-weird-wait-til-you-check-out-entangled-time

*In both forward and backward directions, quantum correlations span the 
causal void between the death of one photon and the birth of the other.*

*We cannot afford to ignore spatial or temporal nonlocality in future 
metaphysics: whether or not the boots fit, we'll have to wear 'em.*

(What Would Vic (Victor Stenger) Say?)

@philipthrift

 

-- 
You received this message because you are subscribed to the Google Groups 
"Everything List" group.
To unsubscribe from this group and stop receiving emails from it, send an email 
to everything-list+unsubscr...@googlegroups.com.
To view this discussion on the web visit 
https://groups.google.com/d/msgid/everything-list/6cbf8c2b-7e91-40c6-b88b-8cd613447e0a%40googlegroups.com.


Re: Entanglement Between Photons that have Never Coexisted

2019-07-07 Thread 'Brent Meeker' via Everything List



On 7/7/2019 6:25 AM, Philip Thrift wrote:



On Sunday, July 7, 2019 at 7:50:10 AM UTC-5, Philip Thrift wrote:



On Sunday, July 7, 2019 at 7:41:24 AM UTC-5, Lawrence Crowell wrote:

On Sunday, July 7, 2019 at 7:03:05 AM UTC-5, Eva wrote:

@Lawrence Crowell

Very interesting, does it mean that everything is
connected not only spatially but also temporarily?


Sort of. The Wheeler delayed choice experiment indicates there
is a time aspect to entanglement as well. Look that up on
Wikipedia, and it is a bit odd. Quantum states and their
entanglements are not something that exist in space or time,
but which may have a representation in such. Things get a
little odd with time because there is no such thing as a
universal time operator. If there were it would mean the
conjugate of time, which is energy, as an operator can't have
a discrete spectrum.

@Philip Thrift

Retrocausation? So, I'm thirsty because I will drink
water? This is to much for me :/



Huw Price and Wharton have been beating this dead horse for a
long time. Not many physicists take this seriously, for it
would mean there is an underlying causal mechanism that would
obey Bell inequalities. The Bell theorem illustrate how
quantum physics violates these. The retrocauality idea is very
much an auslander conjecture that not many take seriously.

LC





This topic has been discussed for about 20 years here, beginning
in the original group.


By 'here' I meant on atvoid and atvoid-2 (the sequel), not 
everything-list.


But the point is this topic was discussed for 20 years, beginning in 
the group started by Victor Stenger (Timeless Reality). Physics does 
not rule backward (or downward, for that matter) causation in or out, 
and someone who tells you absolutely it is ruled out has no 
understanding of physics.


In general it is ruled out by definition, as in classical mechanics: The 
past and future are all determined by any sufficient set of conditions, 
so when we give past conditions we say they cause the future condition 
and when we give the future condition we say it is caused by the past.


Brent





https://groups.google.com/forum/#!forum/atvoid


Some are a dense today about this as they were 20 years ago. Don't
buy into the fundamentalist cult catechism (and a total
misunderstanding of physics) expressed above.

@philip thrift

--
You received this message because you are subscribed to the Google 
Groups "Everything List" group.
To unsubscribe from this group and stop receiving emails from it, send 
an email to everything-list+unsubscr...@googlegroups.com 
.
To view this discussion on the web visit 
https://groups.google.com/d/msgid/everything-list/d8093e3a-65f8-4636-8f74-b509370597bc%40googlegroups.com 
.


--
You received this message because you are subscribed to the Google Groups 
"Everything List" group.
To unsubscribe from this group and stop receiving emails from it, send an email 
to everything-list+unsubscr...@googlegroups.com.
To view this discussion on the web visit 
https://groups.google.com/d/msgid/everything-list/d63c226b-7e8d-e00b-eabc-5c4bf0ccab4d%40verizon.net.


Re: Entanglement Between Photons that have Never Coexisted

2019-07-07 Thread Philip Thrift


On Sunday, July 7, 2019 at 4:14:16 PM UTC-5, Lawrence Crowell wrote:
>
> On Sunday, July 7, 2019 at 2:18:15 PM UTC-5, Brent wrote:
>>
>>
>>
>> On 7/6/2019 4:50 PM, Lawrence Crowell wrote:
>>
>> n Saturday, July 6, 2019 at 6:04:18 PM UTC-5, Philip Thrift wrote: 
>>>
>>>
>>>
>>> *In conclusion, we have demonstrated quantum entanglement between two 
>>> photons that do not share coexistence. Although one photon is measured even 
>>> before the other is created, full quantum correlations were observed **by 
>>> measuring the density matrix of the two photons, conditioned on the result 
>>> of the projecting measurement.*
>>>
>>> A demonstration of retrocausation (retrodependency). 
>>>
>>> @philipthrift
>>>
>>
>> NO!! That violates Bell's inequalities and this measurement was done with 
>> the stats that violate Bell's inequalities.
>>
>> LC
>>
>>
>> You mean "That would satisfy Bell's inequalities..." don't you?
>>
>> Brent
>>
>
> I should have said violates Bell's theorem. Bell's theorem is on how QM 
> violates classical inequalities for probabilities. Any attempt to wire up 
> some underpinning to QM that is classical would mean quantum systems would 
> have a hidden variable that would obey the inequalities. 
>
> LC 
>

 

*Locality, Independence and the Pro-Liberty Bell*
https://arxiv.org/abs/quant-ph/9602020

Construed as an argument against hidden variable theories, Bell's Theorem 
assumes that hidden variables would be independent of future measurement 
settings. This Independence Assumption (IA) is rarely questioned. Bell 
considered relaxing it to avoid non-locality, but thought that the 
resulting view left no room for free will. However, Bell seems to have 
failed to distinguish two different strategies for giving up IA. One 
strategy takes for granted the Principle of the Common Cause, which 
requires that a correlation between hidden variables and measurement 
settings be explained by a joint correlation with some unknown factor in 
their common past. The other strategy rejects the Principle of the Common 
Cause, and argues that the required correlation might be due to the known 
interaction between the object system and the measuring device in their 
common future. Bell and most others who have discussed these issues have 
focussed on the former strategy, but because the two approaches have not 
been properly distinguished, it has not been well appreciated that there is 
a quite different way to relax IA. This paper distinguishes the two 
strategies, and argues that the latter is considerably more appealing than 
the former.


@philipthrift 

-- 
You received this message because you are subscribed to the Google Groups 
"Everything List" group.
To unsubscribe from this group and stop receiving emails from it, send an email 
to everything-list+unsubscr...@googlegroups.com.
To view this discussion on the web visit 
https://groups.google.com/d/msgid/everything-list/ec4274a7-2a1a-4e67-940f-56c42ec0293d%40googlegroups.com.


Re: Entanglement Between Photons that have Never Coexisted

2019-07-07 Thread Lawrence Crowell
On Sunday, July 7, 2019 at 2:18:15 PM UTC-5, Brent wrote:
>
>
>
> On 7/6/2019 4:50 PM, Lawrence Crowell wrote:
>
> n Saturday, July 6, 2019 at 6:04:18 PM UTC-5, Philip Thrift wrote: 
>>
>>
>>
>> *In conclusion, we have demonstrated quantum entanglement between two 
>> photons that do not share coexistence. Although one photon is measured even 
>> before the other is created, full quantum correlations were observed **by 
>> measuring the density matrix of the two photons, conditioned on the result 
>> of the projecting measurement.*
>>
>> A demonstration of retrocausation (retrodependency). 
>>
>> @philipthrift
>>
>
> NO!! That violates Bell's inequalities and this measurement was done with 
> the stats that violate Bell's inequalities.
>
> LC
>
>
> You mean "That would satisfy Bell's inequalities..." don't you?
>
> Brent
>

I should have said violates Bell's theorem. Bell's theorem is on how QM 
violates classical inequalities for probabilities. Any attempt to wire up 
some underpinning to QM that is classical would mean quantum systems would 
have a hidden variable that would obey the inequalities. 

LC 

-- 
You received this message because you are subscribed to the Google Groups 
"Everything List" group.
To unsubscribe from this group and stop receiving emails from it, send an email 
to everything-list+unsubscr...@googlegroups.com.
To view this discussion on the web visit 
https://groups.google.com/d/msgid/everything-list/caaf622d-7834-4418-9595-909cecc2d98e%40googlegroups.com.


Re: Entanglement Between Photons that have Never Coexisted

2019-07-07 Thread 'Brent Meeker' via Everything List



On 7/6/2019 4:50 PM, Lawrence Crowell wrote:

n Saturday, July 6, 2019 at 6:04:18 PM UTC-5, Philip Thrift wrote:



/In conclusion, we have demonstrated quantum entanglement between
two photons that do not share coexistence. Although one photon is
measured even before the other is created, full quantum
correlations were observed //by measuring the density matrix of
the two photons, conditioned on the result of the projecting
measurement./

A demonstration of retrocausation (retrodependency).

@philipthrift


NO!! That violates Bell's inequalities and this measurement was done 
with the stats that violate Bell's inequalities.


LC


You mean "That would satisfy Bell's inequalities..." don't you?

Brent

--
You received this message because you are subscribed to the Google Groups 
"Everything List" group.
To unsubscribe from this group and stop receiving emails from it, send an email 
to everything-list+unsubscr...@googlegroups.com.
To view this discussion on the web visit 
https://groups.google.com/d/msgid/everything-list/66b7a347-bd0b-82f4-6d77-3d3ec0919087%40verizon.net.


Re: Entanglement Between Photons that have Never Coexisted

2019-07-07 Thread Philip Thrift


On Sunday, July 7, 2019 at 7:50:10 AM UTC-5, Philip Thrift wrote:
>
>
>
> On Sunday, July 7, 2019 at 7:41:24 AM UTC-5, Lawrence Crowell wrote:
>>
>> On Sunday, July 7, 2019 at 7:03:05 AM UTC-5, Eva wrote:
>>>
>>> @Lawrence Crowell
>>>
>>> Very interesting, does it mean that everything is connected not only 
>>> spatially but also temporarily?
>>>
>>
>> Sort of. The Wheeler delayed choice experiment indicates there is a time 
>> aspect to entanglement as well. Look that up on Wikipedia, and it is a bit 
>> odd. Quantum states and their entanglements are not something that exist in 
>> space or time, but which may have a representation in such. Things get a 
>> little odd with time because there is no such thing as a universal time 
>> operator. If there were it would mean the conjugate of time, which is 
>> energy, as an operator can't have a discrete spectrum. 
>>  
>>
>>> @Philip Thrift
>>>
>>> Retrocausation? So, I'm thirsty because I will drink water? This is to 
>>> much for me :/
>>>
>>
>>
>> Huw Price and Wharton have been beating this dead horse for a long time. 
>> Not many physicists take this seriously, for it would mean there is an 
>> underlying causal mechanism that would obey Bell inequalities. The Bell 
>> theorem illustrate how quantum physics violates these. The retrocauality 
>> idea is very much an auslander conjecture that not many take seriously.
>>
>> LC
>>
>
>
>
>
> This topic has been discussed for about 20 years here, beginning in the 
> original group.
>

By 'here' I meant on atvoid and atvoid-2 (the sequel), not everything-list.

But the point is this topic was discussed for 20 years, beginning in the 
group started by Victor Stenger (Timeless Reality). Physics does not rule 
backward (or downward, for that matter) causation in or out, and someone 
who tells you absolutely it is ruled out has no understanding of physics.



>https://groups.google.com/forum/#!forum/atvoid
>
> Some are a dense today about this as they were 20 years ago. Don't buy 
> into the fundamentalist cult catechism (and a total misunderstanding of 
> physics) expressed above.
>
>  
> @philip thrift
>

-- 
You received this message because you are subscribed to the Google Groups 
"Everything List" group.
To unsubscribe from this group and stop receiving emails from it, send an email 
to everything-list+unsubscr...@googlegroups.com.
To view this discussion on the web visit 
https://groups.google.com/d/msgid/everything-list/d8093e3a-65f8-4636-8f74-b509370597bc%40googlegroups.com.


Re: Entanglement Between Photons that have Never Coexisted

2019-07-07 Thread Philip Thrift


On Sunday, July 7, 2019 at 7:41:24 AM UTC-5, Lawrence Crowell wrote:
>
> On Sunday, July 7, 2019 at 7:03:05 AM UTC-5, Eva wrote:
>>
>> @Lawrence Crowell
>>
>> Very interesting, does it mean that everything is connected not only 
>> spatially but also temporarily?
>>
>
> Sort of. The Wheeler delayed choice experiment indicates there is a time 
> aspect to entanglement as well. Look that up on Wikipedia, and it is a bit 
> odd. Quantum states and their entanglements are not something that exist in 
> space or time, but which may have a representation in such. Things get a 
> little odd with time because there is no such thing as a universal time 
> operator. If there were it would mean the conjugate of time, which is 
> energy, as an operator can't have a discrete spectrum. 
>  
>
>> @Philip Thrift
>>
>> Retrocausation? So, I'm thirsty because I will drink water? This is to 
>> much for me :/
>>
>
>
> Huw Price and Wharton have been beating this dead horse for a long time. 
> Not many physicists take this seriously, for it would mean there is an 
> underlying causal mechanism that would obey Bell inequalities. The Bell 
> theorem illustrate how quantum physics violates these. The retrocauality 
> idea is very much an auslander conjecture that not many take seriously.
>
> LC
>




This topic has been discussed for about 20 years here, beginning in the 
original group.

   https://groups.google.com/forum/#!forum/atvoid

Some are a dense today about this as they were 20 years ago. Don't buy into 
the fundamentalist cult catechism (and a total misunderstanding of physics) 
expressed above.

 
@philip thrift

-- 
You received this message because you are subscribed to the Google Groups 
"Everything List" group.
To unsubscribe from this group and stop receiving emails from it, send an email 
to everything-list+unsubscr...@googlegroups.com.
To view this discussion on the web visit 
https://groups.google.com/d/msgid/everything-list/8c298d1d-20c6-4df0-9567-705366f9be2a%40googlegroups.com.


Re: Entanglement Between Photons that have Never Coexisted

2019-07-07 Thread Lawrence Crowell
On Sunday, July 7, 2019 at 7:03:05 AM UTC-5, Eva wrote:
>
> @Lawrence Crowell
>
> Very interesting, does it mean that everything is connected not only 
> spatially but also temporarily?
>

Sort of. The Wheeler delayed choice experiment indicates there is a time 
aspect to entanglement as well. Look that up on Wikipedia, and it is a bit 
odd. Quantum states and their entanglements are not something that exist in 
space or time, but which may have a representation in such. Things get a 
little odd with time because there is no such thing as a universal time 
operator. If there were it would mean the conjugate of time, which is 
energy, as an operator can't have a discrete spectrum. 
 

> @Philip Thrift
>
> Retrocausation? So, I'm thirsty because I will drink water? This is to 
> much for me :/
>


Huw Price and Wharton have been beating this dead horse for a long time. 
Not many physicists take this seriously, for it would mean there is an 
underlying causal mechanism that would obey Bell inequalities. The Bell 
theorem illustrate how quantum physics violates these. The retrocauality 
idea is very much an auslander conjecture that not many take seriously.

LC

-- 
You received this message because you are subscribed to the Google Groups 
"Everything List" group.
To unsubscribe from this group and stop receiving emails from it, send an email 
to everything-list+unsubscr...@googlegroups.com.
To view this discussion on the web visit 
https://groups.google.com/d/msgid/everything-list/74a7f676-b3b2-4bbd-ac4a-ea4c623cb48d%40googlegroups.com.


Re: Entanglement Between Photons that have Never Coexisted

2019-07-07 Thread Eva
@Lawrence Crowell

Very interesting, does it mean that everything is connected not only spatially 
but also temporarily?

@Philip Thrift

Retrocausation? So, I'm thirsty because I will drink water? This is to much for 
me :/



-- 
You received this message because you are subscribed to the Google Groups 
"Everything List" group.
To unsubscribe from this group and stop receiving emails from it, send an email 
to everything-list+unsubscr...@googlegroups.com.
To view this discussion on the web visit 
https://groups.google.com/d/msgid/everything-list/ab5680e4-1cb7-41bf-8d79-7bc1cf2ca3e5%40googlegroups.com.


Re: Entanglement Between Photons that have Never Coexisted

2019-07-06 Thread Philip Thrift

You are completely clueless, and in addition give false information about 
the subject.

@philipthrift

On Saturday, July 6, 2019 at 6:50:29 PM UTC-5, Lawrence Crowell wrote:
>
> On Saturday, July 6, 2019 at 6:04:18 PM UTC-5, Philip Thrift wrote:
>>
>>
>>
>> *In conclusion, we have demonstrated quantum entanglement between two 
>> photons that do not share coexistence. Although one photon is measured even 
>> before the other is created, full quantum correlations were observed **by 
>> measuring the density matrix of the two photons, conditioned on the result 
>> of the projecting measurement.*
>>
>> A demonstration of retrocausation (retrodependency). 
>>
>> @philipthrift
>>
>
> NO!! That violates Bell's inequalities and this measurement was done with 
> the stats that violate Bell's inequalities.
>
> LC
>  
>
>>
>> On Saturday, July 6, 2019 at 5:17:07 PM UTC-5, Lawrence Crowell wrote:
>>>
>>> This is interesting, where photons that existed at different times can 
>>> be entangled.
>>>
>>> https://arxiv.org/pdf/1209.4191.pdf
>>>
>>> Entanglement Between Photons that have Never Coexisted 
>>>
>>> E. Megidish, A. Halevy, T. Shacham, T. Dvir, L. Dovrat, and H. S. 
>>> Eisenberg 
>>>
>>> Racah Institute of Physics, Hebrew University of Jerusalem, Jerusalem 
>>> 91904, Israel 
>>>
>>> The role of the timing and order of quantum measurements is not just a 
>>> fundamental question of quantum mechanics, but also a puzzling one. Any 
>>> part of a quantum system that has finished evolving, can be measured 
>>> immediately or saved for later, without affecting the final results, 
>>> regardless of the continued evolution of the rest of the system. In 
>>> addition, the non-locality of quantum mechanics, as manifested by 
>>> entanglement, does not apply only to particles with spatial separation, but 
>>> also with temporal separation. Here we demonstrate these principles by 
>>> generating and fully characterizing an entangled pair of photons that never 
>>> coexisted. Using entanglement swapping between two temporally separated 
>>> photon pairs we entangle one photon from the first pair with another photon 
>>> from the second pair. The first photon was detected even before the other 
>>> was created. The observed quantum correlations manifest the non-locality of 
>>> quantum mechanics in spacetime. 
>>>
>>

-- 
You received this message because you are subscribed to the Google Groups 
"Everything List" group.
To unsubscribe from this group and stop receiving emails from it, send an email 
to everything-list+unsubscr...@googlegroups.com.
To view this discussion on the web visit 
https://groups.google.com/d/msgid/everything-list/149955d4-389f-4a5c-852e-466b29b0ee18%40googlegroups.com.


Re: Entanglement Between Photons that have Never Coexisted

2019-07-06 Thread Lawrence Crowell
On Saturday, July 6, 2019 at 6:04:18 PM UTC-5, Philip Thrift wrote:
>
>
>
> *In conclusion, we have demonstrated quantum entanglement between two 
> photons that do not share coexistence. Although one photon is measured even 
> before the other is created, full quantum correlations were observed **by 
> measuring the density matrix of the two photons, conditioned on the result 
> of the projecting measurement.*
>
> A demonstration of retrocausation (retrodependency). 
>
> @philipthrift
>

NO!! That violates Bell's inequalities and this measurement was done with 
the stats that violate Bell's inequalities.

LC
 

>
> On Saturday, July 6, 2019 at 5:17:07 PM UTC-5, Lawrence Crowell wrote:
>>
>> This is interesting, where photons that existed at different times can be 
>> entangled.
>>
>> https://arxiv.org/pdf/1209.4191.pdf
>>
>> Entanglement Between Photons that have Never Coexisted 
>>
>> E. Megidish, A. Halevy, T. Shacham, T. Dvir, L. Dovrat, and H. S. 
>> Eisenberg 
>>
>> Racah Institute of Physics, Hebrew University of Jerusalem, Jerusalem 
>> 91904, Israel 
>>
>> The role of the timing and order of quantum measurements is not just a 
>> fundamental question of quantum mechanics, but also a puzzling one. Any 
>> part of a quantum system that has finished evolving, can be measured 
>> immediately or saved for later, without affecting the final results, 
>> regardless of the continued evolution of the rest of the system. In 
>> addition, the non-locality of quantum mechanics, as manifested by 
>> entanglement, does not apply only to particles with spatial separation, but 
>> also with temporal separation. Here we demonstrate these principles by 
>> generating and fully characterizing an entangled pair of photons that never 
>> coexisted. Using entanglement swapping between two temporally separated 
>> photon pairs we entangle one photon from the first pair with another photon 
>> from the second pair. The first photon was detected even before the other 
>> was created. The observed quantum correlations manifest the non-locality of 
>> quantum mechanics in spacetime. 
>>
>

-- 
You received this message because you are subscribed to the Google Groups 
"Everything List" group.
To unsubscribe from this group and stop receiving emails from it, send an email 
to everything-list+unsubscr...@googlegroups.com.
To view this discussion on the web visit 
https://groups.google.com/d/msgid/everything-list/24516145-2c50-4dd3-8d65-33dee9c7af87%40googlegroups.com.


Re: Entanglement Between Photons that have Never Coexisted

2019-07-06 Thread Philip Thrift


*In conclusion, we have demonstrated quantum entanglement between two 
photons that do not share coexistence. Although one photon is measured even 
before the other is created, full quantum correlations were observed **by 
measuring the density matrix of the two photons, conditioned on the result 
of the projecting measurement.*

A demonstration of retrocausation (retrodependency). 

@philipthrift

On Saturday, July 6, 2019 at 5:17:07 PM UTC-5, Lawrence Crowell wrote:
>
> This is interesting, where photons that existed at different times can be 
> entangled.
>
> https://arxiv.org/pdf/1209.4191.pdf
>
> Entanglement Between Photons that have Never Coexisted 
>
> E. Megidish, A. Halevy, T. Shacham, T. Dvir, L. Dovrat, and H. S. 
> Eisenberg 
>
> Racah Institute of Physics, Hebrew University of Jerusalem, Jerusalem 
> 91904, Israel 
>
> The role of the timing and order of quantum measurements is not just a 
> fundamental question of quantum mechanics, but also a puzzling one. Any 
> part of a quantum system that has finished evolving, can be measured 
> immediately or saved for later, without affecting the final results, 
> regardless of the continued evolution of the rest of the system. In 
> addition, the non-locality of quantum mechanics, as manifested by 
> entanglement, does not apply only to particles with spatial separation, but 
> also with temporal separation. Here we demonstrate these principles by 
> generating and fully characterizing an entangled pair of photons that never 
> coexisted. Using entanglement swapping between two temporally separated 
> photon pairs we entangle one photon from the first pair with another photon 
> from the second pair. The first photon was detected even before the other 
> was created. The observed quantum correlations manifest the non-locality of 
> quantum mechanics in spacetime. 
>

-- 
You received this message because you are subscribed to the Google Groups 
"Everything List" group.
To unsubscribe from this group and stop receiving emails from it, send an email 
to everything-list+unsubscr...@googlegroups.com.
To view this discussion on the web visit 
https://groups.google.com/d/msgid/everything-list/702eb3d3-21c6-4124-9297-0b72850b8784%40googlegroups.com.


Re: Entanglement

2018-06-11 Thread agrayson2000


On Tuesday, June 12, 2018 at 2:35:10 AM UTC, Brent wrote:
>
>
>
> On 6/11/2018 7:12 PM, agrays...@gmail.com  wrote:
>
>
>
> On Tuesday, June 12, 2018 at 1:53:42 AM UTC, Brent wrote: 
>>
>>
>>
>> On 6/11/2018 6:26 PM, agrays...@gmail.com wrote:
>>
>>
>>
>>
>>
>> On Monday, June 11, 2018 at 10:57:59 PM UTC, Brent wrote: 
>>>
>>>
>>>
>>> On 6/11/2018 3:22 PM, agrays...@gmail.com wrote:
>>>
>>>
> I am not sure this make sense (with the SWE). The cat is always 
> isolated, in some sense. 
>


 * IMO totally wrong. In fact now you're contradicting what you wrote in 
 a recent post. The cat is NEVER ISOLATED, VIRTUALLY BY DEFINITION OF WHAT 
 MACRO MEANS. NEVER ISOLATED IMPLIES NEVER IN A SUPERPOSITION. AG *

>>>
>>> * In the real world the cat is never isolated, nor can it be isolated 
>>> insofar as it consists of a huge number of particles already entangled with 
>>> its environment. This is the meaning of "macro" ! If you insist on 
>>> imagining it as isolated for your thought experiment, you will generate a 
>>> paradox, as Schroedinger did.  AG*
>>>
>>>
>>> Schroedinger obviously chose a cat to emphasize the absurdity, but it 
>>> also makes the analysis hard to think about.  Not only is the cat 
>>> macroscopic, the atomic decay is distributed over a continuum. I think it 
>>> helps to think of a simpler experiment in which the atom is just in a box 
>>> which is lined with photographic plates.  So the atom is in a superposition 
>>> of undecayed and decayed and interacted with a silver halide atom.  It is 
>>> clear that it is the interaction with the silver halide atom that gets 
>>> amplified to a macroscopic dot of silver which decoheres the system in 
>>> orthogonal "worlds" in which the spot is in different places and happens at 
>>> different times.
>>>
>>> Brent
>>>
>>
>> *I don't see how a single silver halide atom is amplified to a 
>> macroscopic dot of silver. *
>>
>>
>> That's because you don't bother to look up anything.  You expect other 
>> people to look it up for you and then explain it to you.  Here's Wikipedia, 
>> but I'm not explaining it.
>>
>
> * The original SINGLE atom in your example is not amplified, as I 
> suspected. More important, you ducked the main issue IMO. AG *
>
>
> As Oliver Heaviside said, "I have given you an explanation.  I'm not 
> obliged to give you an understanding."
>
> Brent
>

*I don't see what your example is supposed to demonstrate; that the sliver 
of silver is macro, isolated, and then decoheres? Why is it isolated? Even 
so, how does that justify putting macro entities in superpositions? Maybe I 
should ask Heaviside. But first I need to first consult Clark who will know 
how to resurrect him (Heaviside). **AG*
 

>
>
>> *Silver halides are used in photographic film and photographic paper, 
>> including graphic art film and paper, where silver halide crystals in 
>> gelatin are coated on to a film base, glass or paper substrate. The gelatin 
>> is a vital part of the emulsion as the protective colloid of appropriate 
>> physical and chemical properties. Gelatin may also contain trace elements 
>> (such as sulfur) which increase the light sensitivity of the emulsion, 
>> although modern practice uses gelatin without such components. When 
>> absorbed by an AgX crystal, photons cause electrons to be promoted to a 
>> conduction band (de-localized electron orbital with higher energy than a 
>> valence band) which can be attracted by a sensitivity speck, which is a 
>> shallow electron trap, which may be a crystalline defect or a cluster of 
>> silver sulfide, gold, other trace elements (dopant), or combination 
>> thereof, and then combined with an interstitial silver ion to form silver 
>> metal speck.[1]*
>>
>>
>>
>> *When a silver halide crystal is exposed to light, a sensitivity speck on 
>> the surface of the crystal is turned into a small speck of metallic silver 
>> (these comprise the invisible or latent image). If the speck of silver 
>> contains approximately four or more atoms, it is rendered developable - 
>> meaning that it can undergo development which turns the entire crystal into 
>> metallic silver. *Brent
>>
>> *Going back to my analysis, I think I have shown the fallacy of using a 
>> macro entities in a superposition (since they can never be isolated). But 
>> this is the starting point of decoherence theory, as exemplified by the wf 
>> Bruce recently presented for a spin 1/2 measurement (where the apparatus, 
>> observer and remaining environment appear in the superposition). AG*
>> -- 
>> You received this message because you are subscribed to the Google Groups 
>> "Everything List" group.
>> To unsubscribe from this group and stop receiving emails from it, send an 
>> email to everything-li...@googlegroups.com.
>> To post to this group, send email to everyth...@googlegroups.com.
>> Visit this group at https://groups.google.com/group/everything-list.
>> For more options, visit 

Re: Entanglement

2018-06-11 Thread agrayson2000


On Tuesday, June 12, 2018 at 2:12:51 AM UTC, agrays...@gmail.com wrote:
>
>
>
> On Tuesday, June 12, 2018 at 1:53:42 AM UTC, Brent wrote:
>>
>>
>>
>> On 6/11/2018 6:26 PM, agrays...@gmail.com wrote:
>>
>>
>>
>>
>>
>> On Monday, June 11, 2018 at 10:57:59 PM UTC, Brent wrote: 
>>>
>>>
>>>
>>> On 6/11/2018 3:22 PM, agrays...@gmail.com wrote:
>>>
>>>
> I am not sure this make sense (with the SWE). The cat is always 
> isolated, in some sense. 
>


 * IMO totally wrong. In fact now you're contradicting what you wrote in 
 a recent post. The cat is NEVER ISOLATED, VIRTUALLY BY DEFINITION OF WHAT 
 MACRO MEANS. NEVER ISOLATED IMPLIES NEVER IN A SUPERPOSITION. AG *

>>>
>>> * In the real world the cat is never isolated, nor can it be isolated 
>>> insofar as it consists of a huge number of particles already entangled with 
>>> its environment. This is the meaning of "macro" ! If you insist on 
>>> imagining it as isolated for your thought experiment, you will generate a 
>>> paradox, as Schroedinger did.  AG*
>>>
>>>
>>> Schroedinger obviously chose a cat to emphasize the absurdity, but it 
>>> also makes the analysis hard to think about.  Not only is the cat 
>>> macroscopic, the atomic decay is distributed over a continuum. I think it 
>>> helps to think of a simpler experiment in which the atom is just in a box 
>>> which is lined with photographic plates.  So the atom is in a superposition 
>>> of undecayed and decayed and interacted with a silver halide atom.  It is 
>>> clear that it is the interaction with the silver halide atom that gets 
>>> amplified to a macroscopic dot of silver which decoheres the system in 
>>> orthogonal "worlds" in which the spot is in different places and happens at 
>>> different times.
>>>
>>> Brent
>>>
>>
>> *I don't see how a single silver halide atom is amplified to a 
>> macroscopic dot of silver. *
>>
>>
>> That's because you don't bother to look up anything.  You expect other 
>> people to look it up for you and then explain it to you.  Here's Wikipedia, 
>> but I'm not explaining it.
>>
>
> *The original SINGLE atom in your example is not amplified, as I 
> suspected. More important, you ducked the main issue IMO. AG *
>

*You know, if you're referring to a chemical reaction that converts some 
silver halide atoms into specks of silver, there's no particular mystery. 
This, however, is not the same as something micro become macro (which is 
how I read your comment). In any event, I fail to see how macro systems 
which cannot be isolated, and thus cannot be prepared in a superposition, 
can be represented mathematically in a superposition of states, which is 
the starting point for decoherence. AG* 

>
> *photographic paper, including graphic art film and paper, where silver 
>> halide crystals in gelatin are coated on to a film base, glass or paper 
>> substrate. The gelatin is a vital part of the emulsion as the protective 
>> colloid of appropriate physical and chemical properties. Gelatin may also 
>> contain trace elements (such as sulfur) which increase the light 
>> sensitivity of the emulsion, although modern practice uses gelatin without 
>> such components. When absorbed by an AgX crystal, photons cause electrons 
>> to be promoted to a conduction band (de-localized electron orbital with 
>> higher energy than a valence band) which can be attracted by a sensitivity 
>> speck, which is a shallow electron trap, which may be a crystalline defect 
>> or a cluster of silver sulfide, gold, other trace elements (dopant), or 
>> combination thereof, and then combined with an interstitial silver ion to 
>> form silver metal speck.[1]*
>>
>>
>>
>> *When a silver halide crystal is exposed to light, a sensitivity speck on 
>> the surface of the crystal is turned into a small speck of metallic silver 
>> (these comprise the invisible or latent image). If the speck of silver 
>> contains approximately four or more atoms, it is rendered developable - 
>> meaning that it can undergo development which turns the entire crystal into 
>> metallic silver. *Brent
>>
>> *Going back to my analysis, I think I have shown the fallacy of using a 
>> macro entities in a superposition (since they can never be isolated). But 
>> this is the starting point of decoherence theory, as exemplified by the wf 
>> Bruce recently presented for a spin 1/2 measurement (where the apparatus, 
>> observer and remaining environment appear in the superposition). AG*
>> -- 
>> You received this message because you are subscribed to the Google Groups 
>> "Everything List" group.
>> To unsubscribe from this group and stop receiving emails from it, send an 
>> email to everything-li...@googlegroups.com.
>> To post to this group, send email to everyth...@googlegroups.com.
>> Visit this group at https://groups.google.com/group/everything-list.
>> For more options, visit https://groups.google.com/d/optout.
>>
>>
>>

-- 
You received this message because you are subscribed to 

Re: Entanglement

2018-06-11 Thread Brent Meeker



On 6/11/2018 7:12 PM, agrayson2...@gmail.com wrote:



On Tuesday, June 12, 2018 at 1:53:42 AM UTC, Brent wrote:



On 6/11/2018 6:26 PM, agrays...@gmail.com  wrote:





On Monday, June 11, 2018 at 10:57:59 PM UTC, Brent wrote:



On 6/11/2018 3:22 PM, agrays...@gmail.com wrote:



I am not sure this make sense (with the SWE). The
cat is always isolated, in some sense.

*
IMO totally wrong. In fact now you're contradicting what
you wrote in a recent post. The cat is NEVER ISOLATED,
VIRTUALLY BY DEFINITION OF WHAT MACRO MEANS. NEVER
ISOLATED IMPLIES NEVER IN A SUPERPOSITION. AG
*

*
In the real world the cat is never isolated, nor can it be
isolated insofar as it consists of a huge number of
particles already entangled with its environment. This is
the meaning of "macro" ! If you insist on imagining it as
isolated for your thought experiment, you will generate a
paradox, as Schroedinger did.  AG*


Schroedinger obviously chose a cat to emphasize the
absurdity, but it also makes the analysis hard to think
about.  Not only is the cat macroscopic, the atomic decay is
distributed over a continuum. I think it helps to think of a
simpler experiment in which the atom is just in a box which
is lined with photographic plates.  So the atom is in a
superposition of undecayed and decayed and interacted with a
silver halide atom.  It is clear that it is the interaction
with the silver halide atom that gets amplified to a
macroscopic dot of silver which decoheres the system in
orthogonal "worlds" in which the spot is in different places
and happens at different times.

Brent


*I don't see how a single silver halide atom is amplified to a
macroscopic dot of silver. *


That's because you don't bother to look up anything.  You expect
other people to look it up for you and then explain it to you. 
Here's Wikipedia, but I'm not explaining it.

*
The original SINGLE atom in your example is not amplified, as I 
suspected. More important, you ducked the main issue IMO. AG *


As Oliver Heaviside said, "I have given you an explanation.  I'm not 
obliged to give you an understanding."


Brent



/Silver halides are used in photographic film and photographic
paper, including graphic art film and paper, where silver halide
crystals in gelatin are coated on to a film base, glass or paper
substrate. The gelatin is a vital part of the emulsion as the
protective colloid of appropriate physical and chemical
properties. Gelatin may also contain trace elements (such as
sulfur) which increase the light sensitivity of the emulsion,
although modern practice uses gelatin without such components.
When absorbed by an AgX crystal, photons cause electrons to be
promoted to a conduction band (de-localized electron orbital with
higher energy than a valence band) which can be attracted by a
sensitivity speck, which is a shallow electron trap, which may be
a crystalline defect or a cluster of silver sulfide, gold, other
trace elements (dopant), or combination thereof, and then combined
with an interstitial silver ion to form silver metal speck.[1]//
//
//When a silver halide crystal is exposed to light, a sensitivity
speck on the surface of the crystal is turned into a small speck
of metallic silver (these comprise the invisible or latent image).
If the speck of silver contains approximately four or more atoms,
it is rendered developable - meaning that it can undergo
development which turns the entire crystal into metallic silver.

/Brent/
/

*Going back to my analysis, I think I have shown the fallacy of
using a macro entities in a superposition (since they can never
be isolated). But this is the starting point of decoherence
theory, as exemplified by the wf Bruce recently presented for a
spin 1/2 measurement (where the apparatus, observer and remaining
environment appear in the superposition). AG*
-- 
You received this message because you are subscribed to the

Google Groups "Everything List" group.
To unsubscribe from this group and stop receiving emails from it,
send an email to everything-li...@googlegroups.com .
To post to this group, send email to everyth...@googlegroups.com
.
Visit this group at
https://groups.google.com/group/everything-list
.
For more options, visit https://groups.google.com/d/optout
.


--
You received this message because you are subscribed to the Google 
Groups "Everything List" group.
To unsubscribe from this group and stop receiving emails from it, send 
an email to 

Re: Entanglement

2018-06-11 Thread agrayson2000


On Tuesday, June 12, 2018 at 1:53:42 AM UTC, Brent wrote:
>
>
>
> On 6/11/2018 6:26 PM, agrays...@gmail.com  wrote:
>
>
>
>
>
> On Monday, June 11, 2018 at 10:57:59 PM UTC, Brent wrote: 
>>
>>
>>
>> On 6/11/2018 3:22 PM, agrays...@gmail.com wrote:
>>
>>
 I am not sure this make sense (with the SWE). The cat is always 
 isolated, in some sense. 

>>>
>>>
>>> * IMO totally wrong. In fact now you're contradicting what you wrote in 
>>> a recent post. The cat is NEVER ISOLATED, VIRTUALLY BY DEFINITION OF WHAT 
>>> MACRO MEANS. NEVER ISOLATED IMPLIES NEVER IN A SUPERPOSITION. AG *
>>>
>>
>> * In the real world the cat is never isolated, nor can it be isolated 
>> insofar as it consists of a huge number of particles already entangled with 
>> its environment. This is the meaning of "macro" ! If you insist on 
>> imagining it as isolated for your thought experiment, you will generate a 
>> paradox, as Schroedinger did.  AG*
>>
>>
>> Schroedinger obviously chose a cat to emphasize the absurdity, but it 
>> also makes the analysis hard to think about.  Not only is the cat 
>> macroscopic, the atomic decay is distributed over a continuum. I think it 
>> helps to think of a simpler experiment in which the atom is just in a box 
>> which is lined with photographic plates.  So the atom is in a superposition 
>> of undecayed and decayed and interacted with a silver halide atom.  It is 
>> clear that it is the interaction with the silver halide atom that gets 
>> amplified to a macroscopic dot of silver which decoheres the system in 
>> orthogonal "worlds" in which the spot is in different places and happens at 
>> different times.
>>
>> Brent
>>
>
> *I don't see how a single silver halide atom is amplified to a macroscopic 
> dot of silver. *
>
>
> That's because you don't bother to look up anything.  You expect other 
> people to look it up for you and then explain it to you.  Here's Wikipedia, 
> but I'm not explaining it.
>

*The original SINGLE atom in your example is not amplified, as I suspected. 
More important, you ducked the main issue IMO. AG *

>
> *Silver halides are used in photographic film and photographic paper, 
> including graphic art film and paper, where silver halide crystals in 
> gelatin are coated on to a film base, glass or paper substrate. The gelatin 
> is a vital part of the emulsion as the protective colloid of appropriate 
> physical and chemical properties. Gelatin may also contain trace elements 
> (such as sulfur) which increase the light sensitivity of the emulsion, 
> although modern practice uses gelatin without such components. When 
> absorbed by an AgX crystal, photons cause electrons to be promoted to a 
> conduction band (de-localized electron orbital with higher energy than a 
> valence band) which can be attracted by a sensitivity speck, which is a 
> shallow electron trap, which may be a crystalline defect or a cluster of 
> silver sulfide, gold, other trace elements (dopant), or combination 
> thereof, and then combined with an interstitial silver ion to form silver 
> metal speck.[1]*
>
>
>
> *When a silver halide crystal is exposed to light, a sensitivity speck on 
> the surface of the crystal is turned into a small speck of metallic silver 
> (these comprise the invisible or latent image). If the speck of silver 
> contains approximately four or more atoms, it is rendered developable - 
> meaning that it can undergo development which turns the entire crystal into 
> metallic silver. *Brent
>
> *Going back to my analysis, I think I have shown the fallacy of using a 
> macro entities in a superposition (since they can never be isolated). But 
> this is the starting point of decoherence theory, as exemplified by the wf 
> Bruce recently presented for a spin 1/2 measurement (where the apparatus, 
> observer and remaining environment appear in the superposition). AG*
> -- 
> You received this message because you are subscribed to the Google Groups 
> "Everything List" group.
> To unsubscribe from this group and stop receiving emails from it, send an 
> email to everything-li...@googlegroups.com .
> To post to this group, send email to everyth...@googlegroups.com 
> .
> Visit this group at https://groups.google.com/group/everything-list.
> For more options, visit https://groups.google.com/d/optout.
>
>
>

-- 
You received this message because you are subscribed to the Google Groups 
"Everything List" group.
To unsubscribe from this group and stop receiving emails from it, send an email 
to everything-list+unsubscr...@googlegroups.com.
To post to this group, send email to everything-list@googlegroups.com.
Visit this group at https://groups.google.com/group/everything-list.
For more options, visit https://groups.google.com/d/optout.


Re: Entanglement

2018-06-11 Thread Bruce Kellett

From: *Bruno Marchal* mailto:marc...@ulb.ac.be>>
On 11 Jun 2018, at 14:48, Bruce Kellett > wrote:


From: *Bruno Marchal* mailto:marc...@ulb.ac.be>>

Only in the sense that the biological brain has evolved through 
decoherence with respect to some base, but as you say, that process 
lead to the same result from the 1p perspective of those who have 
chosen the base, or have the base imposed through decoherence and 
evolution, say.
If not then QM would be inconsistent, and had to different laws of 
physics for different observers.


[BK] That would be the result absent decoherence to the stable basis 
for any measurement.



OK. We don’t differ on the fundamentals. I would need to revise Zurek 
and Zeh to assures myself that some base are more stable than other 
for physical reason (and not simply Everett-anthropic one),


I don't think an anthropic approach is adequate here. That relies on 
some unexplained magic to account for the fact that evolution has 
selected our brains to perceive in some preferred basis. But that is not 
an explanation -- that is an excuse. I don't know if Zeh has actually 
explored this in more detail. He seems content with decoherence as 
delocalization of the phases into the environment. But Zurek is quite 
critical of ending one's account there: "Popular accounts of decoherence 
... often start from the observation that when a quantum system S 
interacts with some environment E 'phase relations in S are lost'. This 
is a caricature, at best incomplete if not misleading: It begs the 
question: 'Phases between what?'. This in turn leads directly to the 
main issue addressed by einselection: 'What is the preferred basis?'. 
This question is often muddled in 'folklore' accounts of decoherence." 
(arXiv:0707.2832)


Zurek here hits the nail on the head: loss of phase relations does not 
explain what are the basis states between which phase relations are 
lost. Any more complete account must explain how preferred basis states 
emerge. Anthropic accounts just appeal to magic for this -- evolution is 
not a dynamical account of the emergence of a preferred basis. According 
to Zurek, preferred states of quantum systems emerge from the dynamics. 
In that he cannot be faulted -- there is no other non-magical way in 
which this could happen.


Schlosshauer gives a more complete account of the these dynamics than is 
given by Zurek. (Schlosshauer, arXiv:1404.2635) In summary, Schlosshauer 
points to the fact that basis states that are stable against decoherence 
are those for which the corresponding quantum operator commutes with the 
interaction Hamiltonian. I think that this is both a necessary and 
sufficient condition for states to be stable against decoherence. But 
even then, it is not an entirely satisfactory dynamical account. The 
problem is that for position measurements, for example, the stable 
states are eigenstates of the position operator, but the interaction 
Hamiltonian is generally given by point particle interactions, and those 
are themselves defined in terms of the same position operator. This is 
circular -- the same position operator defines both the basis states and 
the interaction Hamiltonian so they necessarily commute: But that would 
be true no matter what the position operator was -- provided one used 
the same operator to define both the states and the Hamiltonian. So this 
does not rule out alternative position operators which would have 
different sets of basis states, given by superpositions of our usual 
basis states. This does not, therefore, explain why we do not see 
superpositions of dots on the screen in position measurements, or 
superpositions of classical pointer readings.


I do not know the answer to this problem. I do not think that it has 
really been addressed by either Zurek or Schlosshauer. Clearly, the 
dynamics must be central to the emergence of preferred basis states, but 
I do not see a my way to a non-circular account of this. We might, 
unfortunately, be up against a 'brute fact' that has no more fundamental 
explanation. Alternatively, the answer might lie in a full quantum 
understanding of the nature of space itself -- somehow the underlying 
nature of spacetime determines that the interaction dynamics will be in 
terms of a position operator whose eigenstates are delta functions on 
the real line. Some such explanation is required.



but when I do that I eventually put myself on a slope leading to the 
problem of marrying the quantum and gravity, a nightmare from which I 
come back to mechanism rather quickly :)


I seem to have ended up with the problem of giving a quantum account of 
gravity as well. But I do not think that mechanism is going to gave you 
an answer to this.


Bruce

--
You received this message because you are subscribed to the Google Groups 
"Everything List" group.
To unsubscribe from this group and stop receiving emails from it, send an email 
to 

Re: Entanglement

2018-06-11 Thread Brent Meeker



On 6/11/2018 6:26 PM, agrayson2...@gmail.com wrote:





On Monday, June 11, 2018 at 10:57:59 PM UTC, Brent wrote:



On 6/11/2018 3:22 PM, agrays...@gmail.com  wrote:



I am not sure this make sense (with the SWE). The cat is
always isolated, in some sense.

*
IMO totally wrong. In fact now you're contradicting what you
wrote in a recent post. The cat is NEVER ISOLATED, VIRTUALLY
BY DEFINITION OF WHAT MACRO MEANS. NEVER ISOLATED IMPLIES
NEVER IN A SUPERPOSITION. AG
*

*
In the real world the cat is never isolated, nor can it be
isolated insofar as it consists of a huge number of particles
already entangled with its environment. This is the meaning of
"macro" ! If you insist on imagining it as isolated for your
thought experiment, you will generate a paradox, as Schroedinger
did.  AG*


Schroedinger obviously chose a cat to emphasize the absurdity, but
it also makes the analysis hard to think about.  Not only is the
cat macroscopic, the atomic decay is distributed over a continuum.
I think it helps to think of a simpler experiment in which the
atom is just in a box which is lined with photographic plates.  So
the atom is in a superposition of undecayed and decayed and
interacted with a silver halide atom.  It is clear that it is the
interaction with the silver halide atom that gets amplified to a
macroscopic dot of silver which decoheres the system in orthogonal
"worlds" in which the spot is in different places and happens at
different times.

Brent


*I don't see how a single silver halide atom is amplified to a 
macroscopic dot of silver. *


That's because you don't bother to look up anything.  You expect other 
people to look it up for you and then explain it to you. Here's 
Wikipedia, but I'm not explaining it.


/Silver halides are used in photographic film and photographic paper, 
including graphic art film and paper, where silver halide crystals in 
gelatin are coated on to a film base, glass or paper substrate. The 
gelatin is a vital part of the emulsion as the protective colloid of 
appropriate physical and chemical properties. Gelatin may also contain 
trace elements (such as sulfur) which increase the light sensitivity of 
the emulsion, although modern practice uses gelatin without such 
components. When absorbed by an AgX crystal, photons cause electrons to 
be promoted to a conduction band (de-localized electron orbital with 
higher energy than a valence band) which can be attracted by a 
sensitivity speck, which is a shallow electron trap, which may be a 
crystalline defect or a cluster of silver sulfide, gold, other trace 
elements (dopant), or combination thereof, and then combined with an 
interstitial silver ion to form silver metal speck.[1]//

//
//When a silver halide crystal is exposed to light, a sensitivity speck 
on the surface of the crystal is turned into a small speck of metallic 
silver (these comprise the invisible or latent image). If the speck of 
silver contains approximately four or more atoms, it is rendered 
developable - meaning that it can undergo development which turns the 
entire crystal into metallic silver.


/Brent/
/
*Going back to my analysis, I think I have shown the fallacy of using 
a macro entities in a superposition (since they can never be 
isolated). But this is the starting point of decoherence theory, as 
exemplified by the wf Bruce recently presented for a spin 1/2 
measurement (where the apparatus, observer and remaining environment 
appear in the superposition). AG*

--
You received this message because you are subscribed to the Google 
Groups "Everything List" group.
To unsubscribe from this group and stop receiving emails from it, send 
an email to everything-list+unsubscr...@googlegroups.com 
.
To post to this group, send email to everything-list@googlegroups.com 
.

Visit this group at https://groups.google.com/group/everything-list.
For more options, visit https://groups.google.com/d/optout.


--
You received this message because you are subscribed to the Google Groups 
"Everything List" group.
To unsubscribe from this group and stop receiving emails from it, send an email 
to everything-list+unsubscr...@googlegroups.com.
To post to this group, send email to everything-list@googlegroups.com.
Visit this group at https://groups.google.com/group/everything-list.
For more options, visit https://groups.google.com/d/optout.


Re: Entanglement

2018-06-11 Thread agrayson2000




On Monday, June 11, 2018 at 10:57:59 PM UTC, Brent wrote:
>
>
>
> On 6/11/2018 3:22 PM, agrays...@gmail.com  wrote:
>
>
>>> I am not sure this make sense (with the SWE). The cat is always 
>>> isolated, in some sense. 
>>>
>>
>>
>> * IMO totally wrong. In fact now you're contradicting what you wrote in a 
>> recent post. The cat is NEVER ISOLATED, VIRTUALLY BY DEFINITION OF WHAT 
>> MACRO MEANS. NEVER ISOLATED IMPLIES NEVER IN A SUPERPOSITION. AG *
>>
>
> * In the real world the cat is never isolated, nor can it be isolated 
> insofar as it consists of a huge number of particles already entangled with 
> its environment. This is the meaning of "macro" ! If you insist on 
> imagining it as isolated for your thought experiment, you will generate a 
> paradox, as Schroedinger did.  AG*
>
>
> Schroedinger obviously chose a cat to emphasize the absurdity, but it also 
> makes the analysis hard to think about.  Not only is the cat macroscopic, 
> the atomic decay is distributed over a continuum. I think it helps to think 
> of a simpler experiment in which the atom is just in a box which is lined 
> with photographic plates.  So the atom is in a superposition of undecayed 
> and decayed and interacted with a silver halide atom.  It is clear that it 
> is the interaction with the silver halide atom that gets amplified to a 
> macroscopic dot of silver which decoheres the system in orthogonal "worlds" 
> in which the spot is in different places and happens at different times.
>
> Brent
>

*I don't see how a single silver halide atom is amplified to a macroscopic 
dot of silver. Going back to my analysis, I think I have shown the fallacy 
of using a macro entities in a superposition (since they can never be 
isolated). But this is the starting point of decoherence theory, as 
exemplified by the wf Bruce recently presented for a spin 1/2 measurement 
(where the apparatus, observer and remaining environment appear in the 
superposition). AG*

-- 
You received this message because you are subscribed to the Google Groups 
"Everything List" group.
To unsubscribe from this group and stop receiving emails from it, send an email 
to everything-list+unsubscr...@googlegroups.com.
To post to this group, send email to everything-list@googlegroups.com.
Visit this group at https://groups.google.com/group/everything-list.
For more options, visit https://groups.google.com/d/optout.


Re: Entanglement

2018-06-11 Thread Brent Meeker



On 6/11/2018 3:22 PM, agrayson2...@gmail.com wrote:



I am not sure this make sense (with the SWE). The cat is
always isolated, in some sense.

*
IMO totally wrong. In fact now you're contradicting what you wrote
in a recent post. The cat is NEVER ISOLATED, VIRTUALLY BY
DEFINITION OF WHAT MACRO MEANS. NEVER ISOLATED IMPLIES NEVER IN A
SUPERPOSITION. AG
*

*
In the real world the cat is never isolated, nor can it be isolated 
insofar as it consists of a huge number of particles already entangled 
with its environment. This is the meaning of "macro" ! If you insist 
on imagining it as isolated for your thought experiment, you will 
generate a paradox, as Schroedinger did.  AG*


Schroedinger obviously chose a cat to emphasize the absurdity, but it 
also makes the analysis hard to think about.  Not only is the cat 
macroscopic, the atomic decay is distributed over a continuum. I think 
it helps to think of a simpler experiment in which the atom is just in a 
box which is lined with photographic plates.  So the atom is in a 
superposition of undecayed and decayed and interacted with a silver 
halide atom.  It is clear that it is the interaction with the silver 
halide atom that gets amplified to a macroscopic dot of silver which 
decoheres the system in orthogonal "worlds" in which the spot is in 
different places and happens at different times.


Brent

--
You received this message because you are subscribed to the Google Groups 
"Everything List" group.
To unsubscribe from this group and stop receiving emails from it, send an email 
to everything-list+unsubscr...@googlegroups.com.
To post to this group, send email to everything-list@googlegroups.com.
Visit this group at https://groups.google.com/group/everything-list.
For more options, visit https://groups.google.com/d/optout.


Re: Entanglement

2018-06-11 Thread agrayson2000


On Monday, June 11, 2018 at 7:39:45 PM UTC, agrays...@gmail.com wrote:
>
>
>
> On Monday, June 11, 2018 at 4:19:34 PM UTC, Bruno Marchal wrote:
>>
>>
>> On 11 Jun 2018, at 12:59, agrays...@gmail.com wrote:
>>
>>
>>
>> On Monday, June 11, 2018 at 10:40:13 AM UTC, Bruno Marchal wrote:
>>>
>>>
>>> On 11 Jun 2018, at 07:06, agrays...@gmail.com wrote:
>>>
>>>
>>>
>>> On Monday, June 11, 2018 at 2:20:47 AM UTC, agrays...@gmail.com wrote:



 On Monday, June 11, 2018 at 2:09:25 AM UTC, Bruce wrote:
>
> From: 
>
> On Monday, June 11, 2018 at 1:37:53 AM UTC, Bruce wrote: 
>>
>> From: Bruno Marchal > Everett prove the contrary, and he convinced me when I read it. I 
>> found “his proof” used in many books on quantum computing, although with 
>> different motivation. Thee result of an experiment, obviously depend of 
>> what you measure, but when you embed the observer in the wave, you get 
>> that 
>> what they find is independent of the choice of the base used to describe 
>> the “observer” and the “observed”. If not, the MW would already be 
>> refuted.
>>
>>
>> In that case, MW is refuted. Clearly, what the observer finds is 
>> dependent on the basis in which he is described. Or else experiments 
>> would 
>> not have definite results when described in the laboratory from the 1p 
>> perspective. Even if you take the 'bird' view of the whole multiverse -- 
>> which is, I agree, independent of the basis in which it is described -- 
>> the 
>> view of any observer embedded in the multiverse is totally 
>> basis-dependent. 
>> That is, after all, what we mean by 'worlds' -- the view from within, or 
>> the 1p view. But that view depends on how you describe it: the way in 
>> which 
>> you partition the multiverse itself. Only certain very special bases are 
>> robust against environmental decoherence -- how else do you resolve the 
>> Schrödinger cat issue?
>>
>> Bruce
>>
>
> *So you find the resolution in the fact that according to decoherence 
> theory, the cat is simultaneously alive and dead for only short time?  AG*
>
>
> Decoherence has resolved the basis question long before the cyanide 
> has hit the cat.
>
> Bruce
>

 *I don't think you've answered the question. Isn't the cat in a 
 superposition of alive and dead before the cyanide hits? Did Schroedinger 
 write an incorrect wf? If so, what is the correct one IYO? AG *

>>>
>>> *I surmise your position is that decoherence happens so quickly, that 
>>> the superposition Schroedinger wrote was really a mixed state. If so, I 
>>> don't see this as a solution to the paradox, unless you want to allow the 
>>> existence of a simultaneously alive and dead cat for a very, very short 
>>> time. AG* 
>>>
>>>
>>>
>>> That is why I prefer Bohm’s version of the cat, where the cat alive/dead 
>>> state is corrupted with the up/down state of some particles. It ease the 
>>> mind by showing that the time is not an issue. If you can completely 
>>> isolate the cat from the environment (which is technically impossible), you 
>>> can maintain the cat in the dead + alive superposition state as long as you 
>>> want. If you isolate successfully the entire laboratory including you, 
>>> Then, someone else can resurrect the cat, relatively to himself, despite 
>>> you saw it dead. 
>>>
>>> The reason why we cannot do this in principle, is that we cannot isolate 
>>> the cat, and if the cat, when the cat is dead+alive, interact with some 
>>> particles in the environment, you can no mare factorize the cat state, 
>>> without tracking that particles.
>>>
>>> I don’t think it make sense to confine the superposition in the 
>>> microscopic domain, nor in the short-time domain. If the SWE is correct, 
>>> the superposition never disappear, unless a collapse assumption is made, 
>>> but then it cannot be described by QM. Only by QM + exception rules for the 
>>> observer or the measuring apparatus, but there are no evidences for that.
>>>
>>> Bruno
>>>
>>
>> *See my solution to the S Cat on the other thread.  Since the cat can 
>> never be isolated, it can never be in a superposition, which generates the 
>> paradox. And since coherence can never occur, no need to apply 
>> decoherence!  AG*
>>
>>
>>
>> I am not sure this make sense (with the SWE). The cat is always isolated, 
>> in some sense. 
>>
>
>
> *IMO totally wrong. In fact now you're contradicting what you wrote in a 
> recent post. The cat is NEVER ISOLATED, VIRTUALLY BY DEFINITION OF WHAT 
> MACRO MEANS. NEVER ISOLATED IMPLIES NEVER IN A SUPERPOSITION. AG*
>

*In the real world the cat is never isolated, nor can it be isolated 
insofar as it consists of a huge number of particles already entangled with 
its environment. This is the meaning of "macro" ! If you insist on 
imagining it as isolated for your thought 

Re: Entanglement

2018-06-11 Thread agrayson2000


On Monday, June 11, 2018 at 4:19:34 PM UTC, Bruno Marchal wrote:
>
>
> On 11 Jun 2018, at 12:59, agrays...@gmail.com  wrote:
>
>
>
> On Monday, June 11, 2018 at 10:40:13 AM UTC, Bruno Marchal wrote:
>>
>>
>> On 11 Jun 2018, at 07:06, agrays...@gmail.com wrote:
>>
>>
>>
>> On Monday, June 11, 2018 at 2:20:47 AM UTC, agrays...@gmail.com wrote:
>>>
>>>
>>>
>>> On Monday, June 11, 2018 at 2:09:25 AM UTC, Bruce wrote:

 From: 

 On Monday, June 11, 2018 at 1:37:53 AM UTC, Bruce wrote: 
>
> From: Bruno Marchal  Everett prove the contrary, and he convinced me when I read it. I 
> found “his proof” used in many books on quantum computing, although with 
> different motivation. Thee result of an experiment, obviously depend of 
> what you measure, but when you embed the observer in the wave, you get 
> that 
> what they find is independent of the choice of the base used to describe 
> the “observer” and the “observed”. If not, the MW would already be 
> refuted.
>
>
> In that case, MW is refuted. Clearly, what the observer finds is 
> dependent on the basis in which he is described. Or else experiments 
> would 
> not have definite results when described in the laboratory from the 1p 
> perspective. Even if you take the 'bird' view of the whole multiverse -- 
> which is, I agree, independent of the basis in which it is described -- 
> the 
> view of any observer embedded in the multiverse is totally 
> basis-dependent. 
> That is, after all, what we mean by 'worlds' -- the view from within, or 
> the 1p view. But that view depends on how you describe it: the way in 
> which 
> you partition the multiverse itself. Only certain very special bases are 
> robust against environmental decoherence -- how else do you resolve the 
> Schrödinger cat issue?
>
> Bruce
>

 *So you find the resolution in the fact that according to decoherence 
 theory, the cat is simultaneously alive and dead for only short time?  AG*


 Decoherence has resolved the basis question long before the cyanide has 
 hit the cat.

 Bruce

>>>
>>> *I don't think you've answered the question. Isn't the cat in a 
>>> superposition of alive and dead before the cyanide hits? Did Schroedinger 
>>> write an incorrect wf? If so, what is the correct one IYO? AG *
>>>
>>
>> *I surmise your position is that decoherence happens so quickly, that the 
>> superposition Schroedinger wrote was really a mixed state. If so, I don't 
>> see this as a solution to the paradox, unless you want to allow the 
>> existence of a simultaneously alive and dead cat for a very, very short 
>> time. AG* 
>>
>>
>>
>> That is why I prefer Bohm’s version of the cat, where the cat alive/dead 
>> state is corrupted with the up/down state of some particles. It ease the 
>> mind by showing that the time is not an issue. If you can completely 
>> isolate the cat from the environment (which is technically impossible), you 
>> can maintain the cat in the dead + alive superposition state as long as you 
>> want. If you isolate successfully the entire laboratory including you, 
>> Then, someone else can resurrect the cat, relatively to himself, despite 
>> you saw it dead. 
>>
>> The reason why we cannot do this in principle, is that we cannot isolate 
>> the cat, and if the cat, when the cat is dead+alive, interact with some 
>> particles in the environment, you can no mare factorize the cat state, 
>> without tracking that particles.
>>
>> I don’t think it make sense to confine the superposition in the 
>> microscopic domain, nor in the short-time domain. If the SWE is correct, 
>> the superposition never disappear, unless a collapse assumption is made, 
>> but then it cannot be described by QM. Only by QM + exception rules for the 
>> observer or the measuring apparatus, but there are no evidences for that.
>>
>> Bruno
>>
>
> *See my solution to the S Cat on the other thread.  Since the cat can 
> never be isolated, it can never be in a superposition, which generates the 
> paradox. And since coherence can never occur, no need to apply 
> decoherence!  AG*
>
>
>
> I am not sure this make sense (with the SWE). The cat is always isolated, 
> in some sense. 
>



*IMO totally wrong. In fact now you're contradicting what you wrote in a 
recent post. The cat is NEVER ISOLATED, VIRTUALLY BY DEFINITION OF WHAT 
MACRO MEANS. NEVER ISOLATED IMPLIES NEVER IN A SUPERPOSITION. AG*[snip]

-- 
You received this message because you are subscribed to the Google Groups 
"Everything List" group.
To unsubscribe from this group and stop receiving emails from it, send an email 
to everything-list+unsubscr...@googlegroups.com.
To post to this group, send email to everything-list@googlegroups.com.
Visit this group at https://groups.google.com/group/everything-list.
For more options, visit https://groups.google.com/d/optout.


Re: Entanglement

2018-06-11 Thread Bruno Marchal

> On 11 Jun 2018, at 14:48, Bruce Kellett  wrote:
> 
> From: Bruno Marchal mailto:marc...@ulb.ac.be>>
>>> On 11 Jun 2018, at 03:37, Bruce Kellett < 
>>> bhkell...@optusnet.com.au 
>>> > wrote:
>>> 
>>> From: Bruno Marchal mailto:marc...@ulb.ac.be>
> On 8 Jun 2018, at 14:55, Bruce Kellett < 
> bhkell...@optusnet.com.au 
> > wrote:
> 
> The choice of basis makes all the difference in the world.
 
 Everett prove the contrary, and he convinced me when I read it. I found 
 “his proof” used in many books on quantum computing, although with 
 different motivation. Thee result of an experiment, obviously depend of 
 what you measure, but when you embed the observer in the wave, you get 
 that what they find is independent of the choice of the base used to 
 describe the “observer” and the “observed”. If not, the MW would already 
 be refuted.
>>> 
>>> In that case, MW is refuted. Clearly, what the observer finds is dependent 
>>> on the basis in which he is described.
>> 
>> ?
>> 
>> I disagree. The finding can depend on what the observer decide to measure, 
>> which is akin to choosing a base,
> 
> I think there is more to it than this. We can choose a base in which to 
> describe the state, but we are not able to choose the base for the actual 
> measurement result.

I am not sure I understand. It seems to me that the base for the measurement 
result will be decided by the observer (already or later, even at the last 
moment, perhaps, like in Aspect experience).



> In a spin measurement of a spin half particle, we can decide to orient the 
> magnet at any angle to the direction of motion. For convenience in 
> understanding the dynamics, we then expand the spin wave function in the 
> basis corresponding to that orientation; that is choosing what to measure.

Yes.


> But the result of our measurement in that orientation is that the particle 
> emerges from the S-G magnet on either the up or down trajectory.

OK.


> The actual measurement is then made at screen downstream of the magnet. We do 
> not have any control over the basis for the resulting position measurement.

Ah! If that is what you mean, I am OK. But that will be a slight dispersion. Of 
course some electron, going to down could appears on the up, because the 
“position-wave" are spread, but that should be negligible with the down/up spin 
prediction.



> Decoherence decides that for us, and the stable basis for position 
> measurements is the set of delta functions at each point along the spatial 
> axis. Why do we not see the 'up' or 'down' result as a superposition of a lot 
> of different positions on the screen? We don't because such superpositions 
> are not stable under decoherence.

I will have to meditate on this. I take that decoherence is already in Everett, 
and is simply entanglement. The why our brain prefer position is a refinement, 
by Zeh indeed, and Zurek. I have not yet completely solved all problems raised 
by this. 



> 
>> but the couple “observer + that chosen base” can be studied in any base, and 
>> the same result will described in the memories of the observer. I will 
>> search Everett proofs of this, as he is the one who convinced me on this.
> 
> The situation as I understand it is the following. The original wave function 
> |psi> can be expanded in any basis that spans the corresponding Hilbert 
> space. So we can have
> 
>  |psi> = Sum_i c_i |a_i>
>   = Sum_j d_j |b_j>,
> 
> where the |a_i> and |b_j> are sets of vectors which independently span the 
> space. The expansion coefficients are different, so in general the c_i =/=  
> d_j.
> 
> If we measure in one basis, say that of the |a_i>, the result is one of the 
> |a_i> with probability |c_i|^2. However, if the measurement corresponds to 
> the other basis (that will be a different operator in the Hilbert space), we 
> will, in general, get a different result, one of the |b_j> with probability 
> |d_j|^2.

OK.




> The results of these measurement will be different. If we now decohere these 
> states with the environment, in one case the environment will be entangled 
> with the |a_i> states, and in the other case, the environment (including the 
> observer) will be entangled with the |b_j> states.

OK. But here, the choice of the measurement just determines the future results 
of the measurement for the observer in some branches. The choice of the base 
determines our possible continuations. 



> So the observer will see different things according to the basis in which he 
> is working and the measurement made.

In that sense above, I agree. 

My sense is well explained by Everett in the Graham DeWitt selected papers, 
page 38.




> 
> Now since the bases both relate to the same Hilbert space, and the same 
> original state is expanded, the basis vectors 

Re: Entanglement

2018-06-11 Thread Bruno Marchal

> On 11 Jun 2018, at 12:59, agrayson2...@gmail.com wrote:
> 
> 
> 
> On Monday, June 11, 2018 at 10:40:13 AM UTC, Bruno Marchal wrote:
> 
>> On 11 Jun 2018, at 07:06, agrays...@gmail.com  wrote:
>> 
>> 
>> 
>> On Monday, June 11, 2018 at 2:20:47 AM UTC, agrays...@gmail.com 
>>  wrote:
>> 
>> 
>> On Monday, June 11, 2018 at 2:09:25 AM UTC, Bruce wrote:
>> From: >
>>> On Monday, June 11, 2018 at 1:37:53 AM UTC, Bruce wrote:
>>> From: Bruno Marchal 
>>> Everett prove the contrary, and he convinced me when I read it. I found 
>>> “his proof” used in many books on quantum computing, although with 
>>> different motivation. Thee result of an experiment, obviously depend of 
>>> what you measure, but when you embed the observer in the wave, you get that 
>>> what they find is independent of the choice of the base used to describe 
>>> the “observer” and the “observed”. If not, the MW would already be refuted.
>>> 
>>> In that case, MW is refuted. Clearly, what the observer finds is dependent 
>>> on the basis in which he is described. Or else experiments would not have 
>>> definite results when described in the laboratory from the 1p perspective. 
>>> Even if you take the 'bird' view of the whole multiverse -- which is, I 
>>> agree, independent of the basis in which it is described -- the view of any 
>>> observer embedded in the multiverse is totally basis-dependent. That is, 
>>> after all, what we mean by 'worlds' -- the view from within, or the 1p 
>>> view. But that view depends on how you describe it: the way in which you 
>>> partition the multiverse itself. Only certain very special bases are robust 
>>> against environmental decoherence -- how else do you resolve the 
>>> Schrödinger cat issue?
>>> 
>>> Bruce
>>> 
>>> So you find the resolution in the fact that according to decoherence 
>>> theory, the cat is simultaneously alive and dead for only short time?  AG
>> 
>> Decoherence has resolved the basis question long before the cyanide has hit 
>> the cat.
>> 
>> Bruce
>> 
>> I don't think you've answered the question. Isn't the cat in a superposition 
>> of alive and dead before the cyanide hits? Did Schroedinger write an 
>> incorrect wf? If so, what is the correct one IYO? AG
>> 
>> I surmise your position is that decoherence happens so quickly, that the 
>> superposition Schroedinger wrote was really a mixed state. If so, I don't 
>> see this as a solution to the paradox, unless you want to allow the 
>> existence of a simultaneously alive and dead cat for a very, very short 
>> time. AG 
> 
> 
> That is why I prefer Bohm’s version of the cat, where the cat alive/dead 
> state is corrupted with the up/down state of some particles. It ease the mind 
> by showing that the time is not an issue. If you can completely isolate the 
> cat from the environment (which is technically impossible), you can maintain 
> the cat in the dead + alive superposition state as long as you want. If you 
> isolate successfully the entire laboratory including you, Then, someone else 
> can resurrect the cat, relatively to himself, despite you saw it dead. 
> 
> The reason why we cannot do this in principle, is that we cannot isolate the 
> cat, and if the cat, when the cat is dead+alive, interact with some particles 
> in the environment, you can no mare factorize the cat state, without tracking 
> that particles.
> 
> I don’t think it make sense to confine the superposition in the microscopic 
> domain, nor in the short-time domain. If the SWE is correct, the 
> superposition never disappear, unless a collapse assumption is made, but then 
> it cannot be described by QM. Only by QM + exception rules for the observer 
> or the measuring apparatus, but there are no evidences for that.
> 
> Bruno
> 
> See my solution to the S Cat on the other thread.  Since the cat can never be 
> isolated, it can never be in a superposition, which generates the paradox. 
> And since coherence can never occur, no need to apply decoherence!  AG


I am not sure this make sense (with the SWE). The cat is always isolated, in 
some sense. Let me prepare an electron in the state (u + d) (the superposition 
of up + down, I will not use Dirac notation, because it is not easy in mail). 
Then I put the cat near a device which kill it if the electron is down, and let 
him alive if the spin is up. 

What happens is this. In between the cat and me there are particles, some 
bouncing on the cat and other, soon or later bounding on some particles having 
bounce with the cat, or with a particles having bounced … etc.

Let us simplify, and consider that there is only one such particles, and it 
goes in the state - if the cat is dead, and in the state + if the cat is alive.

The cat is describe by C, and C°° described the cat alive, and C° describes the 
cat dead.

We have the overall state (in the box if you want) 

C(u+d)

By linearity of the tensor product (used here to describe a multi-body system), 
that is the same as the state


Re: Entanglement

2018-06-11 Thread Bruce Kellett

From: *Bruno Marchal* mailto:marc...@ulb.ac.be>>
On 11 Jun 2018, at 03:37, Bruce Kellett > wrote:


From: *Bruno Marchal* mailto:marc...@ulb.ac.be>
On 8 Jun 2018, at 14:55, Bruce Kellett > wrote:


The choice of basis makes all the difference in the world.


Everett prove the contrary, and he convinced me when I read it. I 
found “his proof” used in many books on quantum computing, although 
with different motivation. Thee result of an experiment, obviously 
depend of what you measure, but when you embed the observer in the 
wave, you get that what they find is independent of the choice of 
the base used to describe the “observer” and the “observed”. If not, 
the MW would already be refuted.


In that case, MW is refuted. Clearly, what the observer finds is 
dependent on the basis in which he is described.


?

I disagree. The finding can depend on what the observer decide to 
measure, which is akin to choosing a base,


I think there is more to it than this. We can choose a base in which to 
describe the state, but we are not able to choose the base for the 
actual measurement result. In a spin measurement of a spin half 
particle, we can decide to orient the magnet at any angle to the 
direction of motion. For convenience in understanding the dynamics, we 
then expand the spin wave function in the basis corresponding to that 
orientation; that is choosing what to measure. But the result of our 
measurement in that orientation is that the particle emerges from the 
S-G magnet on either the up or down trajectory. The actual measurement 
is then made at screen downstream of the magnet. We do not have any 
control over the basis for the resulting position measurement. 
Decoherence decides that for us, and the stable basis for position 
measurements is the set of delta functions at each point along the 
spatial axis. Why do we not see the 'up' or 'down' result as a 
superposition of a lot of different positions on the screen? We don't 
because such superpositions are not stable under decoherence.


but the couple “observer + that chosen base” can be studied in any 
base, and the same result will described in the memories of the 
observer. I will search Everett proofs of this, as he is the one who 
convinced me on this.


The situation as I understand it is the following. The original wave 
function |psi> can be expanded in any basis that spans the corresponding 
Hilbert space. So we can have


 |psi> = Sum_i c_i |a_i>
  = Sum_j d_j |b_j>,

where the |a_i> and |b_j> are sets of vectors which independently span 
the space. The expansion coefficients are different, so in general the 
c_i =/=  d_j.


If we measure in one basis, say that of the |a_i>, the result is one of 
the |a_i> with probability |c_i|^2. However, if the measurement 
corresponds to the other basis (that will be a different operator in the 
Hilbert space), we will, in general, get a different result, one of the 
|b_j> with probability |d_j|^2. The results of these measurement will be 
different. If we now decohere these states with the environment, in one 
case the environment will be entangled with the |a_i> states, and in the 
other case, the environment (including the observer) will be entangled 
with the |b_j> states. So the observer will see different things 
according to the basis in which he is working and the measurement made.


Now since the bases both relate to the same Hilbert space, and the same 
original state is expanded, the basis vectors (and the expansion 
coefficients) are related, so we can always express one set in terms of 
the other:


 |a_i> = Sum_j f_{ij} |b_j>.

So in this sense, the basis chosen does not matter in the overall 
description. But once we take environmental decoherence into account, 
only one basis will be stable, say the |a_i> basis, and if we describe 
things in the |b_j> basis, those basis states are immediately decohered 
into the corresponding |a_i> states. Consequently, any observer will 
only ever see results, or be located in branches (worlds), corresponding 
the the stable decohered base. Describing things in terms of another 
base doesn't change the reality, just as describing the orbit of the 
Moon in terms of a coordinate system based on Jupiter doesn't actually 
change the orbit of the Moon, it just makes the description a lot more 
complicated. However, in the absence of decoherence to a preferred 
stable basis, the results in the }a_i> and }b_j> bases are different.





Or else experiments would not have definite results when described in 
the laboratory from the 1p perspective.


I don’t see why.


Without decoherence, measurements in the |b_j> basis are superpositions 
of the |a_i> states, and by construction, only the |a_i> states are 
stable under decoherence, corresponding to definite results.




Even if you take the 'bird' view of the whole multiverse -- which is, 
I agree, independent of the 

Re: Entanglement

2018-06-11 Thread agrayson2000


On Monday, June 11, 2018 at 10:40:13 AM UTC, Bruno Marchal wrote:
>
>
> On 11 Jun 2018, at 07:06, agrays...@gmail.com  wrote:
>
>
>
> On Monday, June 11, 2018 at 2:20:47 AM UTC, agrays...@gmail.com wrote:
>>
>>
>>
>> On Monday, June 11, 2018 at 2:09:25 AM UTC, Bruce wrote:
>>>
>>> From: 
>>>
>>> On Monday, June 11, 2018 at 1:37:53 AM UTC, Bruce wrote: 

 From: Bruno Marchal >>> Everett prove the contrary, and he convinced me when I read it. I found 
 “his proof” used in many books on quantum computing, although with 
 different motivation. Thee result of an experiment, obviously depend of 
 what you measure, but when you embed the observer in the wave, you get 
 that 
 what they find is independent of the choice of the base used to describe 
 the “observer” and the “observed”. If not, the MW would already be refuted.


 In that case, MW is refuted. Clearly, what the observer finds is 
 dependent on the basis in which he is described. Or else experiments would 
 not have definite results when described in the laboratory from the 1p 
 perspective. Even if you take the 'bird' view of the whole multiverse -- 
 which is, I agree, independent of the basis in which it is described -- 
 the 
 view of any observer embedded in the multiverse is totally 
 basis-dependent. 
 That is, after all, what we mean by 'worlds' -- the view from within, or 
 the 1p view. But that view depends on how you describe it: the way in 
 which 
 you partition the multiverse itself. Only certain very special bases are 
 robust against environmental decoherence -- how else do you resolve the 
 Schrödinger cat issue?

 Bruce

>>>
>>> *So you find the resolution in the fact that according to decoherence 
>>> theory, the cat is simultaneously alive and dead for only short time?  AG*
>>>
>>>
>>> Decoherence has resolved the basis question long before the cyanide has 
>>> hit the cat.
>>>
>>> Bruce
>>>
>>
>> *I don't think you've answered the question. Isn't the cat in a 
>> superposition of alive and dead before the cyanide hits? Did Schroedinger 
>> write an incorrect wf? If so, what is the correct one IYO? AG *
>>
>
> *I surmise your position is that decoherence happens so quickly, that the 
> superposition Schroedinger wrote was really a mixed state. If so, I don't 
> see this as a solution to the paradox, unless you want to allow the 
> existence of a simultaneously alive and dead cat for a very, very short 
> time. AG* 
>
>
>
> That is why I prefer Bohm’s version of the cat, where the cat alive/dead 
> state is corrupted with the up/down state of some particles. It ease the 
> mind by showing that the time is not an issue. If you can completely 
> isolate the cat from the environment (which is technically impossible), you 
> can maintain the cat in the dead + alive superposition state as long as you 
> want. If you isolate successfully the entire laboratory including you, 
> Then, someone else can resurrect the cat, relatively to himself, despite 
> you saw it dead. 
>
> The reason why we cannot do this in principle, is that we cannot isolate 
> the cat, and if the cat, when the cat is dead+alive, interact with some 
> particles in the environment, you can no mare factorize the cat state, 
> without tracking that particles.
>
> I don’t think it make sense to confine the superposition in the 
> microscopic domain, nor in the short-time domain. If the SWE is correct, 
> the superposition never disappear, unless a collapse assumption is made, 
> but then it cannot be described by QM. Only by QM + exception rules for the 
> observer or the measuring apparatus, but there are no evidences for that.
>
> Bruno
>

*See my solution to the S Cat on the other thread.  Since the cat can never 
be isolated, it can never be in a superposition, which generates the 
paradox. And since coherence can never occur, no need to apply 
decoherence!  AG*

-- 
You received this message because you are subscribed to the Google Groups 
"Everything List" group.
To unsubscribe from this group and stop receiving emails from it, send an email 
to everything-list+unsubscr...@googlegroups.com.
To post to this group, send email to everything-list@googlegroups.com.
Visit this group at https://groups.google.com/group/everything-list.
For more options, visit https://groups.google.com/d/optout.


Re: Entanglement

2018-06-11 Thread Bruno Marchal

> On 11 Jun 2018, at 08:56, agrayson2...@gmail.com wrote:
> 
> [big snip]
> 
> For Bruno:
> 
> On Monday, June 11, 2018 at 6:50:51 AM UTC, agrays...@gmail.com wrote:
> 
> Thanks for the data dump. It's way above my head, so not so far above that I 
> can't see the virtue of using arithmetic logic as a starting point for a new 
> take on reality. I might buy the Kindle version of your book, translated by 
> Russell.  You might be wrong, but I give you credit for tackling the arguably 
> most intractable problem; the mind-body problem. Keep in truckin'! AG
> 
> That should be, Keep ON truckin'! AG 

Thanks. I learn English on this list!

B.



> 
> -- 
> You received this message because you are subscribed to the Google Groups 
> "Everything List" group.
> To unsubscribe from this group and stop receiving emails from it, send an 
> email to everything-list+unsubscr...@googlegroups.com 
> .
> To post to this group, send email to everything-list@googlegroups.com 
> .
> Visit this group at https://groups.google.com/group/everything-list 
> .
> For more options, visit https://groups.google.com/d/optout 
> .

-- 
You received this message because you are subscribed to the Google Groups 
"Everything List" group.
To unsubscribe from this group and stop receiving emails from it, send an email 
to everything-list+unsubscr...@googlegroups.com.
To post to this group, send email to everything-list@googlegroups.com.
Visit this group at https://groups.google.com/group/everything-list.
For more options, visit https://groups.google.com/d/optout.


Re: Entanglement

2018-06-11 Thread Bruno Marchal

> On 11 Jun 2018, at 08:50, agrayson2...@gmail.com wrote:
> 
> 
> 
> On Sunday, June 10, 2018 at 3:19:37 PM UTC, Bruno Marchal wrote:
> 
>> On 9 Jun 2018, at 01:10, agrays...@gmail.com  wrote:
>> 
>> 
>> 
>> On Friday, June 8, 2018 at 12:06:33 PM UTC, Bruno Marchal wrote:
>> 
>> On 8 Jun 2018, at 03:30, agrays...@gmail.com <> wrote:
>> 
>> On Thursday, June 7, 2018 at 9:07:37 AM UTC, Bruno Marchal wrote:
>> 
>> [snip]
>> 
>>So consciousness anticipates all quantum experiment that MIGHT occur 
>> in the future,
>> 
>> The arithmetical relations do that. Consciousness only select the histories
>> 
>> and creates those worlds in anticipation? Now we're really getting 
>> deep into woo-woo territory.
>> 
>> On the contrary, we explain how the quantum physical illusion arise from all 
>> computations which are already realised in the block-mindspace given by very 
>> elementary arithmetic, that we never leave.
>> 
>> Here are all my assumptions: classical logic + the axioms of arithmetic (“s” 
>> is intended to denote the successor function x+1):
>> 
>>   Please describe ambiguous (for me) symbols,  AG
>> 
>> OK.
>> 
>> 
>> 
>> 
>> 0 ≠ s(x)OK
>> s(x) = s(y) -> x = yOK
>> x = 0 v Ey(x = s(y))?
>> 
>> A natural number is either null, or has a predecessor. Read “Ex” by it 
>> exists a number x such that ...
>> 
>> 
>> 
>> 
>> x+0 = xOK
>> x+s(y) = s(x+y) OK
>> x*0=0?   Does * mean multiplication? AG
>> 
>> Yes. “x” looks to much like the variable x. 
>> 
>> 
>> 
>> x*s(y)=(x*y)+x?
>> 
>> x multiplied by the successor of y gives the same as x * y + x. Exemple 6 * 
>> 4 = (6 * 3) + 6.
>>> 
>>> I use mechanism only to help people that this has to be a theory of 
>>> everything. It explains very well consciousness (I think), and matter (as 
>>> confirmed up to now).
>>> 
>>> What is the first step from these postulates, to anything? I mean anything. 
>>> What is mechanism?
>> 
>> Mechanism is the hypothesis that our body is a machine, or a natural 
>> machine-like entity. (It has been discussed in the antic China, India and 
>> greek philosopher/theologians. But you need to wait Descartes and Diderot to 
>> see it coming back, but, notably with Diderot, also its use by materialists 
>> to hide the mind-body problem.
>> 
>> Digital Mechanism as I use it in this list, is slightly more precise. The 
>> notion of digital machine is the notion of Emil Post, Alonzo Church, Alan 
>> Turing, and best explained by Stephen Kleene in his papers and book, notably 
>> his “Introduction to Metamathematics” (1952). 
>> Just ask me, and I gave more on this … after the June exams, as my 
>> scheduling get tighter and tighter those days.
>>> Why do we need these postulate to fix anything?
>> 
>> My goal was to reformulate the mind-body problem in the frame of the 
>> Mechanist hypothesis in the cognitive science/philosophy-o-mind/theology.
>> Unfortunately I have been asked to solve it, which I did, but that requires 
>> some familiarity with Mathematical Logic, which is not well taught.
>> Also, the solution is disliked by the “religious” materialists, and I have 
>> underestimate the number of those in some academical circles, and their 
>> influence (I got a price for my PhD which has disappears without 
>> explanation, just to give one example …).
>> 
>> What happened with your Ph’D?
> 
> It was rejected by my old bullying-friends in Brussels University,, at the 
> recievability level (I never mette them) but I defended it without any 
> problem in France (Lille), where I got the price of the best theses, with 4 
> other laureates in the French speaking world, but then the prized 
> disappeared, and the bullying (always by defamation done in my back) 
> continued and get somehow international, as it is easy to mock or disbelieve 
> someone who say we were wrong since a very long time. But all scientists 
> doing their job have no problem with it, if only because they understand the 
> question raised, and that there is not once claim of truth.
> 
> 
> 
>> Are you associated with a university? Which one? Just curious. AG 
> 
> 
> I have a position at Brussels University where I did create IRIDIA, with late 
> Philippe Smets and some others. After the events IRIDIA has been attached to 
> the Faculty of Applied Science. Engineers are more rigorous in metaphysics 
> than scientist whose often confuse hypotheses and dogma. Not all scientists 
> of course. I have worked with Englert, Brout, Nardone, Gross and others at 
> the time Brout and Englert discovered the “Higgs Boson”. I have a very minor 
> role there, except reassuring François Englert that quantum mechanics makes 
> sense even in cosmology. He added a footnote in a paper suggesting the 
> perplex reader to read Everett for a QM making sense without external 
> observer.
> 
> 
> 
> 
>> 
>>> What is the problem you're trying to fix?
>> 
>> The mind-body problem. 

Re: Entanglement

2018-06-11 Thread Bruno Marchal

> On 11 Jun 2018, at 07:06, agrayson2...@gmail.com wrote:
> 
> 
> 
> On Monday, June 11, 2018 at 2:20:47 AM UTC, agrays...@gmail.com wrote:
> 
> 
> On Monday, June 11, 2018 at 2:09:25 AM UTC, Bruce wrote:
> From: >
>> On Monday, June 11, 2018 at 1:37:53 AM UTC, Bruce wrote:
>> From: Bruno Marchal 
>> Everett prove the contrary, and he convinced me when I read it. I found “his 
>> proof” used in many books on quantum computing, although with different 
>> motivation. Thee result of an experiment, obviously depend of what you 
>> measure, but when you embed the observer in the wave, you get that what they 
>> find is independent of the choice of the base used to describe the 
>> “observer” and the “observed”. If not, the MW would already be refuted.
>> 
>> In that case, MW is refuted. Clearly, what the observer finds is dependent 
>> on the basis in which he is described. Or else experiments would not have 
>> definite results when described in the laboratory from the 1p perspective. 
>> Even if you take the 'bird' view of the whole multiverse -- which is, I 
>> agree, independent of the basis in which it is described -- the view of any 
>> observer embedded in the multiverse is totally basis-dependent. That is, 
>> after all, what we mean by 'worlds' -- the view from within, or the 1p view. 
>> But that view depends on how you describe it: the way in which you partition 
>> the multiverse itself. Only certain very special bases are robust against 
>> environmental decoherence -- how else do you resolve the Schrödinger cat 
>> issue?
>> 
>> Bruce
>> 
>> So you find the resolution in the fact that according to decoherence theory, 
>> the cat is simultaneously alive and dead for only short time?  AG
> 
> Decoherence has resolved the basis question long before the cyanide has hit 
> the cat.
> 
> Bruce
> 
> I don't think you've answered the question. Isn't the cat in a superposition 
> of alive and dead before the cyanide hits? Did Schroedinger write an 
> incorrect wf? If so, what is the correct one IYO? AG
> 
> I surmise your position is that decoherence happens so quickly, that the 
> superposition Schroedinger wrote was really a mixed state. If so, I don't see 
> this as a solution to the paradox, unless you want to allow the existence of 
> a simultaneously alive and dead cat for a very, very short time. AG 


That is why I prefer Bohm’s version of the cat, where the cat alive/dead state 
is corrupted with the up/down state of some particles. It ease the mind by 
showing that the time is not an issue. If you can completely isolate the cat 
from the environment (which is technically impossible), you can maintain the 
cat in the dead + alive superposition state as long as you want. If you isolate 
successfully the entire laboratory including you, Then, someone else can 
resurrect the cat, relatively to himself, despite you saw it dead. 

The reason why we cannot do this in principle, is that we cannot isolate the 
cat, and if the cat, when the cat is dead+alive, interact with some particles 
in the environment, you can no mare factorize the cat state, without tracking 
that particles.

I don’t think it make sense to confine the superposition in the microscopic 
domain, nor in the short-time domain. If the SWE is correct, the superposition 
never disappear, unless a collapse assumption is made, but then it cannot be 
described by QM. Only by QM + exception rules for the observer or the measuring 
apparatus, but there are no evidences for that.

Bruno




> 
> -- 
> You received this message because you are subscribed to the Google Groups 
> "Everything List" group.
> To unsubscribe from this group and stop receiving emails from it, send an 
> email to everything-list+unsubscr...@googlegroups.com 
> .
> To post to this group, send email to everything-list@googlegroups.com 
> .
> Visit this group at https://groups.google.com/group/everything-list 
> .
> For more options, visit https://groups.google.com/d/optout 
> .

-- 
You received this message because you are subscribed to the Google Groups 
"Everything List" group.
To unsubscribe from this group and stop receiving emails from it, send an email 
to everything-list+unsubscr...@googlegroups.com.
To post to this group, send email to everything-list@googlegroups.com.
Visit this group at https://groups.google.com/group/everything-list.
For more options, visit https://groups.google.com/d/optout.


Re: Entanglement

2018-06-11 Thread Bruno Marchal

> On 11 Jun 2018, at 03:37, Bruce Kellett  wrote:
> 
> From: Bruno Marchal mailto:marc...@ulb.ac.be>
>>> On 8 Jun 2018, at 14:55, Bruce Kellett < 
>>> bhkell...@optusnet.com.au 
>>> > wrote:
>>> 
>>> From: Bruno Marchal mailto:marc...@ulb.ac.be>>
> On 8 Jun 2018, at 02:32, Bruce Kellett < 
> bhkell...@optusnet.com.au 
> > wrote:
> 
> 
> The SWE does not give a preferred basis. Basing MWI on the Schrödinger 
> equation runs into the basis problem. Few MWI advocates actually take 
> this seriously. And they should.
 
 The relative proportion of histories do not depend on the choice of the 
 base, so the base we use are chosen endemically, like the present moment 
 for example, in the whole of physics. Obviously, we needs brain to assess 
 our results and communicating, and some works, like sure and others, 
 justify the indexical importance of the position base, with respect to the 
 branch where intelligence can develop.
>>> 
>>> What on earth are you talking about? The position basis is not well-defined 
>>> either. The Hilbert space corresponding to the position operator X has an 
>>> infinite number of possible bases -- just like any other Hilbert space. Any 
>>> linear vector space has an infinite number of possible bases. How do you 
>>> choose which one you are going to use? Talking about the relative 
>>> proportion of histories sounds just like the long-since refuted branch 
>>> counting approach to probabilities.
>> 
>> Measure is quite different from counting.
>> 
>> 
>>> And the probabilities for various outcomes most certainly depend on the 
>>> chosen base, as do the outcomes themselves.
>> 
>> Well, we can use what we call in French “le peigne de Dirac”. To make that 
>> precise Laurent Schwartz has invented the theory of distribution. I simplify 
>> things here. Consider that space has been quantised, like in Loop-Gravity or 
>> something. Here, you do a 1004 fallacy, with respect to the goal (helping 
>> Grayson to have an idea of what is QM-without-collapse).
> 
> ?
> 
> 
>> In this situation, what is the role of the SWE since the wf is usually 
>> asserted without any reference to it? Now consider a general case where 
>> the wf for a system is determined using the SWE. Since the solution can 
>> be expanded using difference bases, say E or p, does each possible 
>> expansion, each implying a different possible set of measurements, imply 
>> a different set of worlds using the SWE? TIA, AG
> 
> The Schrödinger equation merely gives the time evolution of the system. 
> To define the problem you have to specify a wave function. It is in the 
> expansion of this wave function in terms of a set of possible eigenvalues 
> that the preferred basis problem arises. So it is not really down to the 
> SE itself, it is a matter for the wave function. Each expansion basis 
> defines a set of worlds, and all bases give different worlds.
 
 That is correct, but the choice of the basis don’t change the relative 
 “proportion of histories”.
>>> 
>>> The choice of basis makes all the difference in the world.
>> 
>> Everett prove the contrary, and he convinced me when I read it. I found “his 
>> proof” used in many books on quantum computing, although with different 
>> motivation. Thee result of an experiment, obviously depend of what you 
>> measure, but when you embed the observer in the wave, you get that what they 
>> find is independent of the choice of the base used to describe the 
>> “observer” and the “observed”. If not, the MW would already be refuted.
> 
> In that case, MW is refuted. Clearly, what the observer finds is dependent on 
> the basis in which he is described.

?

I disagree. The finding can depend on what the observer decide to measure, 
which is akin to choosing a base, but the couple “observer + that chosen base” 
can be studied in any base, and the same result will described in the memories 
of the observer. I will search Everett proofs of this, as he is the one who 
convinced me on this.



> Or else experiments would not have definite results when described in the 
> laboratory from the 1p perspective.

I don’t see why.



> Even if you take the 'bird' view of the whole multiverse -- which is, I 
> agree, independent of the basis

OK. At least we agree on that.


> in which it is described -- the view of any observer embedded in the 
> multiverse is totally basis-dependent.

Only in the sense that the biological brain has evolved through decoherence 
with respect to some base, but as you say, that process lead to the same result 
from the 1p perspective of those who have chosen the base, or have the base 
imposed through decoherence and evolution, say.
If not then QM would be inconsistent, and had to different laws of 

Re: Entanglement

2018-06-11 Thread Bruno Marchal

> On 10 Jun 2018, at 18:53, agrayson2...@gmail.com wrote:
> 
> 
> 
> On Sunday, June 10, 2018 at 11:22:41 AM UTC, Bruno Marchal wrote:
> 
>> On 9 Jun 2018, at 03:52, Bruce Kellett > > wrote:
>> 
>> From: 
>>> 
>>> On Saturday, June 9, 2018 at 12:22:40 AM UTC, Bruce wrote:
>>> 
>>> Are you trolling? Who claimed that having macrosopic entities in a 
>>> superposition was a fallacy?
>>> 
>>> Edwin Schrodinger. AG
> 
> Erwin Schroedinger.  (To AG).
> 
> OK, but how does one type the umlaut? AG 
>> 
>> Schrödinger thought it was an absurdity, not a fallacy because he saw it as 
>> a consequence of his wave equation.(BK)
> 
>  | Yes. (BM)
>  
> Same question I posed to Bruce: And the difference between an absurdity and a 
> fallacy is WHAT?  AG

An absurdity is just something we are not used to. It shocks prejudices, or 
common sense, which are usually cultural or based on some habit of thought.

A fallacy is something wrong. It is an invalid reasoning, leading to informal 
contradiction.





>> But decoherence theory remove the absurdity. (BK)
> 
> I disagree, but will discuss it a new thread. AG 

OK.



> 
> [snip]
> 
> 
> -- 
> You received this message because you are subscribed to the Google Groups 
> "Everything List" group.
> To unsubscribe from this group and stop receiving emails from it, send an 
> email to everything-list+unsubscr...@googlegroups.com 
> .
> To post to this group, send email to everything-list@googlegroups.com 
> .
> Visit this group at https://groups.google.com/group/everything-list 
> .
> For more options, visit https://groups.google.com/d/optout 
> .

-- 
You received this message because you are subscribed to the Google Groups 
"Everything List" group.
To unsubscribe from this group and stop receiving emails from it, send an email 
to everything-list+unsubscr...@googlegroups.com.
To post to this group, send email to everything-list@googlegroups.com.
Visit this group at https://groups.google.com/group/everything-list.
For more options, visit https://groups.google.com/d/optout.


Re: Entanglement

2018-06-11 Thread Bruno Marchal


> On 11 Jun 2018, at 01:53, Russell Standish  wrote:
> 
> On Sun, Jun 10, 2018 at 09:53:37AM -0700, agrayson2...@gmail.com wrote:
>> 
>> 
>> On Sunday, June 10, 2018 at 11:22:41 AM UTC, Bruno Marchal wrote:
>>> 
>>> 
>>> *Edwin Schrodinger. AG*
>>> 
>>> 
>>> Erwin Schroedinger.  (To AG).
>>> 
>> 
>> OK, but how does one type the umlaut? AG 
> 
> He was more correcting Edwin to Erwin. But to answer your question,
> there are many way. For instance, in emacs (which I use), select
> Options>MultiLingualEnvironment>Toggle Input Method. Select tex as the
> input method.
> 
> Then typing \"o will give ö.
> 
> But Germans accept ae ⇔ ä, oe ⇔ ö, ue ⇔ ü and ss ⇔ ß, when written on
> keyboards without those symbols.

Yes. Now, it it seems accepted completely for Schroedinger, which is rarely 
written Schrödinger. But Gödel and Löb are almost invariably written with the 
umlaut, and never with the “e”. So I guess it is also a question of habit.

Bruno



> 
> 
> -- 
> 
> 
> Dr Russell StandishPhone 0425 253119 (mobile)
> Principal, High Performance Coders
> Visiting Senior Research Fellowhpco...@hpcoders.com.au
> Economics, Kingston University http://www.hpcoders.com.au
> 
> 
> -- 
> You received this message because you are subscribed to the Google Groups 
> "Everything List" group.
> To unsubscribe from this group and stop receiving emails from it, send an 
> email to everything-list+unsubscr...@googlegroups.com.
> To post to this group, send email to everything-list@googlegroups.com.
> Visit this group at https://groups.google.com/group/everything-list.
> For more options, visit https://groups.google.com/d/optout.

-- 
You received this message because you are subscribed to the Google Groups 
"Everything List" group.
To unsubscribe from this group and stop receiving emails from it, send an email 
to everything-list+unsubscr...@googlegroups.com.
To post to this group, send email to everything-list@googlegroups.com.
Visit this group at https://groups.google.com/group/everything-list.
For more options, visit https://groups.google.com/d/optout.


Re: Entanglement

2018-06-11 Thread Bruno Marchal


> On 11 Jun 2018, at 01:56, Russell Standish  wrote:
> 
> On Sun, Jun 10, 2018 at 05:19:34PM +0200, Bruno Marchal wrote:
>>> 
>>> What happened with your Ph’D?
>> 
>> It was rejected by my old bullying-friends in Brussels University,, at the 
>> recievability level (I never mette them) but I defended it without any 
>> problem in France (Lille), where I got the price of the best theses, with 4 
>> other laureates in the French speaking world, but then the prized 
>> disappeared, and the bullying (always by defamation done in my back) 
>> continued and get somehow international, as it is easy to mock or disbelieve 
>> someone who say we were wrong since a very long time. But all scientists 
>> doing their job have no problem with it, if only because they understand the 
>> question raised, and that there is not once claim of truth.
> 
> Plug needed for the book "Amoeba's Secret", which details the story,
> and is the (English translation of the) book for which the prize was give.

The prize was actually for the thesis, which should have been published, but 
they ask me to write the story of the thesis, to denounce the bullying in 
academies, which I did. The jury of the prize, led by Edgar Morin, knew that 
the thesis was written under very peculiar circumstances. But the prize 
disappeared, and I guess “academies” still protect themselves more 
efficaciously than the Church or Hollywood. 



> 
> And note above that Bruno mixes up prize and price in English - they
> are the same word in French (well technically homonyms) - prix.

I know that they differ, but still don’t know which one is which. So I tend to 
use price and prize randomly if I have no help from the context. I confess, and 
apologise. 

Bruno



> 
> 
> -- 
> 
> 
> Dr Russell StandishPhone 0425 253119 (mobile)
> Principal, High Performance Coders
> Visiting Senior Research Fellowhpco...@hpcoders.com.au
> Economics, Kingston University http://www.hpcoders.com.au
> 
> 
> -- 
> You received this message because you are subscribed to the Google Groups 
> "Everything List" group.
> To unsubscribe from this group and stop receiving emails from it, send an 
> email to everything-list+unsubscr...@googlegroups.com.
> To post to this group, send email to everything-list@googlegroups.com.
> Visit this group at https://groups.google.com/group/everything-list.
> For more options, visit https://groups.google.com/d/optout.

-- 
You received this message because you are subscribed to the Google Groups 
"Everything List" group.
To unsubscribe from this group and stop receiving emails from it, send an email 
to everything-list+unsubscr...@googlegroups.com.
To post to this group, send email to everything-list@googlegroups.com.
Visit this group at https://groups.google.com/group/everything-list.
For more options, visit https://groups.google.com/d/optout.


Re: Entanglement

2018-06-11 Thread agrayson2000
[big snip]

For Bruno:

On Monday, June 11, 2018 at 6:50:51 AM UTC, agrays...@gmail.com wrote:
>
>
> *Thanks for the data dump. It's way above my head, so not so far above 
> that I can't see the virtue of using arithmetic logic as a starting point 
> for a new take on reality. I might buy the Kindle version of your book, 
> translated by Russell.  You might be wrong, but I give you credit for 
> tackling the arguably most intractable problem; the mind-body problem. Keep 
> in truckin'! AG*
>

*That should be, Keep ON truckin'! AG *

-- 
You received this message because you are subscribed to the Google Groups 
"Everything List" group.
To unsubscribe from this group and stop receiving emails from it, send an email 
to everything-list+unsubscr...@googlegroups.com.
To post to this group, send email to everything-list@googlegroups.com.
Visit this group at https://groups.google.com/group/everything-list.
For more options, visit https://groups.google.com/d/optout.


Re: Entanglement

2018-06-11 Thread agrayson2000


On Sunday, June 10, 2018 at 3:19:37 PM UTC, Bruno Marchal wrote:
>
>
> On 9 Jun 2018, at 01:10, agrays...@gmail.com  wrote:
>
>
>
> On Friday, June 8, 2018 at 12:06:33 PM UTC, Bruno Marchal wrote:
>>
>>
>> On 8 Jun 2018, at 03:30, agrays...@gmail.com wrote:
>>
>> On Thursday, June 7, 2018 at 9:07:37 AM UTC, Bruno Marchal wrote:
>>
>> [snip]
>>
>> *   So consciousness anticipates all quantum experiment that MIGHT 
>> occur in the future, *
>>
>> The arithmetical relations do that. Consciousness only select the 
>> histories
>>
>> *and creates those worlds in anticipation? Now we're really 
>> getting deep into woo-woo territory.*
>>
>> On the contrary, we explain how the quantum physical illusion arise from 
>> all computations which are already realised in the block-mindspace given by 
>> very elementary arithmetic, that we never leave.
>>
>> Here are all my assumptions: classical logic + the axioms of arithmetic 
>> (“s” is intended to denote the successor function x+1):
>>
>> *  Please describe ambiguous (for me) symbols,  AG*
>>
>>
>> OK.
>>
>>
>>
>>
>> 0 ≠ s(x)OK
>> s(x) = s(y) -> x = yOK
>> x = 0 v Ey(x = s(y))?
>>
>>
>> A natural number is either null, or has a predecessor. Read “Ex” by it 
>> exists a number x such that ...
>>
>>
>>
>>
>> x+0 = xOK
>> x+s(y) = s(x+y) OK
>> x*0=0?   *Does * mean multiplication? AG*
>>
>>
>> Yes. “x” looks to much like the variable x. 
>>
>>
>>
>> x*s(y)=(x*y)+x?
>>
>>
>> x multiplied by the successor of y gives the same as x * y + x. Exemple 6 
>> * 4 = (6 * 3) + 6.
>>
>>
>> I use mechanism only to help people that this has to be a theory of 
>> everything. It explains very well consciousness (I think), and matter (as 
>> confirmed up to now).
>>
>> *What is the first step from these postulates, to anything? I mean 
>> anything. What is mechanism? *
>>
>>
>> Mechanism is the hypothesis that our body is a machine, or a natural 
>> machine-like entity. (It has been discussed in the antic China, India and 
>> greek philosopher/theologians. But you need to wait Descartes and Diderot 
>> to see it coming back, but, notably with Diderot, also its use by 
>> materialists to hide the mind-body problem.
>>
>> Digital Mechanism as I use it in this list, is slightly more precise. The 
>> notion of digital machine is the notion of Emil Post, Alonzo Church, Alan 
>> Turing, and best explained by Stephen Kleene in his papers and book, 
>> notably his “Introduction to Metamathematics” (1952). 
>> Just ask me, and I gave more on this … after the June exams, as my 
>> scheduling get tighter and tighter those days.
>>
>> *Why do we need these postulate to fix anything? *
>>
>>
>> My goal was to reformulate the mind-body problem in the frame of the 
>> Mechanist hypothesis in the cognitive science/philosophy-o-mind/theology.
>> Unfortunately I have been asked to solve it, which I did, but that 
>> requires some familiarity with Mathematical Logic, which is not well taught.
>> Also, the solution is disliked by the “religious” materialists, and I 
>> have underestimate the number of those in some academical circles, and 
>> their influence (I got a price for my PhD which has disappears without 
>> explanation, just to give one example …).
>>
>
> *What happened with your Ph’D? *
>
>
> It was rejected by my old bullying-friends in Brussels University,, at the 
> recievability level (I never mette them) but I defended it without any 
> problem in France (Lille), where I got the price of the best theses, with 4 
> other laureates in the French speaking world, but then the prized 
> disappeared, and the bullying (always by defamation done in my back) 
> continued and get somehow international, as it is easy to mock or 
> disbelieve someone who say we were wrong since a very long time. But all 
> scientists doing their job have no problem with it, if only because they 
> understand the question raised, and that there is not once claim of truth.
>
>
>
>
> *Are you associated with a university? Which one? Just curious. AG *
>
>
>
> I have a position at Brussels University where I did create IRIDIA, with 
> late Philippe Smets and some others. After the events IRIDIA has been 
> attached to the Faculty of Applied Science. Engineers are more rigorous in 
> metaphysics than scientist whose often confuse hypotheses and dogma. Not 
> all scientists of course. I have worked with Englert, Brout, Nardone, Gross 
> and others at the time Brout and Englert discovered the “Higgs Boson”. I 
> have a very minor role there, except reassuring François Englert that 
> quantum mechanics makes sense even in cosmology. He added a footnote in a 
> paper suggesting the perplex reader to read Everett for a QM making sense 
> without external observer.
>
>
>
>
>
> *What is the problem you're trying to fix? *
>>
>>
>> The mind-body problem. How a grey brain can create a color perception, 
>> for example. 
>>
>

Re: Entanglement

2018-06-10 Thread agrayson2000


On Monday, June 11, 2018 at 2:20:47 AM UTC, agrays...@gmail.com wrote:
>
>
>
> On Monday, June 11, 2018 at 2:09:25 AM UTC, Bruce wrote:
>>
>> From: 
>>
>> On Monday, June 11, 2018 at 1:37:53 AM UTC, Bruce wrote: 
>>>
>>> From: Bruno Marchal >> Everett prove the contrary, and he convinced me when I read it. I found 
>>> “his proof” used in many books on quantum computing, although with 
>>> different motivation. Thee result of an experiment, obviously depend of 
>>> what you measure, but when you embed the observer in the wave, you get that 
>>> what they find is independent of the choice of the base used to describe 
>>> the “observer” and the “observed”. If not, the MW would already be refuted.
>>>
>>>
>>> In that case, MW is refuted. Clearly, what the observer finds is 
>>> dependent on the basis in which he is described. Or else experiments would 
>>> not have definite results when described in the laboratory from the 1p 
>>> perspective. Even if you take the 'bird' view of the whole multiverse -- 
>>> which is, I agree, independent of the basis in which it is described -- the 
>>> view of any observer embedded in the multiverse is totally basis-dependent. 
>>> That is, after all, what we mean by 'worlds' -- the view from within, or 
>>> the 1p view. But that view depends on how you describe it: the way in which 
>>> you partition the multiverse itself. Only certain very special bases are 
>>> robust against environmental decoherence -- how else do you resolve the 
>>> Schrödinger cat issue?
>>>
>>> Bruce
>>>
>>
>> *So you find the resolution in the fact that according to decoherence 
>> theory, the cat is simultaneously alive and dead for only short time?  AG*
>>
>>
>> Decoherence has resolved the basis question long before the cyanide has 
>> hit the cat.
>>
>> Bruce
>>
>
> *I don't think you've answered the question. Isn't the cat in a 
> superposition of alive and dead before the cyanide hits? Did Schroedinger 
> write an incorrect wf? If so, what is the correct one IYO? AG *
>

*I surmise your position is that decoherence happens so quickly, that the 
superposition Schroedinger wrote was really a mixed state. If so, I don't 
see this as a solution to the paradox, unless you want to allow the 
existence of a simultaneously alive and dead cat for a very, very short 
time. AG* 

-- 
You received this message because you are subscribed to the Google Groups 
"Everything List" group.
To unsubscribe from this group and stop receiving emails from it, send an email 
to everything-list+unsubscr...@googlegroups.com.
To post to this group, send email to everything-list@googlegroups.com.
Visit this group at https://groups.google.com/group/everything-list.
For more options, visit https://groups.google.com/d/optout.


Re: Entanglement

2018-06-10 Thread agrayson2000


On Monday, June 11, 2018 at 2:09:25 AM UTC, Bruce wrote:
>
> From: >
>
> On Monday, June 11, 2018 at 1:37:53 AM UTC, Bruce wrote: 
>>
>> From: Bruno Marchal > Everett prove the contrary, and he convinced me when I read it. I found 
>> “his proof” used in many books on quantum computing, although with 
>> different motivation. Thee result of an experiment, obviously depend of 
>> what you measure, but when you embed the observer in the wave, you get that 
>> what they find is independent of the choice of the base used to describe 
>> the “observer” and the “observed”. If not, the MW would already be refuted.
>>
>>
>> In that case, MW is refuted. Clearly, what the observer finds is 
>> dependent on the basis in which he is described. Or else experiments would 
>> not have definite results when described in the laboratory from the 1p 
>> perspective. Even if you take the 'bird' view of the whole multiverse -- 
>> which is, I agree, independent of the basis in which it is described -- the 
>> view of any observer embedded in the multiverse is totally basis-dependent. 
>> That is, after all, what we mean by 'worlds' -- the view from within, or 
>> the 1p view. But that view depends on how you describe it: the way in which 
>> you partition the multiverse itself. Only certain very special bases are 
>> robust against environmental decoherence -- how else do you resolve the 
>> Schrödinger cat issue?
>>
>> Bruce
>>
>
> *So you find the resolution in the fact that according to decoherence 
> theory, the cat is simultaneously alive and dead for only short time?  AG*
>
>
> Decoherence has resolved the basis question long before the cyanide has 
> hit the cat.
>
> Bruce
>

*I don't think you've answered the question. Isn't the cat in a 
superposition of alive and dead before the cyanide hits? Did Schroedinger 
write an incorrect wf? If so, what is the correct one IYO? AG *

-- 
You received this message because you are subscribed to the Google Groups 
"Everything List" group.
To unsubscribe from this group and stop receiving emails from it, send an email 
to everything-list+unsubscr...@googlegroups.com.
To post to this group, send email to everything-list@googlegroups.com.
Visit this group at https://groups.google.com/group/everything-list.
For more options, visit https://groups.google.com/d/optout.


Re: Entanglement

2018-06-10 Thread Bruce Kellett

From: mailto:agrayson2...@gmail.com>>

On Monday, June 11, 2018 at 1:37:53 AM UTC, Bruce wrote:

From: *Bruno Marchal* *So you find the resolution in the fact that according to decoherence 
theory, the cat is simultaneously alive and dead for only short time?  AG*


Decoherence has resolved the basis question long before the cyanide has 
hit the cat.


Bruce

--
You received this message because you are subscribed to the Google Groups 
"Everything List" group.
To unsubscribe from this group and stop receiving emails from it, send an email 
to everything-list+unsubscr...@googlegroups.com.
To post to this group, send email to everything-list@googlegroups.com.
Visit this group at https://groups.google.com/group/everything-list.
For more options, visit https://groups.google.com/d/optout.


Re: Entanglement

2018-06-10 Thread agrayson2000


On Monday, June 11, 2018 at 1:37:53 AM UTC, Bruce wrote:
>
> From: Bruno Marchal 
>
> On 8 Jun 2018, at 14:55, Bruce Kellett < 
> bhke...@optusnet.com.au > wrote:
>
> From: Bruno Marchal >
>
> On 8 Jun 2018, at 02:32, Bruce Kellett < 
> bhke...@optusnet.com.au > wrote:
>
>
> The SWE does not give a preferred basis. Basing MWI on the Schrödinger 
> equation runs into the basis problem. Few MWI advocates actually take this 
> seriously. And they should.
>
>
> The relative proportion of histories do not depend on the choice of the 
> base, so the base we use are chosen endemically, like the present moment 
> for example, in the whole of physics. Obviously, we needs brain to assess 
> our results and communicating, and some works, like sure and others, 
> justify the indexical importance of the position base, with respect to the 
> branch where intelligence can develop.
>
>
> What on earth are you talking about? The position basis is not 
> well-defined either. The Hilbert space corresponding to the position 
> operator X has an infinite number of possible bases -- just like any other 
> Hilbert space. Any linear vector space has an infinite number of possible 
> bases. How do you choose which one you are going to use? Talking about the 
> relative proportion of histories sounds just like the long-since refuted 
> branch counting approach to probabilities. 
>
>
> Measure is quite different from counting.
>
>
> And the probabilities for various outcomes most certainly depend on the 
> chosen base, as do the outcomes themselves.
>
>
> Well, we can use what we call in French “le peigne de Dirac”. To make that 
> precise Laurent Schwartz has invented the theory of distribution. I 
> simplify things here. Consider that space has been quantised, like in 
> Loop-Gravity or something. Here, you do a 1004 fallacy, with respect to the 
> goal (helping Grayson to have an idea of what is QM-without-collapse).
>
>
> ?
>
>
> * In this situation, what is the role of the SWE since the wf is usually 
> asserted without any reference to it? Now consider a general case where the 
> wf for a system is determined using the SWE. Since the solution can be 
> expanded using difference bases, say E or p, does each possible expansion, 
> each implying a different possible set of measurements, imply a different 
> set of worlds using the SWE? TIA, AG*
>
>
> The Schrödinger equation merely gives the time evolution of the system. To 
> define the problem you have to specify a wave function. It is in the 
> expansion of this wave function in terms of a set of possible eigenvalues 
> that the preferred basis problem arises. So it is not really down to the SE 
> itself, it is a matter for the wave function. Each expansion basis defines 
> a set of worlds, and all bases give different worlds.
>
>
> That is correct, but the choice of the basis don’t change the relative 
> “proportion of histories”.
>
>
> The choice of basis makes all the difference in the world. 
>
>
> Everett prove the contrary, and he convinced me when I read it. I found 
> “his proof” used in many books on quantum computing, although with 
> different motivation. Thee result of an experiment, obviously depend of 
> what you measure, but when you embed the observer in the wave, you get that 
> what they find is independent of the choice of the base used to describe 
> the “observer” and the “observed”. If not, the MW would already be refuted.
>
>
> In that case, MW is refuted. Clearly, what the observer finds is dependent 
> on the basis in which he is described. Or else experiments would not have 
> definite results when described in the laboratory from the 1p perspective. 
> Even if you take the 'bird' view of the whole multiverse -- which is, I 
> agree, independent of the basis in which it is described -- the view of any 
> observer embedded in the multiverse is totally basis-dependent. That is, 
> after all, what we mean by 'worlds' -- the view from within, or the 1p 
> view. But that view depends on how you describe it: the way in which you 
> partition the multiverse itself. Only certain very special bases are robust 
> against environmental decoherence -- how else do you resolve the 
> Schrödinger cat issue?
>
> Bruce
>

*So you find the resolution in the fact that according to decoherence 
theory, the cat is simultaneously alive and dead for only short time?  AG*

-- 
You received this message because you are subscribed to the Google Groups 
"Everything List" group.
To unsubscribe from this group and stop receiving emails from it, send an email 
to everything-list+unsubscr...@googlegroups.com.
To post to this group, send email to everything-list@googlegroups.com.
Visit this group at https://groups.google.com/group/everything-list.
For more options, visit https://groups.google.com/d/optout.


Re: Entanglement

2018-06-10 Thread Bruce Kellett

From: *Bruno Marchal* mailto:marc...@ulb.ac.be>

On 8 Jun 2018, at 14:55, Bruce Kellett  wrote:

From: *Bruno Marchal* mailto:marc...@ulb.ac.be>>
On 8 Jun 2018, at 02:32, Bruce Kellett  
wrote:



The SWE does not give a preferred basis. Basing MWI on the 
Schrödinger equation runs into the basis problem. Few MWI advocates 
actually take this seriously. And they should.


The relative proportion of histories do not depend on the choice of 
the base, so the base we use are chosen endemically, like the 
present moment for example, in the whole of physics. Obviously, we 
needs brain to assess our results and communicating, and some works, 
like sure and others, justify the indexical importance of the 
position base, with respect to the branch where intelligence can 
develop.


What on earth are you talking about? The position basis is not 
well-defined either. The Hilbert space corresponding to the position 
operator X has an infinite number of possible bases -- just like any 
other Hilbert space. Any linear vector space has an infinite number 
of possible bases. How do you choose which one you are going to use? 
Talking about the relative proportion of histories sounds just like 
the long-since refuted branch counting approach to probabilities.


Measure is quite different from counting.


And the probabilities for various outcomes most certainly depend on 
the chosen base, as do the outcomes themselves.


Well, we can use what we call in French “le peigne de Dirac”. To make 
that precise Laurent Schwartz has invented the theory of distribution. 
I simplify things here. Consider that space has been quantised, like 
in Loop-Gravity or something. Here, you do a 1004 fallacy, with 
respect to the goal (helping Grayson to have an idea of what is 
QM-without-collapse).


?


*In this situation, what is the role of the SWE since the wf is 
usually asserted without any reference to it? Now consider a 
general case where the wf for a system is determined using the 
SWE. Since the solution can be expanded using difference bases, 
say E or p, does each possible expansion, each implying a 
different possible set of measurements, imply a different set of 
worlds using the SWE? TIA, AG*


The Schrödinger equation merely gives the time evolution of the 
system. To define the problem you have to specify a wave function. 
It is in the expansion of this wave function in terms of a set of 
possible eigenvalues that the preferred basis problem arises. So it 
is not really down to the SE itself, it is a matter for the wave 
function. Each expansion basis defines a set of worlds, and all 
bases give different worlds.


That is correct, but the choice of the basis don’t change the 
relative “proportion of histories”.


The choice of basis makes all the difference in the world.


Everett prove the contrary, and he convinced me when I read it. I 
found “his proof” used in many books on quantum computing, although 
with different motivation. Thee result of an experiment, obviously 
depend of what you measure, but when you embed the observer in the 
wave, you get that what they find is independent of the choice of the 
base used to describe the “observer” and the “observed”. If not, the 
MW would already be refuted.


In that case, MW is refuted. Clearly, what the observer finds is 
dependent on the basis in which he is described. Or else experiments 
would not have definite results when described in the laboratory from 
the 1p perspective. Even if you take the 'bird' view of the whole 
multiverse -- which is, I agree, independent of the basis in which it is 
described -- the view of any observer embedded in the multiverse is 
totally basis-dependent. That is, after all, what we mean by 'worlds' -- 
the view from within, or the 1p view. But that view depends on how you 
describe it: the way in which you partition the multiverse itself. Only 
certain very special bases are robust against environmental decoherence 
-- how else do you resolve the Schrödinger cat issue?


Bruce

--
You received this message because you are subscribed to the Google Groups 
"Everything List" group.
To unsubscribe from this group and stop receiving emails from it, send an email 
to everything-list+unsubscr...@googlegroups.com.
To post to this group, send email to everything-list@googlegroups.com.
Visit this group at https://groups.google.com/group/everything-list.
For more options, visit https://groups.google.com/d/optout.


Re: Entanglement

2018-06-10 Thread agrayson2000


On Monday, June 11, 2018 at 12:01:39 AM UTC, Russell Standish wrote:
>
> On Sun, Jun 10, 2018 at 05:19:34PM +0200, Bruno Marchal wrote: 
> > > 
> > > What happened with your Ph’D? 
> > 
> > It was rejected by my old bullying-friends in Brussels University,, at 
> the recievability level (I never mette them) but I defended it without any 
> problem in France (Lille), where I got the price of the best theses, with 4 
> other laureates in the French speaking world, but then the prized 
> disappeared, and the bullying (always by defamation done in my back) 
> continued and get somehow international, as it is easy to mock or 
> disbelieve someone who say we were wrong since a very long time. But all 
> scientists doing their job have no problem with it, if only because they 
> understand the question raised, and that there is not once claim of truth. 
>
> Plug needed for the book "Amoeba's Secret", which details the story, 
> and is the (English translation of the) book for which the prize was give. 
>
> And note above that Bruno mixes up prize and price in English - they 
> are the same word in French (well technically homonyms) - prix. 
>

Thanks for the clarification. Looks like an interesting story. Looks like 
Bruno picked the hardest problem to tackle. AG 

>
>
> -- 
>
>  
>
> Dr Russell StandishPhone 0425 253119 (mobile) 
> Principal, High Performance Coders 
> Visiting Senior Research Fellowhpc...@hpcoders.com.au 
>  
> Economics, Kingston University http://www.hpcoders.com.au 
>  
>
>

-- 
You received this message because you are subscribed to the Google Groups 
"Everything List" group.
To unsubscribe from this group and stop receiving emails from it, send an email 
to everything-list+unsubscr...@googlegroups.com.
To post to this group, send email to everything-list@googlegroups.com.
Visit this group at https://groups.google.com/group/everything-list.
For more options, visit https://groups.google.com/d/optout.


Re: Entanglement

2018-06-10 Thread Russell Standish
On Sun, Jun 10, 2018 at 09:53:37AM -0700, agrayson2...@gmail.com wrote:
> 
> 
> On Sunday, June 10, 2018 at 11:22:41 AM UTC, Bruno Marchal wrote:
> >
> >
> > *Edwin Schrodinger. AG*
> >
> >
> > Erwin Schroedinger.  (To AG).
> >
> 
> OK, but how does one type the umlaut? AG 

He was more correcting Edwin to Erwin. But to answer your question,
there are many way. For instance, in emacs (which I use), select
Options>MultiLingualEnvironment>Toggle Input Method. Select tex as the
input method.

Then typing \"o will give ö.

But Germans accept ae ⇔ ä, oe ⇔ ö, ue ⇔ ü and ss ⇔ ß, when written on
keyboards without those symbols.


-- 


Dr Russell StandishPhone 0425 253119 (mobile)
Principal, High Performance Coders
Visiting Senior Research Fellowhpco...@hpcoders.com.au
Economics, Kingston University http://www.hpcoders.com.au


-- 
You received this message because you are subscribed to the Google Groups 
"Everything List" group.
To unsubscribe from this group and stop receiving emails from it, send an email 
to everything-list+unsubscr...@googlegroups.com.
To post to this group, send email to everything-list@googlegroups.com.
Visit this group at https://groups.google.com/group/everything-list.
For more options, visit https://groups.google.com/d/optout.


Re: Entanglement

2018-06-10 Thread Russell Standish
On Sun, Jun 10, 2018 at 05:19:34PM +0200, Bruno Marchal wrote:
> > 
> > What happened with your Ph’D?
> 
> It was rejected by my old bullying-friends in Brussels University,, at the 
> recievability level (I never mette them) but I defended it without any 
> problem in France (Lille), where I got the price of the best theses, with 4 
> other laureates in the French speaking world, but then the prized 
> disappeared, and the bullying (always by defamation done in my back) 
> continued and get somehow international, as it is easy to mock or disbelieve 
> someone who say we were wrong since a very long time. But all scientists 
> doing their job have no problem with it, if only because they understand the 
> question raised, and that there is not once claim of truth.

Plug needed for the book "Amoeba's Secret", which details the story,
and is the (English translation of the) book for which the prize was give.

And note above that Bruno mixes up prize and price in English - they
are the same word in French (well technically homonyms) - prix.


-- 


Dr Russell StandishPhone 0425 253119 (mobile)
Principal, High Performance Coders
Visiting Senior Research Fellowhpco...@hpcoders.com.au
Economics, Kingston University http://www.hpcoders.com.au


-- 
You received this message because you are subscribed to the Google Groups 
"Everything List" group.
To unsubscribe from this group and stop receiving emails from it, send an email 
to everything-list+unsubscr...@googlegroups.com.
To post to this group, send email to everything-list@googlegroups.com.
Visit this group at https://groups.google.com/group/everything-list.
For more options, visit https://groups.google.com/d/optout.


Re: Entanglement

2018-06-10 Thread agrayson2000


On Sunday, June 10, 2018 at 11:22:41 AM UTC, Bruno Marchal wrote:
>
>
> On 9 Jun 2018, at 03:52, Bruce Kellett  > wrote:
>
> From:  
>
>
> On Saturday, June 9, 2018 at 12:22:40 AM UTC, Bruce wrote: 
>>
>>
>> Are you trolling? Who claimed that having macrosopic entities in a 
>> superposition was a fallacy? 
>>
>
> *Edwin Schrodinger. AG*
>
>
> Erwin Schroedinger.  (To AG).
>

OK, but how does one type the umlaut? AG 

>
> Schrödinger thought it was an absurdity, not a fallacy because he saw it 
> as a consequence of his wave equation.(BK)
>
>  | Yes. (BM)
 
*Same question I posed to Bruce: And the difference between an absurdity 
and a fallacy is WHAT?  AG*

> But decoherence theory remove the absurdity. (BK)
>
> *I disagree, but will discuss it a new thread. AG *

[snip]

-- 
You received this message because you are subscribed to the Google Groups 
"Everything List" group.
To unsubscribe from this group and stop receiving emails from it, send an email 
to everything-list+unsubscr...@googlegroups.com.
To post to this group, send email to everything-list@googlegroups.com.
Visit this group at https://groups.google.com/group/everything-list.
For more options, visit https://groups.google.com/d/optout.


Re: Entanglement

2018-06-10 Thread Bruno Marchal

> On 9 Jun 2018, at 01:10, agrayson2...@gmail.com wrote:
> 
> 
> 
> On Friday, June 8, 2018 at 12:06:33 PM UTC, Bruno Marchal wrote:
> 
> On 8 Jun 2018, at 03:30, agrays...@gmail.com  wrote:
> 
> On Thursday, June 7, 2018 at 9:07:37 AM UTC, Bruno Marchal wrote:
> 
> [snip]
> 
>So consciousness anticipates all quantum experiment that MIGHT occur 
> in the future,
> 
> The arithmetical relations do that. Consciousness only select the histories
> 
> and creates those worlds in anticipation? Now we're really getting 
> deep into woo-woo territory.
> 
> On the contrary, we explain how the quantum physical illusion arise from all 
> computations which are already realised in the block-mindspace given by very 
> elementary arithmetic, that we never leave.
> 
> Here are all my assumptions: classical logic + the axioms of arithmetic (“s” 
> is intended to denote the successor function x+1):
> 
>   Please describe ambiguous (for me) symbols,  AG
> 
> OK.
> 
> 
> 
> 
> 0 ≠ s(x)OK
> s(x) = s(y) -> x = yOK
> x = 0 v Ey(x = s(y))?
> 
> A natural number is either null, or has a predecessor. Read “Ex” by it exists 
> a number x such that ...
> 
> 
> 
> 
> x+0 = xOK
> x+s(y) = s(x+y) OK
> x*0=0?   Does * mean multiplication? AG
> 
> Yes. “x” looks to much like the variable x. 
> 
> 
> 
> x*s(y)=(x*y)+x?
> 
> x multiplied by the successor of y gives the same as x * y + x. Exemple 6 * 4 
> = (6 * 3) + 6.
>> 
>> I use mechanism only to help people that this has to be a theory of 
>> everything. It explains very well consciousness (I think), and matter (as 
>> confirmed up to now).
>> 
>> What is the first step from these postulates, to anything? I mean anything. 
>> What is mechanism?
> 
> Mechanism is the hypothesis that our body is a machine, or a natural 
> machine-like entity. (It has been discussed in the antic China, India and 
> greek philosopher/theologians. But you need to wait Descartes and Diderot to 
> see it coming back, but, notably with Diderot, also its use by materialists 
> to hide the mind-body problem.
> 
> Digital Mechanism as I use it in this list, is slightly more precise. The 
> notion of digital machine is the notion of Emil Post, Alonzo Church, Alan 
> Turing, and best explained by Stephen Kleene in his papers and book, notably 
> his “Introduction to Metamathematics” (1952). 
> Just ask me, and I gave more on this … after the June exams, as my scheduling 
> get tighter and tighter those days.
>> Why do we need these postulate to fix anything?
> 
> My goal was to reformulate the mind-body problem in the frame of the 
> Mechanist hypothesis in the cognitive science/philosophy-o-mind/theology.
> Unfortunately I have been asked to solve it, which I did, but that requires 
> some familiarity with Mathematical Logic, which is not well taught.
> Also, the solution is disliked by the “religious” materialists, and I have 
> underestimate the number of those in some academical circles, and their 
> influence (I got a price for my PhD which has disappears without explanation, 
> just to give one example …).
> 
> What happened with your Ph’D?

It was rejected by my old bullying-friends in Brussels University,, at the 
recievability level (I never mette them) but I defended it without any problem 
in France (Lille), where I got the price of the best theses, with 4 other 
laureates in the French speaking world, but then the prized disappeared, and 
the bullying (always by defamation done in my back) continued and get somehow 
international, as it is easy to mock or disbelieve someone who say we were 
wrong since a very long time. But all scientists doing their job have no 
problem with it, if only because they understand the question raised, and that 
there is not once claim of truth.



> Are you associated with a university? Which one? Just curious. AG 


I have a position at Brussels University where I did create IRIDIA, with late 
Philippe Smets and some others. After the events IRIDIA has been attached to 
the Faculty of Applied Science. Engineers are more rigorous in metaphysics than 
scientist whose often confuse hypotheses and dogma. Not all scientists of 
course. I have worked with Englert, Brout, Nardone, Gross and others at the 
time Brout and Englert discovered the “Higgs Boson”. I have a very minor role 
there, except reassuring François Englert that quantum mechanics makes sense 
even in cosmology. He added a footnote in a paper suggesting the perplex reader 
to read Everett for a QM making sense without external observer.




> 
>> What is the problem you're trying to fix?
> 
> The mind-body problem. How a grey brain can create a color perception, for 
> example.
> 
> Unsolved IMO, Not a trivial problem. AG


Glad to hear this.


>  
> But it is more deep than that, as eventually, Mechanism is shown incompatible 
> with materialism and/or physicalism, which is the actual paradigm in 

Re: Entanglement

2018-06-10 Thread Bruno Marchal

> On 9 Jun 2018, at 03:52, Bruce Kellett  wrote:
> 
> From: mailto:agrayson2...@gmail.com>
>> 
>> On Saturday, June 9, 2018 at 12:22:40 AM UTC, Bruce wrote:
>> 
>> Are you trolling? Who claimed that having macrosopic entities in a 
>> superposition was a fallacy?
>> 
>> Edwin Schrodinger. AG

Erwin Schroedinger.  (To AG).


> 
> Schrödinger thought it was an absurdity, not a fallacy because he saw it as a 
> consequence of his wave equation.

Yes.


> But decoherence theory remove the absurdity.

Partially  with Everett. It remains still  more absurd  with Copenhagen, or at 
least unclear.
The absurdity is entirely removed with mechanism, as long as its prediction are 
correct (of course).

Bruno




> 
> Bruce
> 
> -- 
> You received this message because you are subscribed to the Google Groups 
> "Everything List" group.
> To unsubscribe from this group and stop receiving emails from it, send an 
> email to everything-list+unsubscr...@googlegroups.com 
> .
> To post to this group, send email to everything-list@googlegroups.com 
> .
> Visit this group at https://groups.google.com/group/everything-list 
> .
> For more options, visit https://groups.google.com/d/optout 
> .

-- 
You received this message because you are subscribed to the Google Groups 
"Everything List" group.
To unsubscribe from this group and stop receiving emails from it, send an email 
to everything-list+unsubscr...@googlegroups.com.
To post to this group, send email to everything-list@googlegroups.com.
Visit this group at https://groups.google.com/group/everything-list.
For more options, visit https://groups.google.com/d/optout.


Re: Entanglement

2018-06-08 Thread agrayson2000


On Saturday, June 9, 2018 at 12:22:40 AM UTC, Bruce wrote:
>
> From: >
>
>
> On Friday, June 8, 2018 at 12:55:13 PM UTC, Bruce wrote:
>
> The Schrödinger equation merely gives the time evolution of the system. To 
> define the problem you have to specify a wave function. It is in the 
> expansion of this wave function in terms of a set of possible eigenvalues 
> that the preferred basis problem arises. So it is not really down to the SE 
> itself, it is a matter for the wave function. Each expansion basis defines 
> a set of worlds, and all bases give different worlds.
>
>
>
> * If we measure E, aren't we defacto measuring p, since the two 
> observables are related by a simple mathematical expression? Yet you assert 
> they represent different worlds. Is this because the measuring apparatus 
> differs if the observable are different? AG *
>
>
> Measuring E or p can be related, as for a photon, or unrelated, as for a 
> measurement of the energy levels of a molecule. But in either case, there 
> are an infinite number of possible bases in which to express the energy or 
> momentum of a state. These are the different worlds to which I am referring 
> -- the difference between an energy or a momentum measurement is not 
> relevant in this context.
>





*I don't follow. What is the distinguishing feature of worlds created by 
measuring E and p? Isn't what is measured, distinguishes worlds, which are 
allegedly copied under the assumption that what could have been measured in 
this world, but were not, were measured in some identical copy?On the other 
issue, if you find a cat alive and dead simultaneously absurd (or 
fallacious), why is it any less absurd (or fallacious) if it happens for 
only a short time, until decoherence occurs? And if you find Many Worlds 
absurd (or fallacious) on multiple grounds, why do you affirm it when it's 
implied by decoherence superpositions?AG*


That is correct, but the choice of the basis don’t change the relative 
“proportion of histories”.


The choice of basis makes all the difference in the world. Now that we 
understand decoherence, the only bases that are useful are those that are 
robust against environmental decoherence. That is why we don't see 
superpositions of live and dead cats -- that superposition base is not 
robust.

*You seem to be regressing, or shall we say relapsing into the fallacy. 
ISTM you have previously acknowledged that we don't see superpositions of 
live and dead cats because of the fallacy of including macro entities in a 
superposition -- which is what Edwin was trying to warn us against. Nothing 
to do about robustness against environmental decoherence, which assumes an 
actual superposition exists for some short duration. CMIIAW. AG*


Are you trolling? Who claimed that having macrosopic entities in a 
superposition was a fallacy? Such superpositions are generally very short 
lived because of decoherence, but they can certainly be formed. The basis 
which would described such macrosopic superpositions is not robust against 
decoherence -- which is all that I have ever claimed.

Bruce

-- 
You received this message because you are subscribed to the Google Groups 
"Everything List" group.
To unsubscribe from this group and stop receiving emails from it, send an email 
to everything-list+unsubscr...@googlegroups.com.
To post to this group, send email to everything-list@googlegroups.com.
Visit this group at https://groups.google.com/group/everything-list.
For more options, visit https://groups.google.com/d/optout.


Re: Entanglement

2018-06-08 Thread agrayson2000


On Saturday, June 9, 2018 at 1:52:21 AM UTC, Bruce wrote:
>
> From:  
>
>
> On Saturday, June 9, 2018 at 12:22:40 AM UTC, Bruce wrote: 
>>
>>
>> Are you trolling? Who claimed that having macrosopic entities in a 
>> superposition was a fallacy? 
>>
>
> *Edwin Schrodinger. AG*
>
>
> Schrödinger thought it was an absurdity, not a fallacy because he saw it 
> as a consequence of his wave equation. But decoherence theory remove the 
> absurdity.
>
> Bruce
>

*And the distinction between Absurdity and Fallacy is what? You used macro 
superpositions in decoherence theory and what did you get? Copies of 
worlds! I rest my case. AG*

-- 
You received this message because you are subscribed to the Google Groups 
"Everything List" group.
To unsubscribe from this group and stop receiving emails from it, send an email 
to everything-list+unsubscr...@googlegroups.com.
To post to this group, send email to everything-list@googlegroups.com.
Visit this group at https://groups.google.com/group/everything-list.
For more options, visit https://groups.google.com/d/optout.


Re: Entanglement

2018-06-08 Thread agrayson2000


On Saturday, June 9, 2018 at 1:52:21 AM UTC, Bruce wrote:
>
> From:  
>
>
> On Saturday, June 9, 2018 at 12:22:40 AM UTC, Bruce wrote: 
>>
>>
>> Are you trolling? Who claimed that having macrosopic entities in a 
>> superposition was a fallacy? 
>>
>
> *Edwin Schrodinger. AG*
>
>
> Schrödinger thought it was an absurdity, not a fallacy because he saw it 
> as a consequence of his wave equation. But decoherence theory remove the 
> absurdity.
>
> Bruce
>

-- 
You received this message because you are subscribed to the Google Groups 
"Everything List" group.
To unsubscribe from this group and stop receiving emails from it, send an email 
to everything-list+unsubscr...@googlegroups.com.
To post to this group, send email to everything-list@googlegroups.com.
Visit this group at https://groups.google.com/group/everything-list.
For more options, visit https://groups.google.com/d/optout.


Re: Entanglement

2018-06-08 Thread Bruce Kellett

From: mailto:agrayson2...@gmail.com>


On Saturday, June 9, 2018 at 12:22:40 AM UTC, Bruce wrote:


Are you trolling? Who claimed that having macrosopic entities in a
superposition was a fallacy?


*Edwin Schrodinger. AG*


Schrödinger thought it was an absurdity, not a fallacy because he saw it 
as a consequence of his wave equation. But decoherence theory remove the 
absurdity.


Bruce

--
You received this message because you are subscribed to the Google Groups 
"Everything List" group.
To unsubscribe from this group and stop receiving emails from it, send an email 
to everything-list+unsubscr...@googlegroups.com.
To post to this group, send email to everything-list@googlegroups.com.
Visit this group at https://groups.google.com/group/everything-list.
For more options, visit https://groups.google.com/d/optout.


Re: Entanglement

2018-06-08 Thread agrayson2000


On Saturday, June 9, 2018 at 12:22:40 AM UTC, Bruce wrote:
>
> From: >
>
>
> On Friday, June 8, 2018 at 12:55:13 PM UTC, Bruce wrote:
>
> The Schrödinger equation merely gives the time evolution of the system. To 
> define the problem you have to specify a wave function. It is in the 
> expansion of this wave function in terms of a set of possible eigenvalues 
> that the preferred basis problem arises. So it is not really down to the SE 
> itself, it is a matter for the wave function. Each expansion basis defines 
> a set of worlds, and all bases give different worlds.
>
>
>
> * If we measure E, aren't we defacto measuring p, since the two 
> observables are related by a simple mathematical expression? Yet you assert 
> they represent different worlds. Is this because the measuring apparatus 
> differs if the observable are different? AG *
>
>
> Measuring E or p can be related, as for a photon, or unrelated, as for a 
> measurement of the energy levels of a molecule. But in either case, there 
> are an infinite number of possible bases in which to express the energy or 
> momentum of a state. These are the different worlds to which I am referring 
> -- the difference between an energy or a momentum measurement is not 
> relevant in this context.
>
>
> That is correct, but the choice of the basis don’t change the relative 
> “proportion of histories”.
>
>
> The choice of basis makes all the difference in the world. Now that we 
> understand decoherence, the only bases that are useful are those that are 
> robust against environmental decoherence. That is why we don't see 
> superpositions of live and dead cats -- that superposition base is not 
> robust.
>
> *You seem to be regressing, or shall we say relapsing into the fallacy. 
> ISTM you have previously acknowledged that we don't see superpositions of 
> live and dead cats because of the fallacy of including macro entities in a 
> superposition -- which is what Edwin was trying to warn us against. Nothing 
> to do about robustness against environmental decoherence, which assumes an 
> actual superposition exists for some short duration. CMIIAW. AG*
>
>
> Are you trolling? Who claimed that having macrosopic entities in a 
> superposition was a fallacy? 
>

*Edwin Schrodinger. AG*
 

> Such superpositions are generally very short lived because of decoherence, 
> but they can certainly be formed. The basis which would described such 
> macrosopic superpositions is not robust against decoherence -- which is all 
> that I have ever claimed.
>
> Bruce
>
>

-- 
You received this message because you are subscribed to the Google Groups 
"Everything List" group.
To unsubscribe from this group and stop receiving emails from it, send an email 
to everything-list+unsubscr...@googlegroups.com.
To post to this group, send email to everything-list@googlegroups.com.
Visit this group at https://groups.google.com/group/everything-list.
For more options, visit https://groups.google.com/d/optout.


Re: Entanglement

2018-06-08 Thread Bruce Kellett

From: mailto:agrayson2...@gmail.com>>


On Friday, June 8, 2018 at 12:55:13 PM UTC, Bruce wrote:
The Schrödinger equation merely gives the time evolution of the 
system. To define the problem you have to specify a wave function. 
It is in the expansion of this wave function in terms of a set of 
possible eigenvalues that the preferred basis problem arises. So it 
is not really down to the SE itself, it is a matter for the wave 
function. Each expansion basis defines a set of worlds, and all 
bases give different worlds.

*
If we measure E, aren't we defacto measuring p, since the two 
observables are related by a simple mathematical expression? Yet you 
assert they represent different worlds. Is this because the measuring 
apparatus differs if the observable are different? AG

*


Measuring E or p can be related, as for a photon, or unrelated, as for a 
measurement of the energy levels of a molecule. But in either case, 
there are an infinite number of possible bases in which to express the 
energy or momentum of a state. These are the different worlds to which I 
am referring -- the difference between an energy or a momentum 
measurement is not relevant in this context.




That is correct, but the choice of the basis don’t change the 
relative “proportion of histories”.


The choice of basis makes all the difference in the world. Now 
that we understand decoherence, the only bases that are useful are 
those that are robust against environmental decoherence. That is why 
we don't see superpositions of live and dead cats -- that 
superposition base is not robust.


*You seem to be regressing, or shall we say relapsing into the 
fallacy. ISTM you have previously acknowledged that we don't see 
superpositions of live and dead cats because of the fallacy of 
including macro entities in a superposition -- which is what Edwin was 
trying to warn us against. Nothing to do about robustness against 
environmental decoherence, which assumes an actual superposition 
exists for some short duration. CMIIAW. AG*


Are you trolling? Who claimed that having macrosopic entities in a 
superposition was a fallacy? Such superpositions are generally very 
short lived because of decoherence, but they can certainly be formed. 
The basis which would described such macrosopic superpositions is not 
robust against decoherence -- which is all that I have ever claimed.


Bruce

--
You received this message because you are subscribed to the Google Groups 
"Everything List" group.
To unsubscribe from this group and stop receiving emails from it, send an email 
to everything-list+unsubscr...@googlegroups.com.
To post to this group, send email to everything-list@googlegroups.com.
Visit this group at https://groups.google.com/group/everything-list.
For more options, visit https://groups.google.com/d/optout.


Re: Entanglement

2018-06-08 Thread agrayson2000


On Friday, June 8, 2018 at 12:55:13 PM UTC, Bruce wrote:
>
> From: *Bruno Marchal* >
>
> On 8 Jun 2018, at 02:32, Bruce Kellett < 
> bhke...@optusnet.com.au > wrote:
>
>
> The SWE does not give a preferred basis. Basing MWI on the Schrödinger 
> equation runs into the basis problem. Few MWI advocates actually take this 
> seriously. And they should.
>
>
> The relative proportion of histories do not depend on the choice of the 
> base, so the base we use are chosen endemically, like the present moment 
> for example, in the whole of physics. Obviously, we needs brain to assess 
> our results and communicating, and some works, like sure and others, 
> justify the indexical importance of the position base, with respect to the 
> branch where intelligence can develop.
>
>
> What on earth are you talking about? The position basis is not 
> well-defined either. The Hilbert space corresponding to the position 
> operator X has an infinite number of possible bases -- just like any other 
> Hilbert space. Any linear vector space has an infinite number of possible 
> bases. How do you choose which one you are going to use? Talking about the 
> relative proportion of histories sounds just like the long-since refuted 
> branch counting approach to probabilities. And the probabilities for 
> various outcomes most certainly depend on the chosen base, as do the 
> outcomes themselves.
>

 

* In this situation, what is the role of the SWE since the wf is usually 
asserted without any reference to it? Now consider a general case where the 
wf for a system is determined using the SWE. Since the solution can be 
expanded using difference bases, say E or p, does each possible expansion, 
each implying a different possible set of measurements, imply a different 
set of worlds using the SWE? TIA, AG*


The Schrödinger equation merely gives the time evolution of the system. To 
define the problem you have to specify a wave function. It is in the 
expansion of this wave function in terms of a set of possible eigenvalues 
that the preferred basis problem arises. So it is not really down to the SE 
itself, it is a matter for the wave function. Each expansion basis defines 
a set of worlds, and all bases give different worlds.



*If we measure E, aren't we defacto measuring p, since the two observables 
are related by a simple mathematical expression? Yet you assert they 
represent different worlds. Is this because the measuring apparatus differs 
if the observable are different? AG *


That is correct, but the choice of the basis don’t change the relative 
“proportion of histories”.


The choice of basis makes all the difference in the world. Now that we 
understand decoherence, the only bases that are useful are those that are 
robust against environmental decoherence. That is why we don't see 
superpositions of live and dead cats -- that superposition base is not 
robust.


*You seem to be regressing, or shall we say relapsing into the fallacy. 
ISTM you have previously acknowledged that we don't see superpositions of 
live and dead cats because of the fallacy of including macro entities in a 
superposition -- which is what Edwin was trying to warn us against. Nothing 
to do about robustness against environmental decoherence, which assumes an 
actual superposition exists for some short duration. CMIIAW. AG *

It threats only the naïve conception of “worlds”, which has led to the 
works of Griffith and Omnes (and Gel Mann & Hartle). That works remains 
still a bit naïve with respect of the type of histories we can encounter in 
arithmetic.


The consistent histories approach is just another way of considering many 
worlds. The histories are no more unique than are the worlds.

Bruce

-- 
You received this message because you are subscribed to the Google Groups 
"Everything List" group.
To unsubscribe from this group and stop receiving emails from it, send an email 
to everything-list+unsubscr...@googlegroups.com.
To post to this group, send email to everything-list@googlegroups.com.
Visit this group at https://groups.google.com/group/everything-list.
For more options, visit https://groups.google.com/d/optout.


Re: Entanglement

2018-06-08 Thread agrayson2000


On Friday, June 8, 2018 at 12:06:33 PM UTC, Bruno Marchal wrote:
>
>
> On 8 Jun 2018, at 03:30, agrays...@gmail.com  wrote:
>
> On Thursday, June 7, 2018 at 9:07:37 AM UTC, Bruno Marchal wrote:
>
> [snip]
>
> *   So consciousness anticipates all quantum experiment that MIGHT 
> occur in the future, *
>
> The arithmetical relations do that. Consciousness only select the histories
>
> *and creates those worlds in anticipation? Now we're really 
> getting deep into woo-woo territory.*
>
> On the contrary, we explain how the quantum physical illusion arise from 
> all computations which are already realised in the block-mindspace given by 
> very elementary arithmetic, that we never leave.
>
> Here are all my assumptions: classical logic + the axioms of arithmetic 
> (“s” is intended to denote the successor function x+1):
>
> *  Please describe ambiguous (for me) symbols,  AG*
>
>
> OK.
>
>
>
>
> 0 ≠ s(x)OK
> s(x) = s(y) -> x = yOK
> x = 0 v Ey(x = s(y))?
>
>
> A natural number is either null, or has a predecessor. Read “Ex” by it 
> exists a number x such that ...
>
>
>
>
> x+0 = xOK
> x+s(y) = s(x+y) OK
> x*0=0?   *Does * mean multiplication? AG*
>
>
> Yes. “x” looks to much like the variable x. 
>
>
>
> x*s(y)=(x*y)+x?
>
>
> x multiplied by the successor of y gives the same as x * y + x. Exemple 6 
> * 4 = (6 * 3) + 6.
>
>
> I use mechanism only to help people that this has to be a theory of 
> everything. It explains very well consciousness (I think), and matter (as 
> confirmed up to now).
>
> *What is the first step from these postulates, to anything? I mean 
> anything. What is mechanism? *
>
>
> Mechanism is the hypothesis that our body is a machine, or a natural 
> machine-like entity. (It has been discussed in the antic China, India and 
> greek philosopher/theologians. But you need to wait Descartes and Diderot 
> to see it coming back, but, notably with Diderot, also its use by 
> materialists to hide the mind-body problem.
>
> Digital Mechanism as I use it in this list, is slightly more precise. The 
> notion of digital machine is the notion of Emil Post, Alonzo Church, Alan 
> Turing, and best explained by Stephen Kleene in his papers and book, 
> notably his “Introduction to Metamathematics” (1952). 
> Just ask me, and I gave more on this … after the June exams, as my 
> scheduling get tighter and tighter those days.
>
> *Why do we need these postulate to fix anything? *
>
>
> My goal was to reformulate the mind-body problem in the frame of the 
> Mechanist hypothesis in the cognitive science/philosophy-o-mind/theology.
> Unfortunately I have been asked to solve it, which I did, but that 
> requires some familiarity with Mathematical Logic, which is not well taught.
> Also, the solution is disliked by the “religious” materialists, and I have 
> underestimate the number of those in some academical circles, and their 
> influence (I got a price for my PhD which has disappears without 
> explanation, just to give one example …).
>



*What happened with your Ph'D? Are you associated with a university? Which 
one? Just curious. AG *

> *What is the problem you're trying to fix? *
>
>
> The mind-body problem. How a grey brain can create a color perception, for 
> example. 
>



*Unsolved IMO, Not a trivial problem. AG *

> But it is more deep than that, as eventually, Mechanism is shown 
> incompatible with materialism and/or physicalism, which is the actual 
> paradigm in most metaphysics and theologies.
>

*What is Materialism? If Mechanism is as you defined above -- that the body 
is a machine or like a machine -- why is it incompatible with materialism? 
AG*

> *How can these postulates explain consciousness? TIA, AG*
>
>
> Eventually by the logics of self-reference discovered by Gödel and Löb in 
> arithmetic, and axiomatised completely (at the propositional level) by 
> Solovay. 
>
> If you agree that for a conscious being, consciousness is true, non 
> doubtable, but also non provable and non definable, then it is long but not 
> difficult to show that all universal machine (in the sense of Church …) can 
> introspect itself (in the sense provided by Gödel), and discover some thing 
> obeying to those axiomatic description of consciousness. 99% of 
> consciousness is explained, + an explanation why the last 1% has to be felt 
> by the machine as utterly not explainable. 
>


*Introspection -- the great unsolved problem. I have difficulty giving you 
the benefit of the doubt if you believe in a silly theory such as the MWI. 
AG *

>
> More on this later, perhaps.
>
> Bruno 
>

-- 
You received this message because you are subscribed to the Google Groups 
"Everything List" group.
To unsubscribe from this group and stop receiving emails from it, send an email 
to everything-list+unsubscr...@googlegroups.com.
To post to this group, send email to everything-list@googlegroups.com.
Visit 

Re: Entanglement

2018-06-08 Thread Bruno Marchal

> On 8 Jun 2018, at 14:55, Bruce Kellett  wrote:
> 
> From: Bruno Marchal mailto:marc...@ulb.ac.be>>
>>> On 8 Jun 2018, at 02:32, Bruce Kellett < 
>>> bhkell...@optusnet.com.au 
>>> > wrote:
>>> 
>>> 
>>> The SWE does not give a preferred basis. Basing MWI on the Schrödinger 
>>> equation runs into the basis problem. Few MWI advocates actually take this 
>>> seriously. And they should.
>> 
>> The relative proportion of histories do not depend on the choice of the 
>> base, so the base we use are chosen endemically, like the present moment for 
>> example, in the whole of physics. Obviously, we needs brain to assess our 
>> results and communicating, and some works, like sure and others, justify the 
>> indexical importance of the position base, with respect to the branch where 
>> intelligence can develop.
> 
> What on earth are you talking about? The position basis is not well-defined 
> either. The Hilbert space corresponding to the position operator X has an 
> infinite number of possible bases -- just like any other Hilbert space. Any 
> linear vector space has an infinite number of possible bases. How do you 
> choose which one you are going to use? Talking about the relative proportion 
> of histories sounds just like the long-since refuted branch counting approach 
> to probabilities.

Measure is quite different from counting.


> And the probabilities for various outcomes most certainly depend on the 
> chosen base, as do the outcomes themselves.

Well, we can use what we call in French “le peigne de Dirac”. To make that 
precise Laurent Schwartz has invented the theory of distribution. I simplify 
things here. Consider that space has been quantised, like in Loop-Gravity or 
something. Here, you do a 1004 fallacy, with respect to the goal (helping 
Grayson to have an idea of what is QM-without-collapse).



> 
> 
> 
 In this situation, what is the role of the SWE since the wf is usually 
 asserted without any reference to it? Now consider a general case where 
 the wf for a system is determined using the SWE. Since the solution can be 
 expanded using difference bases, say E or p, does each possible expansion, 
 each implying a different possible set of measurements, imply a different 
 set of worlds using the SWE? TIA, AG
>>> 
>>> The Schrödinger equation merely gives the time evolution of the system. To 
>>> define the problem you have to specify a wave function. It is in the 
>>> expansion of this wave function in terms of a set of possible eigenvalues 
>>> that the preferred basis problem arises. So it is not really down to the SE 
>>> itself, it is a matter for the wave function. Each expansion basis defines 
>>> a set of worlds, and all bases give different worlds.
>> 
>> That is correct, but the choice of the basis don’t change the relative 
>> “proportion of histories”.
> 
> The choice of basis makes all the difference in the world.

Everett prove the contrary, and he convinced me when I read it. I found “his 
proof” used in many books on quantum computing, although with different 
motivation. Thee result of an experiment, obviously depend of what you measure, 
but when you embed the observer in the wave, you get that what they find is 
independent of the choice of the base used to describe the “observer” and the 
“observed”. If not, the MW would already be refuted.



> Now that we understand decoherence, the only bases that are useful are those 
> that are robust against environmental decoherence. That is why we don't see 
> superpositions of live and dead cats -- that superposition base is not robust.

No problem with this.


> 
>> It threats only the naïve conception of “worlds”, which has led to the works 
>> of Griffith and Omnes (and Gel Mann & Hartle). That works remains still a 
>> bit naïve with respect of the type of histories we can encounter in 
>> arithmetic.
> 
> The consistent histories approach is just another way of considering many 
> worlds. The histories are no more unique than are the worlds.

Yes, indeed. Only the math makes more sense here, but many-histories belongs to 
the "many-worlds” family, I agree. I prefer it, because the word “world” is 
much ambiguous, and never defined.

Bruno



> 
> Bruce
> 
> 
> -- 
> You received this message because you are subscribed to the Google Groups 
> "Everything List" group.
> To unsubscribe from this group and stop receiving emails from it, send an 
> email to everything-list+unsubscr...@googlegroups.com 
> .
> To post to this group, send email to everything-list@googlegroups.com 
> .
> Visit this group at https://groups.google.com/group/everything-list 
> .
> For more options, visit https://groups.google.com/d/optout 
> .

-- 
You received this message 

Re: Entanglement

2018-06-08 Thread Bruce Kellett

From: *Bruno Marchal* mailto:marc...@ulb.ac.be>>
On 8 Jun 2018, at 02:32, Bruce Kellett > wrote:



The SWE does not give a preferred basis. Basing MWI on the 
Schrödinger equation runs into the basis problem. Few MWI advocates 
actually take this seriously. And they should.


The relative proportion of histories do not depend on the choice of 
the base, so the base we use are chosen endemically, like the present 
moment for example, in the whole of physics. Obviously, we needs brain 
to assess our results and communicating, and some works, like sure and 
others, justify the indexical importance of the position base, with 
respect to the branch where intelligence can develop.


What on earth are you talking about? The position basis is not 
well-defined either. The Hilbert space corresponding to the position 
operator X has an infinite number of possible bases -- just like any 
other Hilbert space. Any linear vector space has an infinite number of 
possible bases. How do you choose which one you are going to use? 
Talking about the relative proportion of histories sounds just like the 
long-since refuted branch counting approach to probabilities. And the 
probabilities for various outcomes most certainly depend on the chosen 
base, as do the outcomes themselves.




*In this situation, what is the role of the SWE since the wf is 
usually asserted without any reference to it? Now consider a general 
case where the wf for a system is determined using the SWE. Since 
the solution can be expanded using difference bases, say E or p, 
does each possible expansion, each implying a different possible set 
of measurements, imply a different set of worlds using the SWE? TIA, AG*


The Schrödinger equation merely gives the time evolution of the 
system. To define the problem you have to specify a wave function. It 
is in the expansion of this wave function in terms of a set of 
possible eigenvalues that the preferred basis problem arises. So it 
is not really down to the SE itself, it is a matter for the wave 
function. Each expansion basis defines a set of worlds, and all bases 
give different worlds.


That is correct, but the choice of the basis don’t change the relative 
“proportion of histories”.


The choice of basis makes all the difference in the world. Now that we 
understand decoherence, the only bases that are useful are those that 
are robust against environmental decoherence. That is why we don't see 
superpositions of live and dead cats -- that superposition base is not 
robust.


It threats only the naïve conception of “worlds”, which has led to the 
works of Griffith and Omnes (and Gel Mann & Hartle). That works 
remains still a bit naïve with respect of the type of histories we can 
encounter in arithmetic.


The consistent histories approach is just another way of considering 
many worlds. The histories are no more unique than are the worlds.


Bruce

--
You received this message because you are subscribed to the Google Groups 
"Everything List" group.
To unsubscribe from this group and stop receiving emails from it, send an email 
to everything-list+unsubscr...@googlegroups.com.
To post to this group, send email to everything-list@googlegroups.com.
Visit this group at https://groups.google.com/group/everything-list.
For more options, visit https://groups.google.com/d/optout.


Re: Entanglement

2018-06-08 Thread Bruno Marchal

> On 8 Jun 2018, at 03:30, agrayson2...@gmail.com wrote:
> 
> On Thursday, June 7, 2018 at 9:07:37 AM UTC, Bruno Marchal wrote:
> 
> [snip]
> 
>So consciousness anticipates all quantum experiment that MIGHT occur 
> in the future,
> 
> The arithmetical relations do that. Consciousness only select the histories
> 
> and creates those worlds in anticipation? Now we're really getting 
> deep into woo-woo territory.
> 
> On the contrary, we explain how the quantum physical illusion arise from all 
> computations which are already realised in the block-mindspace given by very 
> elementary arithmetic, that we never leave.
> 
> Here are all my assumptions: classical logic + the axioms of arithmetic (“s” 
> is intended to denote the successor function x+1):
> 
>   Please describe ambiguous (for me) symbols,  AG

OK.



> 
> 0 ≠ s(x)OK
> s(x) = s(y) -> x = yOK
> x = 0 v Ey(x = s(y))?

A natural number is either null, or has a predecessor. Read “Ex” by it exists a 
number x such that ...




> x+0 = xOK
> x+s(y) = s(x+y) OK
> x*0=0?   Does * mean multiplication? AG

Yes. “x” looks to much like the variable x. 



> x*s(y)=(x*y)+x?

x multiplied by the successor of y gives the same as x * y + x. Exemple 6 * 4 = 
(6 * 3) + 6.




> 
> I use mechanism only to help people that this has to be a theory of 
> everything. It explains very well consciousness (I think), and matter (as 
> confirmed up to now).
> 
> What is the first step from these postulates, to anything? I mean anything. 
> What is mechanism?

Mechanism is the hypothesis that our body is a machine, or a natural 
machine-like entity. (It has been discussed in the antic China, India and greek 
philosopher/theologians. But you need to wait Descartes and Diderot to see it 
coming back, but, notably with Diderot, also its use by materialists to hide 
the mind-body problem.

Digital Mechanism as I use it in this list, is slightly more precise. The 
notion of digital machine is the notion of Emil Post, Alonzo Church, Alan 
Turing, and best explained by Stephen Kleene in his papers and book, notably 
his “Introduction to Metamathematics” (1952). 
Just ask me, and I gave more on this … after the June exams, as my scheduling 
get tighter and tighter those days.




> Why do we need these postulate to fix anything?

My goal was to reformulate the mind-body problem in the frame of the Mechanist 
hypothesis in the cognitive science/philosophy-o-mind/theology.
Unfortunately I have been asked to solve it, which I did, but that requires 
some familiarity with Mathematical Logic, which is not well taught.
Also, the solution is disliked by the “religious” materialists, and I have 
underestimate the number of those in some academical circles, and their 
influence (I got a price for my PhD which has disappears without explanation, 
just to give one example …).



> What is the problem you're trying to fix?

The mind-body problem. How a grey brain can create a color perception, for 
example. But it is more deep than that, as eventually, Mechanism is shown 
incompatible with materialism and/or physicalism, which is the actual paradigm 
in most metaphysics and theologies.



> How can these postulates explain consciousness? TIA, AG

Eventually by the logics of self-reference discovered by Gödel and Löb in 
arithmetic, and axiomatised completely (at the propositional level) by Solovay. 

If you agree that for a conscious being, consciousness is true, non doubtable, 
but also non provable and non definable, then it is long but not difficult to 
show that all universal machine (in the sense of Church …) can introspect 
itself (in the sense provided by Gödel), and discover some thing obeying to 
those axiomatic description of consciousness. 99% of consciousness is 
explained, + an explanation why the last 1% has to be felt by the machine as 
utterly not explainable. 

More on this later, perhaps.

Bruno 






> 
> Bruno
> 
> -- 
> You received this message because you are subscribed to the Google Groups 
> "Everything List" group.
> To unsubscribe from this group and stop receiving emails from it, send an 
> email to everything-list+unsubscr...@googlegroups.com 
> .
> To post to this group, send email to everything-list@googlegroups.com 
> .
> Visit this group at https://groups.google.com/group/everything-list 
> .
> For more options, visit https://groups.google.com/d/optout 
> .

-- 
You received this message because you are subscribed to the Google Groups 
"Everything List" group.
To unsubscribe from this group and stop receiving emails from it, send an email 
to everything-list+unsubscr...@googlegroups.com.
To post to this group, send email to everything-list@googlegroups.com.
Visit this 

Re: Entanglement

2018-06-08 Thread Bruno Marchal

> On 8 Jun 2018, at 02:32, Bruce Kellett  wrote:
> 
> From: mailto:agrayson2...@gmail.com>>
>> 
>> On Thursday, June 7, 2018 at 11:32:23 AM UTC, Bruce wrote:
>> From: >
>>> 
>>> On Tuesday, June 5, 2018 at 3:05:40 AM UTC, Bruce wrote:
>>> From: < agrays...@gmail.com 
>>> >
 
 On Tuesday, June 5, 2018 at 1:18:29 AM UTC, Bruce wrote:
 From: < agrays...@gmail.com 
 >
> 
> Remember that the analysis I have given above is schematic, representing 
> the general progression of unitary evolution. It is not specific to any 
> particular case, or any particular number of possible outcomes for the 
> experiment.
> 
> Bruce
> 
> OK. For economy we can write,  (|+>|e+> + |->|e->),  where e stands for 
> the entire universe other than the particle whose spin is being measured. 
> What is the status of the interference between the terms in this 
> superposition? For a quantum superposition to make sense, there must be 
> interference between the terms in the sum. At least that's my 
> understanding of the quantum principle of superposition. But the universe 
> excluding the particle being measured seems to have no definable wave 
> length; hence, I don't see that this superposition makes any sense in how 
> superposition is applied. Would appreciate your input on this issue. TIA, 
> AG
 
 A superposition is just a sum of vectors in Hilbert space. If these 
 vectors are orthogonal there is no interference between them. Your quest 
 for a wavelength in every superposition is the wrong way to look at 
 things. Macroscopic objects have vanishingly small deBroglie wavelengths, 
 but the can still be represented as vectors in a HIlbert space, so can 
 still form superpositions. I think you are looking for absolute 
 classicality in quantum phenomena -- that is impossible, by definition.
 
 Bruce
 
 If that's the case, why all the fuss about Schrodinger's cat? AG
>>> 
>>> Is there a fuss about Schrödinger's cat? Whatever fuss there is, is not 
>>> about the possibility of a superposition of live and dead cats. It is about 
>>> choosing the correct basis in which to describe the physical situation. The 
>>> Schrödinger equation does not specify a basis, and that is its main 
>>> drawback. In fact, that observation alone is sufficient to sink the naive 
>>> many-worlds enthusiast -- he doesn't know in which basis the multiplication 
>>> of worlds occurs.
>>> 
>>> Bruce
>>> 
>>> Interesting point. Do you mean that if one solved the SE for some standard 
>>> quantum problem (nothing fancy, no decoherence modeled), one can generally 
>>> expand the solution in different bases, say p, E, or x, and each expansion 
>>> would imply a different set of worlds using the MWI?  Are there other bases 
>>> besides these three? I'm thinking there could be an infinite set of basis 
>>> vectors since, by analogy, IIUC, for the simple 2-dimensional vector space 
>>> of "little pointy things", I think every pair of non co-linear vectors 
>>> could form a basis (so most bases are not orthogonal). AG
>> 
>> There are an indefinite number of possible sets of basis vectors in any 
>> Hilbert space. Think of the 2-dimensional space for a spin half particle -- 
>> one can form a set of orthonormal basis vectors for every direction in the 
>> 3-sphere. Different bases are not different observables such as p, E, or x. 
>> Each such observable has its own Hilbert space and an infinite set of 
>> possible bases. Each set of basis vectors is just a linearly independent set 
>> of sums over some other basis. It is easier to visualize this in the case of 
>> a simple linear vector space. Think of 3-dimensional Euclidean space. You 
>> can choose a set of three axes, but these can be rotated into any direction. 
>> Or linear combinations can be formed that are not necessarily orthogonal. 
>> For physical situations in QM, some bases are more useful than others, but 
>> the choice of basis is by no means unique.
>> 
>> Bruce
>> 
>> OK. I understand your comments .But let me rephrase the issues as I 
>> conflated some of them above. In the spin half case, were you claiming that 
>> each orientation of the SG device implies a different world according to the 
>> MWI, and if so, does the MWI make no sense since the SWE does not indicate 
>> which orientation is in play?
> 
> The SWE does not give a preferred basis. Basing MWI on the Schrödinger 
> equation runs into the basis problem. Few MWI advocates actually take this 
> seriously. And they should.

The relative proportion of histories do not depend on the choice of the base, 
so the base we use are chosen endemically, like the present moment for example, 
in the whole of physics. Obviously, we needs brain to assess our results and 
communicating, and some works, like 

Re: Entanglement

2018-06-07 Thread agrayson2000


On Friday, June 8, 2018 at 12:32:18 AM UTC, Bruce wrote:
>
> From: >
>
>
> On Thursday, June 7, 2018 at 11:32:23 AM UTC, Bruce wrote: 
>>
>> From: 
>>
>>
>> On Tuesday, June 5, 2018 at 3:05:40 AM UTC, Bruce wrote: 
>>>
>>> From: 
>>>
>>>
>>> On Tuesday, June 5, 2018 at 1:18:29 AM UTC, Bruce wrote: 

 From: 


 Remember that the analysis I have given above is schematic, 
 representing the general progression of unitary evolution. It is not 
 specific to any particular case, or any particular number of possible 
 outcomes for the experiment.

 Bruce

 *OK. For economy we can write,  ** (|+>|e+> + |->|e->),  where e 
 stands for the entire universe other than the particle whose spin is being 
 measured. What is the status of the interference between the terms in this 
 superposition? For a quantum superposition to make sense, there must be 
 interference between the terms in the sum. At least that's my 
 understanding 
 of the quantum principle of superposition. But the universe excluding the 
 particle being measured seems to have no definable wave length; hence, I 
 don't see that this superposition makes any sense in how superposition is 
 applied. Would appreciate your input on this issue. TIA, AG*


 A superposition is just a sum of vectors in Hilbert space. If these 
 vectors are orthogonal there is no interference between them. Your quest 
 for a wavelength in every superposition is the wrong way to look at 
 things. 
 Macroscopic objects have vanishingly small deBroglie wavelengths, but the 
 can still be represented as vectors in a HIlbert space, so can still form 
 superpositions. I think you are looking for absolute classicality in 
 quantum phenomena -- that is impossible, by definition.

 Bruce

>>>
>>> *If that's the case, why all the fuss about Schrodinger's cat? AG*
>>>
>>>
>>> Is there a fuss about Schrödinger's cat? Whatever fuss there is, is not 
>>> about the possibility of a superposition of live and dead cats. It is about 
>>> choosing the correct basis in which to describe the physical situation. The 
>>> Schrödinger equation does not specify a basis, and that is its main 
>>> drawback. In fact, that observation alone is sufficient to sink the naive 
>>> many-worlds enthusiast -- he doesn't know in which basis the multiplication 
>>> of worlds occurs.
>>>
>>> Bruce
>>>
>>
>> * Interesting point. Do you mean that if one solved the SE for some 
>> standard quantum problem (nothing fancy, no decoherence modeled), one can 
>> generally expand the solution in different bases, say p, E, or x, and each 
>> expansion would imply a different set of worlds using the MWI?  Are there 
>> other bases besides these three? I'm thinking there could be an infinite 
>> set of basis vectors since, by analogy, IIUC, for the simple 2-dimensional 
>> vector space of "little pointy things", I think every pair of non co-linear 
>> vectors could form a basis (so most bases are not orthogonal). AG*
>>
>>
>> There are an indefinite number of possible sets of basis vectors in any 
>> Hilbert space. Think of the 2-dimensional space for a spin half particle -- 
>> one can form a set of orthonormal basis vectors for every direction in the 
>> 3-sphere. Different bases are not different observables such as p, E, or x. 
>> Each such observable has its own Hilbert space and an infinite set of 
>> possible bases. Each set of basis vectors is just a linearly independent 
>> set of sums over some other basis. It is easier to visualize this in the 
>> case of a simple linear vector space. Think of 3-dimensional Euclidean 
>> space. You can choose a set of three axes, but these can be rotated into 
>> any direction. Or linear combinations can be formed that are not 
>> necessarily orthogonal. For physical situations in QM, some bases are more 
>> useful than others, but the choice of basis is by no means unique.
>>
>> Bruce
>>
>
> *OK. I understand your comments .But let me rephrase the issues as I 
> conflated some of them above. In the spin half case, were you claiming that 
> each orientation of the SG device implies a different world according to 
> the MWI, and if so, does the MWI make no sense since the SWE does not 
> indicate which orientation is in play?*
>
>
> The SWE does not give a preferred basis. Basing MWI on the Schrödinger 
> equation runs into the basis problem. Few MWI advocates actually take this 
> seriously. And they should.
>
> * In this situation, what is the role of the SWE since the wf is usually 
> asserted without any reference to it? Now consider a general case where the 
> wf for a system is determined using the SWE. Since the solution can be 
> expanded using difference bases, say E or p, does each possible expansion, 
> each implying a different possible set of measurements, imply a different 
> set of worlds using the SWE? TIA, AG*
>
>
> The Schrödinger 

Re: Entanglement

2018-06-07 Thread agrayson2000
On Thursday, June 7, 2018 at 9:07:37 AM UTC, Bruno Marchal wrote:

[snip]

*   So consciousness anticipates all quantum experiment that MIGHT 
occur in the future, *

The arithmetical relations do that. Consciousness only select the histories

*and creates those worlds in anticipation? Now we're really getting 
deep into woo-woo territory.*

On the contrary, we explain how the quantum physical illusion arise from 
all computations which are already realised in the block-mindspace given by 
very elementary arithmetic, that we never leave.

Here are all my assumptions: classical logic + the axioms of arithmetic 
(“s” is intended to denote the successor function x+1):

*  Please describe ambiguous (for me) symbols,  AG*

0 ≠ s(x)OK
s(x) = s(y) -> x = yOK
x = 0 v Ey(x = s(y))?
x+0 = xOK
x+s(y) = s(x+y) OK
x*0=0?   *Does * mean multiplication? AG*
x*s(y)=(x*y)+x?

I use mechanism only to help people that this has to be a theory of 
everything. It explains very well consciousness (I think), and matter (as 
confirmed up to now).

*What is the first step from these postulates, to anything? I mean 
anything. What is mechanism? Why do we need these postulate to fix 
anything? What is the problem you're trying to fix? How can these 
postulates explain consciousness? TIA, AG*

Bruno

-- 
You received this message because you are subscribed to the Google Groups 
"Everything List" group.
To unsubscribe from this group and stop receiving emails from it, send an email 
to everything-list+unsubscr...@googlegroups.com.
To post to this group, send email to everything-list@googlegroups.com.
Visit this group at https://groups.google.com/group/everything-list.
For more options, visit https://groups.google.com/d/optout.


Re: Entanglement

2018-06-07 Thread Bruce Kellett

From: mailto:agrayson2...@gmail.com>>


On Thursday, June 7, 2018 at 11:32:23 AM UTC, Bruce wrote:

From: 


On Tuesday, June 5, 2018 at 3:05:40 AM UTC, Bruce wrote:

From: 


On Tuesday, June 5, 2018 at 1:18:29 AM UTC, Bruce wrote:

From: 


Remember that the analysis I have given above is
schematic, representing the general progression of
unitary evolution. It is not specific to any particular
case, or any particular number of possible outcomes for
the experiment.

Bruce

*OK. For economy we can write, **(|+>|e+> + |->|e->), 
where e stands for the entire universe other than the

particle whose spin is being measured. What is the
status of the interference between the terms in this
superposition? For a quantum superposition to make
sense, there must be interference between the terms in
the sum. At least that's my understanding of the
quantum principle of superposition. But the universe
excluding the particle being measured seems to have no
definable wave length; hence, I don't see that this
superposition makes any sense in how superposition is
applied. Would appreciate your input on this issue.
TIA, AG*


A superposition is just a sum of vectors in Hilbert
space. If these vectors are orthogonal there is no
interference between them. Your quest for a wavelength
in every superposition is the wrong way to look at
things. Macroscopic objects have vanishingly small
deBroglie wavelengths, but the can still be represented
as vectors in a HIlbert space, so can still form
superpositions. I think you are looking for absolute
classicality in quantum phenomena -- that is impossible,
by definition.

Bruce


*If that's the case, why all the fuss about Schrodinger's
cat? AG*


Is there a fuss about Schrödinger's cat? Whatever fuss there
is, is not about the possibility of a superposition of live
and dead cats. It is about choosing the correct basis in
which to describe the physical situation. The Schrödinger
equation does not specify a basis, and that is its main
drawback. In fact, that observation alone is sufficient to
sink the naive many-worlds enthusiast -- he doesn't know in
which basis the multiplication of worlds occurs.

Bruce

*
Interesting point. Do you mean that if one solved the SE for some
standard quantum problem (nothing fancy, no decoherence modeled),
one can generally expand the solution in different bases, say p,
E, or x, and each expansion would imply a different set of worlds
using the MWI?  Are there other bases besides these three? I'm
thinking there could be an infinite set of basis vectors since,
by analogy, IIUC, for the simple 2-dimensional vector space of
"little pointy things", I think every pair of non co-linear
vectors could form a basis (so most bases are not orthogonal). AG*


There are an indefinite number of possible sets of basis vectors
in any Hilbert space. Think of the 2-dimensional space for a spin
half particle -- one can form a set of orthonormal basis vectors
for every direction in the 3-sphere. Different bases are not
different observables such as p, E, or x. Each such observable has
its own Hilbert space and an infinite set of possible bases. Each
set of basis vectors is just a linearly independent set of sums
over some other basis. It is easier to visualize this in the case
of a simple linear vector space. Think of 3-dimensional Euclidean
space. You can choose a set of three axes, but these can be
rotated into any direction. Or linear combinations can be formed
that are not necessarily orthogonal. For physical situations in
QM, some bases are more useful than others, but the choice of
basis is by no means unique.

Bruce


*OK. I understand your comments .But let me rephrase the issues as I 
conflated some of them above. In the spin half case, were you claiming 
that each orientation of the SG device implies a different world 
according to the MWI, and if so, does the MWI make no sense since the 
SWE does not indicate which orientation is in play?*


The SWE does not give a preferred basis. Basing MWI on the Schrödinger 
equation runs into the basis problem. Few MWI advocates actually take 
this seriously. And they should.


*In this situation, what is the role of the SWE since the wf is 
usually asserted without any reference to it? Now consider a general 
case where the wf for a system is determined using the SWE. Since the 
solution can be expanded using difference bases, say E or p, does each 

Re: Entanglement

2018-06-07 Thread agrayson2000


On Thursday, June 7, 2018 at 11:32:23 AM UTC, Bruce wrote:
>
> From: >
>
>
> On Tuesday, June 5, 2018 at 3:05:40 AM UTC, Bruce wrote: 
>>
>> From: 
>>
>>
>> On Tuesday, June 5, 2018 at 1:18:29 AM UTC, Bruce wrote: 
>>>
>>> From: 
>>>
>>>
>>> Remember that the analysis I have given above is schematic, representing 
>>> the general progression of unitary evolution. It is not specific to any 
>>> particular case, or any particular number of possible outcomes for the 
>>> experiment.
>>>
>>> Bruce
>>>
>>> *OK. For economy we can write,  ** (|+>|e+> + |->|e->),  where e stands 
>>> for the entire universe other than the particle whose spin is being 
>>> measured. What is the status of the interference between the terms in this 
>>> superposition? For a quantum superposition to make sense, there must be 
>>> interference between the terms in the sum. At least that's my understanding 
>>> of the quantum principle of superposition. But the universe excluding the 
>>> particle being measured seems to have no definable wave length; hence, I 
>>> don't see that this superposition makes any sense in how superposition is 
>>> applied. Would appreciate your input on this issue. TIA, AG*
>>>
>>>
>>> A superposition is just a sum of vectors in Hilbert space. If these 
>>> vectors are orthogonal there is no interference between them. Your quest 
>>> for a wavelength in every superposition is the wrong way to look at things. 
>>> Macroscopic objects have vanishingly small deBroglie wavelengths, but the 
>>> can still be represented as vectors in a HIlbert space, so can still form 
>>> superpositions. I think you are looking for absolute classicality in 
>>> quantum phenomena -- that is impossible, by definition.
>>>
>>> Bruce
>>>
>>
>> *If that's the case, why all the fuss about Schrodinger's cat? AG*
>>
>>
>> Is there a fuss about Schrödinger's cat? Whatever fuss there is, is not 
>> about the possibility of a superposition of live and dead cats. It is about 
>> choosing the correct basis in which to describe the physical situation. The 
>> Schrödinger equation does not specify a basis, and that is its main 
>> drawback. In fact, that observation alone is sufficient to sink the naive 
>> many-worlds enthusiast -- he doesn't know in which basis the multiplication 
>> of worlds occurs.
>>
>> Bruce
>>
>
> * Interesting point. Do you mean that if one solved the SE for some 
> standard quantum problem (nothing fancy, no decoherence modeled), one can 
> generally expand the solution in different bases, say p, E, or x, and each 
> expansion would imply a different set of worlds using the MWI?  Are there 
> other bases besides these three? I'm thinking there could be an infinite 
> set of basis vectors since, by analogy, IIUC, for the simple 2-dimensional 
> vector space of "little pointy things", I think every pair of non co-linear 
> vectors could form a basis (so most bases are not orthogonal). AG*
>
>
> There are an indefinite number of possible sets of basis vectors in any 
> Hilbert space. Think of the 2-dimensional space for a spin half particle -- 
> one can form a set of orthonormal basis vectors for every direction in the 
> 3-sphere. Different bases are not different observables such as p, E, or x. 
> Each such observable has its own Hilbert space and an infinite set of 
> possible bases. Each set of basis vectors is just a linearly independent 
> set of sums over some other basis. It is easier to visualize this in the 
> case of a simple linear vector space. Think of 3-dimensional Euclidean 
> space. You can choose a set of three axes, but these can be rotated into 
> any direction. Or linear combinations can be formed that are not 
> necessarily orthogonal. For physical situations in QM, some bases are more 
> useful than others, but the choice of basis is by no means unique.
>
> Bruce
>

*OK. I understand your comments .But let me rephrase the issues as I 
conflated some of them above. In the spin half case, were you claiming that 
each orientation of the SG device implies a different world according to 
the MWI, and if so, does the MWI make no sense since the SWE does not 
indicate which orientation is in play? In this situation, what is the role 
of the SWE since the wf is usually asserted without any reference to it? 
Now consider a general case where the wf for a system is determined using 
the SWE. Since the solution can be expanded using difference bases, say E 
or p, does each possible expansion, each implying a different possible set 
of measurements, imply a different set of worlds using the SWE? TIA, AG*

-- 
You received this message because you are subscribed to the Google Groups 
"Everything List" group.
To unsubscribe from this group and stop receiving emails from it, send an email 
to everything-list+unsubscr...@googlegroups.com.
To post to this group, send email to everything-list@googlegroups.com.
Visit this group at https://groups.google.com/group/everything-list.
For more options, visit 

Re: Entanglement

2018-06-07 Thread Bruce Kellett

From: mailto:agrayson2...@gmail.com>>


On Tuesday, June 5, 2018 at 3:05:40 AM UTC, Bruce wrote:

From: 


On Tuesday, June 5, 2018 at 1:18:29 AM UTC, Bruce wrote:

From: 


Remember that the analysis I have given above is schematic,
representing the general progression of unitary evolution.
It is not specific to any particular case, or any particular
number of possible outcomes for the experiment.

Bruce

*OK. For economy we can write, **(|+>|e+> + |->|e->),  where
e stands for the entire universe other than the particle
whose spin is being measured. What is the status of the
interference between the terms in this superposition? For a
quantum superposition to make sense, there must be
interference between the terms in the sum. At least that's
my understanding of the quantum principle of superposition.
But the universe excluding the particle being measured seems
to have no definable wave length; hence, I don't see that
this superposition makes any sense in how superposition is
applied. Would appreciate your input on this issue. TIA, AG*


A superposition is just a sum of vectors in Hilbert space. If
these vectors are orthogonal there is no interference between
them. Your quest for a wavelength in every superposition is
the wrong way to look at things. Macroscopic objects have
vanishingly small deBroglie wavelengths, but the can still be
represented as vectors in a HIlbert space, so can still form
superpositions. I think you are looking for absolute
classicality in quantum phenomena -- that is impossible, by
definition.

Bruce


*If that's the case, why all the fuss about Schrodinger's cat? AG*


Is there a fuss about Schrödinger's cat? Whatever fuss there is,
is not about the possibility of a superposition of live and dead
cats. It is about choosing the correct basis in which to describe
the physical situation. The Schrödinger equation does not specify
a basis, and that is its main drawback. In fact, that observation
alone is sufficient to sink the naive many-worlds enthusiast -- he
doesn't know in which basis the multiplication of worlds occurs.

Bruce

*
Interesting point. Do you mean that if one solved the SE for some 
standard quantum problem (nothing fancy, no decoherence modeled), one 
can generally expand the solution in different bases, say p, E, or x, 
and each expansion would imply a different set of worlds using the 
MWI?  Are there other bases besides these three? I'm thinking there 
could be an infinite set of basis vectors since, by analogy, IIUC, for 
the simple 2-dimensional vector space of "little pointy things", I 
think every pair of non co-linear vectors could form a basis (so most 
bases are not orthogonal). AG*


There are an indefinite number of possible sets of basis vectors in any 
Hilbert space. Think of the 2-dimensional space for a spin half particle 
-- one can form a set of orthonormal basis vectors for every direction 
in the 3-sphere. Different bases are not different observables such as 
p, E, or x. Each such observable has its own Hilbert space and an 
infinite set of possible bases. Each set of basis vectors is just a 
linearly independent set of sums over some other basis. It is easier to 
visualize this in the case of a simple linear vector space. Think of 
3-dimensional Euclidean space. You can choose a set of three axes, but 
these can be rotated into any direction. Or linear combinations can be 
formed that are not necessarily orthogonal. For physical situations in 
QM, some bases are more useful than others, but the choice of basis is 
by no means unique.


Bruce

--
You received this message because you are subscribed to the Google Groups 
"Everything List" group.
To unsubscribe from this group and stop receiving emails from it, send an email 
to everything-list+unsubscr...@googlegroups.com.
To post to this group, send email to everything-list@googlegroups.com.
Visit this group at https://groups.google.com/group/everything-list.
For more options, visit https://groups.google.com/d/optout.


Re: Entanglement

2018-06-07 Thread Bruno Marchal

> On 6 Jun 2018, at 03:17, Brent Meeker  wrote:
> 
> 
> 
> On 6/5/2018 5:05 PM, Bruce Kellett wrote:
>> From: mailto:agrayson2...@gmail.com>>
>>> 
>>> On Tuesday, June 5, 2018 at 7:02:11 PM UTC, Brent wrote:
>>> 
>>> 
>>> On 6/5/2018 2:48 AM, agrays...@gmail.com <> wrote:
 One objective was to convince myself whether the wf you have written for 
 decoherence makes any sense. Originally I thought one needed mutual 
 interference of all components for it to be viable. I doubted whether each 
 component interferes with the others in your proposed wf because the |e> 
 wave functions have no well defined deBroglie wave lengths (which I 
 thought were necessary for a valid quantum superposition).
>>> 
>>> de Broglie wave lengths are useful when thinking about a particle, but a 
>>> complex system with many degrees of freedom has many different energy 
>>> levels available to it and each one evolves with a different frequency.  So 
>>> the de Broglie wavelength is not very useful.
>>> 
>>> Brent
>>> 
>>> Agree. That's what Bruce wrote, in effect, when he noted that the macro 
>>> states in the superposition for decoherence are just symbols for the 
>>> multitude of entanglements, each presumably with its own deBroglie wave 
>>> length. But now I don't see the problem -- the weird implications --with 
>>> these superpositions involving macro systems as dependent on interference. 
>>> If the S Cat's wf can be written as a sum of two states, each entangled 
>>> with the radioactive source, the implication is that the Cat is 
>>> simultaneously alive and dead. It's like a simple vector in the plane -- 
>>> those pointy things -- which can be written as the sum of a horizontal and 
>>> vertical vectors (or a non orthogonal basis). If it can be written as a 
>>> sum, it can be interpreted as manifesting both vectors in the sum 
>>> simultaneously. So, if you want to write state vectors 
>>> to include entanglements with macro systems, you will get cats that are 
>>> alive and dead simultaneously, and in the decoherence case, you'll ger 
>>> copies of this universe, inclusive of copies of observers, etc. That was 
>>> Schrodinger's point; the fallacy of entangling quantum and macro states in 
>>> one wf. AG
>> 
>> Yes, Schrödinger's original intention with the cat scenario was to provide a 
>> reductio ad absurdum: the conclusion of cats being simultaneously alive and 
>> dead was patently absurd. In later life Schrödinger regretted introducing 
>> his wave equation. His idea had been to formulate quantum phenomena in terms 
>> of something easily visulizable in a classical way, such as wave motion. 
>> This was as an antidote to what many saw as the increasing obscurantism of 
>> Bohr and the Copenhagen school. However, he was disappointed by the results, 
>> and by the fact that his wave equation became the standard way of thinking 
>> about quantum processes. Schrödinger was undoubtedly aware that there were 
>> other ways of doing quantum calculations than in terms of his wave equation; 
>> Heisenberg matrix mechanics was already available, then there is Schwinger's 
>> mathematical approach, path integral approaches, and so on. None of which 
>> need mention a wave or a wave equation. They all give the same results for 
>> quantum probabilities so since they were equivalent in this sense, they 
>> were, according to the metaphysics of the time, all considered to be the 
>> same theory.
>> 
>> But it is doubtful if they are all actually the same theory, since they seem 
>> to imply different ontologies. Schrödinger's wave mechanics has led to the 
>> reification of the wave function itself, and the result is many worlds 
>> theory. But if you don't have a wave equation and work only with matrices, 
>> there is no reason to postulate any multiplicity of worlds. The model that 
>> you use for calculations in the theory implies an ontology, and not all 
>> implied ontologies are the same, or even equally useful. People are all to 
>> ready to believe that the simplest ontology of their model is what is 
>> "really real". But they are generally mistaken, as the negative induction of 
>> scientific realism points out.
>> 
>> Bruce
> 
> Well said.  Bohmian QM also gave the same answers as CI, but has a different 
> ontology and is deterministic.  It's randomness comes from ignorance of the 
> initial state.  CI postulates intrinsic randomness.  MWI is deterministic but 
> postulates ignorance of where you'll end up.  QBism assumes personal 
> randomness.

Bohmian QM is a different theory, it is QM-without collapse, but with 
metaphysical particles inference non locally by wave still representing the 
“other worlds”.

All formulation of QM without collapse leads to the same multiverse, but some 
adds selection principles, for reason of personal state, and usually, not in 
way which would make any experimental difference, so that is bad metaphysics, 
Imo.

Bruno



> 
> Brent
> 

Re: Entanglement

2018-06-07 Thread Bruno Marchal

> On 5 Jun 2018, at 05:41, agrayson2...@gmail.com wrote:
> 
> 
> 
> On Tuesday, June 5, 2018 at 3:25:25 AM UTC, agrays...@gmail.com wrote:
> 
> 
> On Monday, June 4, 2018 at 4:26:45 PM UTC, Bruno Marchal wrote:
> 
>> On 3 Jun 2018, at 23:37, agrays...@gmail.com <> wrote:
>> 
>> 
>> 
>> On Sunday, June 3, 2018 at 3:15:13 PM UTC, agrays...@gmail.com 
>>  wrote:
>> 
>> 
>> On Sunday, June 3, 2018 at 1:05:48 PM UTC, Bruno Marchal wrote:
>> 
>>> On 2 Jun 2018, at 00:23, agrays...@gmail.com <> wrote:
>>> 
>>> On Friday, June 1, 2018 at 4:43:29 PM UTC, agrays...@gmail.com 
>>>  wrote:
>>> 
>>> 
>>> On Friday, June 1, 2018 at 3:59:05 PM UTC, Bruno Marchal wrote:
>>> 
 On 31 May 2018, at 23:05, agrays...@gmail.com <> wrote:
 
 
 
 
 How can you have experimental evidence for many worlds if they are 
 disjoint from this world? AG 
 When mathematics points to things which don't exist, it's usually, maybe 
 always, the consequence of some unstated, erroneous assumption in its 
 application. As I previously explained, it's a fallacy to apply the 
 principle of superposition of states to entities that fail to have well 
 defined deBroglie wave lengths (and which therefore can manifest 
 interference) -- such as cats in a box, or instruments, or "environments". 
 That's what Schrodinger warned us about, but the lesson has yet to sink 
 in. AG
 Then quantum mechanics is false somewhere in between the observed and the 
 observer, but there are no evidences to back that claim. One history is no 
 less speculative than many one, and one history makes no sense with the 
 SWE for which evidences abound. Then, simple independent hypothesis leads 
 directly to many histories, so QM as known today do confirm those 
 independent hypothesis, like mechanism in the cognitive science (not in 
 physics).
 
 CMIIAW,
 ?
 
 Correct Me If I Am Wrong. AG 
>>> 
>>> 
>>> OK.
>>> 
>>> 
>>> 
>>> 
 but I think Everett used superpositions of macro states similar to what 
 Bruce wrote earlier, where sums of tensor products are formed using the 
 apparatus and environment.
 
 OK.
 
 That's what I surmised. Thanks for the confirmation. AG 
 All I claimed above is NOT that quantum mechanics is false, but rather 
 than one cannot form a legitimate superposition with entities that have no 
 well defined deBroglie wave length -- since the existence of a well 
 defined wave length is a necessary condition for interference, and that's 
 the core property of a superposition. So, if you indulge this error you 
 will get nonsense, such as a cat which is simultaneously alive and dead. AG
 We cannot measure the “precise wavelength” in practice, but that is not 
 needed to get the superposition state. Actually, you make the same remark 
 that de Broglie himself, who concluded that superposition applies only to 
 light atoms, and fade away on atomic distance.
 
 Nothing can be measured precisely. Do you have a link to his comment? AG
>>> 
>>> 
>>> It is in one of his many book in French. I think it is in its “La théorie 
>>> de la mesure en Mécanique Ondulatoire (Interpretation usuelle et 
>>> interpretation causale)”. He wrote this after retirement when he came back 
>>> to what could that theory means.
>>> 
>>> But today we can get the interference effects with superposed “big” 
>>> molecules, like the 60 carbon  ball, and cosmology indicated possible 
>>> interference between highly dense and massive object.
>>> 
>>> As I expressly stated in an earlier post, billiard and Bucky balls have 
>>> well defined deBroglie wave lengths and thus CAN be included in 
>>> superpositions. However, most macro objects do NOT have well defined 
>>> deBroglie wave lengths, 
>>> 
>>> 
>>> Wave length applies to continuous variables, but when you entangle the cat 
>>> with a spin state, as in Bohm thought experience, we use only the fact that 
>>> the state of the compound object is O, say, and the state of the cat is O * 
>>> cat *(up +down) = (O *(cat alive * up + cat-dead * down)) = O * cat alive 
>>> *up + O * cat dead * down. The two branch are part of the “wave”, or better 
>>> some state in some Hllbert Space (which is just an infinite linear space, 
>>> with some limits, and a scalar product).
>>> 
>>> 
>>> 
>>> 
 such as an instrument in a lab, the lab itself, and the general 
 environment, and CANNOT be included in a superposition,
>>> 
>>> The problem is that by lack of reasonable isolation any object in our hot 
>>> environment decoheres at the speed of light (or not far). The 
>>> “multiplication of the universe” start at each point of the front of the 
>>> universal wave. The universal wave is a sum of all its “front” wave in the 
>>> space-time, structure, except that with GR we have no clue on how to 
>>> proceed (well, The M theory 

Re: Entanglement

2018-06-07 Thread Bruno Marchal

> On 5 Jun 2018, at 05:05, Bruce Kellett  wrote:
> 
> From: mailto:agrayson2...@gmail.com>>
>> 
>> On Tuesday, June 5, 2018 at 1:18:29 AM UTC, Bruce wrote:
>> From: >
>>> 
>>> Remember that the analysis I have given above is schematic, representing 
>>> the general progression of unitary evolution. It is not specific to any 
>>> particular case, or any particular number of possible outcomes for the 
>>> experiment.
>>> 
>>> Bruce
>>> 
>>> OK. For economy we can write,  (|+>|e+> + |->|e->),  where e stands for the 
>>> entire universe other than the particle whose spin is being measured. What 
>>> is the status of the interference between the terms in this superposition? 
>>> For a quantum superposition to make sense, there must be interference 
>>> between the terms in the sum. At least that's my understanding of the 
>>> quantum principle of superposition. But the universe excluding the particle 
>>> being measured seems to have no definable wave length; hence, I don't see 
>>> that this superposition makes any sense in how superposition is applied. 
>>> Would appreciate your input on this issue. TIA, AG
>> 
>> A superposition is just a sum of vectors in Hilbert space. If these vectors 
>> are orthogonal there is no interference between them. Your quest for a 
>> wavelength in every superposition is the wrong way to look at things. 
>> Macroscopic objects have vanishingly small deBroglie wavelengths, but the 
>> can still be represented as vectors in a HIlbert space, so can still form 
>> superpositions. I think you are looking for absolute classicality in quantum 
>> phenomena -- that is impossible, by definition.
>> 
>> Bruce
>> 
>> If that's the case, why all the fuss about Schrodinger's cat? AG
> 
> Is there a fuss about Schrödinger's cat? Whatever fuss there is, is not about 
> the possibility of a superposition of live and dead cats. It is about 
> choosing the correct basis in which to describe the physical situation. The 
> Schrödinger equation does not specify a basis, and that is its main drawback. 
> In fact, that observation alone is sufficient to sink the naive many-worlds 
> enthusiast -- he doesn't know in which basis the multiplication of worlds 
> occurs.

I can agree, and this applies to many popular version of Everett MW, but 
Everett himself clarify this problem, by showing that the relative states gives 
the same measure, whatever the base is chosen. Deutsch was wrong on this fora 
time, but then understood that there is no base problem, as the multiplication 
are relative (which makes the mutilverse really a multi-multiverse). I have 
used this in my explanation of non-locality without FTL in such a setting.

Bruno



> 
> Bruce
> 
> 
> -- 
> You received this message because you are subscribed to the Google Groups 
> "Everything List" group.
> To unsubscribe from this group and stop receiving emails from it, send an 
> email to everything-list+unsubscr...@googlegroups.com 
> .
> To post to this group, send email to everything-list@googlegroups.com 
> .
> Visit this group at https://groups.google.com/group/everything-list 
> .
> For more options, visit https://groups.google.com/d/optout 
> .

-- 
You received this message because you are subscribed to the Google Groups 
"Everything List" group.
To unsubscribe from this group and stop receiving emails from it, send an email 
to everything-list+unsubscr...@googlegroups.com.
To post to this group, send email to everything-list@googlegroups.com.
Visit this group at https://groups.google.com/group/everything-list.
For more options, visit https://groups.google.com/d/optout.


Re: Entanglement

2018-06-06 Thread agrayson2000


On Tuesday, June 5, 2018 at 3:05:40 AM UTC, Bruce wrote:
>
> From: >
>
>
> On Tuesday, June 5, 2018 at 1:18:29 AM UTC, Bruce wrote: 
>>
>> From: 
>>
>>
>> Remember that the analysis I have given above is schematic, representing 
>> the general progression of unitary evolution. It is not specific to any 
>> particular case, or any particular number of possible outcomes for the 
>> experiment.
>>
>> Bruce
>>
>> *OK. For economy we can write,  ** (|+>|e+> + |->|e->),  where e stands 
>> for the entire universe other than the particle whose spin is being 
>> measured. What is the status of the interference between the terms in this 
>> superposition? For a quantum superposition to make sense, there must be 
>> interference between the terms in the sum. At least that's my understanding 
>> of the quantum principle of superposition. But the universe excluding the 
>> particle being measured seems to have no definable wave length; hence, I 
>> don't see that this superposition makes any sense in how superposition is 
>> applied. Would appreciate your input on this issue. TIA, AG*
>>
>>
>> A superposition is just a sum of vectors in Hilbert space. If these 
>> vectors are orthogonal there is no interference between them. Your quest 
>> for a wavelength in every superposition is the wrong way to look at things. 
>> Macroscopic objects have vanishingly small deBroglie wavelengths, but the 
>> can still be represented as vectors in a HIlbert space, so can still form 
>> superpositions. I think you are looking for absolute classicality in 
>> quantum phenomena -- that is impossible, by definition.
>>
>> Bruce
>>
>
> *If that's the case, why all the fuss about Schrodinger's cat? AG*
>
>
> Is there a fuss about Schrödinger's cat? Whatever fuss there is, is not 
> about the possibility of a superposition of live and dead cats. It is about 
> choosing the correct basis in which to describe the physical situation. The 
> Schrödinger equation does not specify a basis, and that is its main 
> drawback. In fact, that observation alone is sufficient to sink the naive 
> many-worlds enthusiast -- he doesn't know in which basis the multiplication 
> of worlds occurs.
>
> Bruce
>

*Interesting point. Do you mean that if one solved the SE for some standard 
quantum problem (nothing fancy, no decoherence modeled), one can generally 
expand the solution in different bases, say p, E, or x, and each expansion 
would imply a different set of worlds using the MWI?  Are there other bases 
besides these three? I'm thinking there could be an infinite set of basis 
vectors since, by analogy, IIUC, for the simple 2-dimensional vector space 
of "little pointy things", I think every pair of non co-linear vectors 
could form a basis (so most bases are not orthogonal). AG*

-- 
You received this message because you are subscribed to the Google Groups 
"Everything List" group.
To unsubscribe from this group and stop receiving emails from it, send an email 
to everything-list+unsubscr...@googlegroups.com.
To post to this group, send email to everything-list@googlegroups.com.
Visit this group at https://groups.google.com/group/everything-list.
For more options, visit https://groups.google.com/d/optout.


Re: Entanglement

2018-06-06 Thread agrayson2000


On Wednesday, June 6, 2018 at 1:17:22 AM UTC, Brent wrote:
>
>
>
> On 6/5/2018 5:05 PM, Bruce Kellett wrote:
>
> From: >
>
>
> On Tuesday, June 5, 2018 at 7:02:11 PM UTC, Brent wrote: 
>>
>>
>>
>> On 6/5/2018 2:48 AM, agrays...@gmail.com wrote:
>>
>> *One objective was to convince myself whether the wf you have written for 
>> decoherence makes any sense. Originally I thought one needed mutual 
>> interference of all components for it to be viable. I doubted whether each 
>> component interferes with the others in your proposed wf because the |e> 
>> wave functions have no well defined deBroglie wave lengths (which I thought 
>> were necessary for a valid quantum superposition).*
>>
>>
>> de Broglie wave lengths are useful when thinking about a particle, but a 
>> complex system with many degrees of freedom has many different energy 
>> levels available to it and each one evolves with a different frequency.  So 
>> the de Broglie wavelength is not very useful.
>>
>> Brent
>>
>
> *Agree. That's what Bruce wrote, in effect, when he noted that the macro 
> states in the superposition for decoherence are just symbols for the 
> multitude of entanglements, each presumably with its own deBroglie wave 
> length**. But now I don't see the problem -- the weird implications 
> --with these superpositions involving macro systems as dependent on 
> interference. If the S Cat's wf can be written as a sum of two states, each 
> entangled with the radioactive source, the implication is that the Cat is 
> simultaneously alive and dead. It's like a simple vector in the plane -- 
> those pointy things -- which can be written as the sum of a horizontal and 
> vertical vectors (or a non orthogonal basis). If it can be written as a 
> sum, it can be interpreted as manifesting both vectors in the sum 
> simultaneously. So, if you want to write state vectors to include 
> entanglements with macro systems, you will get cats that are alive and dead 
> simultaneously, and in the decoherence case, you'll ger copies of this 
> universe, inclusive of copies of observers, etc. That was Schrodinger's 
> point; the fallacy of entangling quantum and macro states in one wf. AG*
>
>
> Yes, Schrödinger's original intention with the cat scenario was to provide 
> a *reductio ad absurdum*: the conclusion of cats being simultaneously 
> alive and dead was patently absurd. In later life Schrödinger regretted 
> introducing his wave equation. His idea had been to formulate quantum 
> phenomena in terms of something easily visulizable in a classical way, such 
> as wave motion. This was as an antidote to what many saw as the increasing 
> obscurantism of Bohr and the Copenhagen school. However, he was 
> disappointed by the results, and by the fact that his wave equation became 
> the standard way of thinking about quantum processes. Schrödinger was 
> undoubtedly aware that there were other ways of doing quantum calculations 
> than in terms of his wave equation; Heisenberg matrix mechanics was already 
> available, then there is Schwinger's mathematical approach, path integral 
> approaches, and so on. None of which need mention a wave or a wave 
> equation. They all give the same results for quantum probabilities so since 
> they were equivalent in this sense, they were, according to the metaphysics 
> of the time, all considered to be the same theory.
>
> But it is doubtful if they are all actually the same theory, since they 
> seem to imply different ontologies. Schrödinger's wave mechanics has led to 
> the reification of the wave function itself, and the result is many worlds 
> theory. But if you don't have a wave equation and work only with matrices, 
> there is no reason to postulate any multiplicity of worlds. The model that 
> you use for calculations in the theory implies an ontology, and not all 
> implied ontologies are the same, or even equally useful. People are all to 
> ready to believe that the simplest ontology of their model is what is 
> "really real". But they are generally mistaken, as the negative induction 
> of scientific realism points out.
>
> Bruce
>
>
> Well said. 
>

Ditto.  AG
 

> Bohmian QM also gave the same answers as CI, but has a different ontology 
> and is deterministic.  It's randomness comes from ignorance of the initial 
> state.  CI postulates intrinsic randomness.  MWI is deterministic but 
> postulates ignorance of where you'll end up.  QBism assumes personal 
> randomness.
>
> Brent
>

-- 
You received this message because you are subscribed to the Google Groups 
"Everything List" group.
To unsubscribe from this group and stop receiving emails from it, send an email 
to everything-list+unsubscr...@googlegroups.com.
To post to this group, send email to everything-list@googlegroups.com.
Visit this group at https://groups.google.com/group/everything-list.
For more options, visit https://groups.google.com/d/optout.


Re: Entanglement

2018-06-05 Thread Brent Meeker



On 6/5/2018 5:05 PM, Bruce Kellett wrote:

From: mailto:agrayson2...@gmail.com>>


On Tuesday, June 5, 2018 at 7:02:11 PM UTC, Brent wrote:



On 6/5/2018 2:48 AM, agrays...@gmail.com wrote:

*One objective was to convince myself whether the wf you have
written for decoherence makes any sense. Originally I thought
one needed mutual interference of all components for it to be
viable. I doubted whether each component interferes with the
others in your proposed wf because the |e> wave functions have
no well defined deBroglie wave lengths (which I thought were
necessary for a valid quantum superposition).*


de Broglie wave lengths are useful when thinking about a
particle, but a complex system with many degrees of freedom has
many different energy levels available to it and each one evolves
with a different frequency.  So the de Broglie wavelength is not
very useful.

Brent


*Agree. That's what Bruce wrote, in effect, when he noted that the 
macro states in the superposition for decoherence are just symbols 
for the multitude of entanglements, each presumably with its own 
deBroglie wave length**. But now I don't see the problem -- the weird 
implications --with these superpositions involving macro systems as 
dependent on interference. If the S Cat's wf can be written as a sum 
of two states, each entangled with the radioactive source, the 
implication is that the Cat is simultaneously alive and dead. It's 
like a simple vector in the plane -- those pointy things -- which can 
be written as the sum of a horizontal and vertical vectors (or a non 
orthogonal basis). If it can be written as a sum, it can be 
interpreted as manifesting both vectors in the sum simultaneously. 
So, if you want to write state vectors to include entanglements with 
macro systems, you will get cats that are alive and dead 
simultaneously, and in the decoherence case, you'll ger copies of 
this universe, inclusive of copies of observers, etc. That was 
Schrodinger's point; the fallacy of entangling quantum and macro 
states in one wf. AG*


Yes, Schrödinger's original intention with the cat scenario was to 
provide a /reductio ad absurdum/: the conclusion of cats being 
simultaneously alive and dead was patently absurd. In later life 
Schrödinger regretted introducing his wave equation. His idea had been 
to formulate quantum phenomena in terms of something easily 
visulizable in a classical way, such as wave motion. This was as an 
antidote to what many saw as the increasing obscurantism of Bohr and 
the Copenhagen school. However, he was disappointed by the results, 
and by the fact that his wave equation became the standard way of 
thinking about quantum processes. Schrödinger was undoubtedly aware 
that there were other ways of doing quantum calculations than in terms 
of his wave equation; Heisenberg matrix mechanics was already 
available, then there is Schwinger's mathematical approach, path 
integral approaches, and so on. None of which need mention a wave or a 
wave equation. They all give the same results for quantum 
probabilities so since they were equivalent in this sense, they were, 
according to the metaphysics of the time, all considered to be the 
same theory.


But it is doubtful if they are all actually the same theory, since 
they seem to imply different ontologies. Schrödinger's wave mechanics 
has led to the reification of the wave function itself, and the result 
is many worlds theory. But if you don't have a wave equation and work 
only with matrices, there is no reason to postulate any multiplicity 
of worlds. The model that you use for calculations in the theory 
implies an ontology, and not all implied ontologies are the same, or 
even equally useful. People are all to ready to believe that the 
simplest ontology of their model is what is "really real". But they 
are generally mistaken, as the negative induction of scientific 
realism points out.


Bruce


Well said.  Bohmian QM also gave the same answers as CI, but has a 
different ontology and is deterministic.  It's randomness comes from 
ignorance of the initial state.  CI postulates intrinsic randomness.  
MWI is deterministic but postulates ignorance of where you'll end up.  
QBism assumes personal randomness.


Brent

--
You received this message because you are subscribed to the Google Groups 
"Everything List" group.
To unsubscribe from this group and stop receiving emails from it, send an email 
to everything-list+unsubscr...@googlegroups.com.
To post to this group, send email to everything-list@googlegroups.com.
Visit this group at https://groups.google.com/group/everything-list.
For more options, visit https://groups.google.com/d/optout.


Re: Entanglement

2018-06-05 Thread Bruce Kellett

From: mailto:agrayson2...@gmail.com>>


On Tuesday, June 5, 2018 at 7:02:11 PM UTC, Brent wrote:



On 6/5/2018 2:48 AM, agrays...@gmail.com wrote:

*One objective was to convince myself whether the wf you have
written for decoherence makes any sense. Originally I thought one
needed mutual interference of all components for it to be viable.
I doubted whether each component interferes with the others in
your proposed wf because the |e> wave functions have no well
defined deBroglie wave lengths (which I thought were necessary
for a valid quantum superposition).*


de Broglie wave lengths are useful when thinking about a particle,
but a complex system with many degrees of freedom has many
different energy levels available to it and each one evolves with
a different frequency.  So the de Broglie wavelength is not very
useful.

Brent


*Agree. That's what Bruce wrote, in effect, when he noted that the 
macro states in the superposition for decoherence are just symbols for 
the multitude of entanglements, each presumably with its own deBroglie 
wave length**. But now I don't see the problem -- the weird 
implications --with these superpositions involving macro systems as 
dependent on interference. If the S Cat's wf can be written as a sum 
of two states, each entangled with the radioactive source, the 
implication is that the Cat is simultaneously alive and dead. It's 
like a simple vector in the plane -- those pointy things -- which can 
be written as the sum of a horizontal and vertical vectors (or a non 
orthogonal basis). If it can be written as a sum, it can be 
interpreted as manifesting both vectors in the sum simultaneously. So, 
if you want to write state vectors to include entanglements with macro 
systems, you will get cats that are alive and dead simultaneously, and 
in the decoherence case, you'll ger copies of this universe, inclusive 
of copies of observers, etc. That was Schrodinger's point; the fallacy 
of entangling quantum and macro states in one wf. AG*


Yes, Schrödinger's original intention with the cat scenario was to 
provide a /reductio ad absurdum/: the conclusion of cats being 
simultaneously alive and dead was patently absurd. In later life 
Schrödinger regretted introducing his wave equation. His idea had been 
to formulate quantum phenomena in terms of something easily visulizable 
in a classical way, such as wave motion. This was as an antidote to what 
many saw as the increasing obscurantism of Bohr and the Copenhagen 
school. However, he was disappointed by the results, and by the fact 
that his wave equation became the standard way of thinking about quantum 
processes. Schrödinger was undoubtedly aware that there were other ways 
of doing quantum calculations than in terms of his wave equation; 
Heisenberg matrix mechanics was already available, then there is 
Schwinger's mathematical approach, path integral approaches, and so on. 
None of which need mention a wave or a wave equation. They all give the 
same results for quantum probabilities so since they were equivalent in 
this sense, they were, according to the metaphysics of the time, all 
considered to be the same theory.


But it is doubtful if they are all actually the same theory, since they 
seem to imply different ontologies. Schrödinger's wave mechanics has led 
to the reification of the wave function itself, and the result is many 
worlds theory. But if you don't have a wave equation and work only with 
matrices, there is no reason to postulate any multiplicity of worlds. 
The model that you use for calculations in the theory implies an 
ontology, and not all implied ontologies are the same, or even equally 
useful. People are all to ready to believe that the simplest ontology of 
their model is what is "really real". But they are generally mistaken, 
as the negative induction of scientific realism points out.


Bruce

--
You received this message because you are subscribed to the Google Groups 
"Everything List" group.
To unsubscribe from this group and stop receiving emails from it, send an email 
to everything-list+unsubscr...@googlegroups.com.
To post to this group, send email to everything-list@googlegroups.com.
Visit this group at https://groups.google.com/group/everything-list.
For more options, visit https://groups.google.com/d/optout.


Re: Entanglement

2018-06-05 Thread agrayson2000


On Tuesday, June 5, 2018 at 7:02:11 PM UTC, Brent wrote:
>
>
>
> On 6/5/2018 2:48 AM, agrays...@gmail.com  wrote:
>
> *One objective was to convince myself whether the wf you have written for 
> decoherence makes any sense. Originally I thought one needed mutual 
> interference of all components for it to be viable. I doubted whether each 
> component interferes with the others in your proposed wf because the |e> 
> wave functions have no well defined deBroglie wave lengths (which I thought 
> were necessary for a valid quantum superposition).*
>
>
> de Broglie wave lengths are useful when thinking about a particle, but a 
> complex system with many degrees of freedom has many different energy 
> levels available to it and each one evolves with a different frequency.  So 
> the de Broglie wavelength is not very useful.
>
> Brent
>

*Agree. That's what Bruce wrote, in effect, when he noted that the macro 
states in the superposition for decoherence are just symbols for the 
multitude of entanglements, each presumably with its own deBroglie wave 
length**. But now I don't see the problem -- the weird implications --with 
these superpositions involving macro systems as dependent on interference. 
If the S Cat's wf can be written as a sum of two states, each entangled 
with the radioactive source, the implication is that the Cat is 
simultaneously alive and dead. It's like a simple vector in the plane -- 
those pointy things -- which can be written as the sum of a horizontal and 
vertical vectors (or a non orthogonal basis). If it can be written as a 
sum, it can be interpreted as manifesting both vectors in the sum 
simultaneously. So, if you want to write state vectors to include 
entanglements with macro systems, you will get cats that are alive and dead 
simultaneously, and in the decoherence case, you'll ger copies of this 
universe, inclusive of copies of observers, etc. That was Schrodinger's 
point; the fallacy of entangling quantum and macro states in one wf. AG*

-- 
You received this message because you are subscribed to the Google Groups 
"Everything List" group.
To unsubscribe from this group and stop receiving emails from it, send an email 
to everything-list+unsubscr...@googlegroups.com.
To post to this group, send email to everything-list@googlegroups.com.
Visit this group at https://groups.google.com/group/everything-list.
For more options, visit https://groups.google.com/d/optout.


Re: Entanglement

2018-06-05 Thread Brent Meeker



On 6/5/2018 2:48 AM, agrayson2...@gmail.com wrote:
*One objective was to convince myself whether the wf you have written 
for decoherence makes any sense. Originally I thought one needed 
mutual interference of all components for it to be viable. I doubted 
whether each component interferes with the others in your proposed wf 
because the |e> wave functions have no well defined deBroglie wave 
lengths (which I thought were necessary for a valid quantum 
superposition).*


de Broglie wave lengths are useful when thinking about a particle, but a 
complex system with many degrees of freedom has many different energy 
levels available to it and each one evolves with a different frequency.  
So the concept de Broglie wavelength is not very useful.


Brent

--
You received this message because you are subscribed to the Google Groups 
"Everything List" group.
To unsubscribe from this group and stop receiving emails from it, send an email 
to everything-list+unsubscr...@googlegroups.com.
To post to this group, send email to everything-list@googlegroups.com.
Visit this group at https://groups.google.com/group/everything-list.
For more options, visit https://groups.google.com/d/optout.


Re: Entanglement

2018-06-05 Thread Brent Meeker



On 6/5/2018 2:05 AM, Bruce Kellett wrote:

From: mailto:agrayson2...@gmail.com>>


On Tuesday, June 5, 2018 at 7:03:28 AM UTC, Bruce wrote:

From: 


On Tuesday, June 5, 2018 at 1:18:29 AM UTC, Bruce wrote:

From: 


Remember that the analysis I have given above is schematic,
representing the general progression of unitary evolution.
It is not specific to any particular case, or any
particular number of possible outcomes for the experiment.

Bruce

*OK. For economy we can write, **(|+>|e+> + |->|e->), 
where e stands for the entire universe other than the
particle whose spin is being measured. What is the status
of the interference between the terms in this
superposition? For a quantum superposition to make sense,
there must be interference between the terms in the sum. At
least that's my understanding of the quantum principle of
superposition. But the universe excluding the particle
being measured seems to have no definable wave length;
hence, I don't see that this superposition makes any sense
in how superposition is applied. Would appreciate your
input on this issue. TIA, AG*


A superposition is just a sum of vectors in Hilbert space.
If these vectors are orthogonal there is no interference
between them.


*As a graduate student, in one of those standard problems,  I
seem to recall solving for the wf of some system using the SWE,
and then expanding the solution using an orthonormal set of
eigenfunctions as the basis (or maybe it was claimed there
exists such an expansion). Are you saying there is no
interference between the basis eigenvectors? TIA, AG*


Basis vectors need not be orthogonal. But if you choose and
orthonormal set then the individual basis vectors do not
interfere, though the superposition made up of such a set may be
thought of as an interference between them. But unless you take
the product of this superposition with something else, you do not
have interference cross terms. This is similar to the basis
problem I referred to earlier. If we have a basis of |dead> and
|alive>, being mutually orthogonal, then the basis vectors
(|dead>+|alive>) and (|dead>-|alive>) are also orthonormal. But
this basis is not stable under decoherence, so the environmental
states corresponding to this basis do interfere to produce the
stable |e_dead> and |e_alive> states.

Thus, if the orthogonal basis you choose is such that the
interaction of each basis vector with the environment leads to
orthogonal environment vectors, then there is no interference
between these environmental states -- this is how classical
states emerge.

Bruce

*
So in the case of the S Cat, the superposition,  ( |alive> 
|undecayed> + |dead>|decayed> ) ,  does NOT imply the cat is 
simultaneously alive and dead because the states in this 
superposition are orthogonal? If that's your conclusion, and if I am 
not misreading the discussions of this problem incorrectly, most, if 
not all of the texts which discuss this problem are completely 
misleading. Is that the situation? AG*


I am not sure exactly what you are wanting to discover in this 
discussion. From my perspective, interference is due to cross terms 
between states, so if there are no cross terms, such as if the states 
are orthogonal, then there is no interference. In the cat example, the 
cat is not both dead and alive simultaneously. If someone wants to 
claim that, then they are saying that the live and dead states are not 
actually orthogonal, so the original state can be re-established by 
the interference between live and dead cats. However, classical states 
do not interfere because the classical dead state is orthogonal to the 
alive state. In MWI, the term FAPP occurs here, because MWIers claim 
that the interference terms never vanish completely. So probably the 
issue comes down to how you react to FAPP orthogonality. If that means 
that the states can still interfere, then they are not actually 
orthogonal. If they can't interfere to reconstruct the original state, 
then decoherence has led to genuinely orthogonal states.


The MWI view is that the cat is in FAPP orthogonal states which include 
orthogonal environments, and obeservers, and hence constitute FAPP 
orthogonal "worlds".    So the cat exists in both these worlds, one 
alive and one dead, but with different probabilities.  This over 
simplifies the situation though.  The radioactive decay could happen at 
any time, and so there is a continuum of worlds, corresponding to the 
different times at which the cat died.


Brent

--
You received this message because you are subscribed to the Google Groups 
"Everything List" group.
To unsubscribe from this group and stop receiving emails from it, send an email 
to 

Re: Entanglement

2018-06-05 Thread agrayson2000


On Tuesday, June 5, 2018 at 9:05:13 AM UTC, Bruce wrote:
>
> From: >
>
>
> On Tuesday, June 5, 2018 at 7:03:28 AM UTC, Bruce wrote: 
>>
>> From: 
>>
>>
>> On Tuesday, June 5, 2018 at 1:18:29 AM UTC, Bruce wrote: 
>>>
>>> From: 
>>>
>>>
>>> Remember that the analysis I have given above is schematic, representing 
>>> the general progression of unitary evolution. It is not specific to any 
>>> particular case, or any particular number of possible outcomes for the 
>>> experiment.
>>>
>>> Bruce
>>>
>>> *OK. For economy we can write,  ** (|+>|e+> + |->|e->),  where e stands 
>>> for the entire universe other than the particle whose spin is being 
>>> measured. What is the status of the interference between the terms in this 
>>> superposition? For a quantum superposition to make sense, there must be 
>>> interference between the terms in the sum. At least that's my understanding 
>>> of the quantum principle of superposition. But the universe excluding the 
>>> particle being measured seems to have no definable wave length; hence, I 
>>> don't see that this superposition makes any sense in how superposition is 
>>> applied. Would appreciate your input on this issue. TIA, AG*
>>>
>>>
>>> A superposition is just a sum of vectors in Hilbert space. If these 
>>> vectors are orthogonal there is no interference between them. 
>>>
>>
>> *As a graduate student, in one of those standard problems,  I seem to 
>> recall solving for the wf of some system using the SWE, and then expanding 
>> the solution using an orthonormal set of eigenfunctions as the basis (or 
>> maybe it was claimed there exists such an expansion). Are you saying there 
>> is no interference between the basis eigenvectors? TIA, AG*
>>
>>
>> Basis vectors need not be orthogonal. But if you choose and orthonormal 
>> set then the individual basis vectors do not interfere, though the 
>> superposition made up of such a set may be thought of as an interference 
>> between them. But unless you take the product of this superposition with 
>> something else, you do not have interference cross terms. This is similar 
>> to the basis problem I referred to earlier. If we have a basis of |dead> 
>> and |alive>, being mutually orthogonal, then the basis vectors 
>> (|dead>+|alive>) and (|dead>-|alive>) are also orthonormal. But this basis 
>> is not stable under decoherence, so the environmental states corresponding 
>> to this basis do interfere to produce the stable |e_dead> and |e_alive> 
>> states.
>>
>> Thus, if the orthogonal basis you choose is such that the interaction of 
>> each basis vector with the environment leads to orthogonal environment 
>> vectors, then there is no interference between these environmental states 
>> -- this is how classical states emerge.
>>
>> Bruce
>>
>
> * So in the case of the S Cat, the superposition,  ( |alive> |undecayed> + 
> |dead>|decayed> ) ,  does NOT imply the cat is simultaneously alive and 
> dead because the states in this superposition are orthogonal? If that's 
> your conclusion, and if I am not misreading the discussions of this problem 
> incorrectly, most, if not all of the texts which discuss this problem are 
> completely misleading. Is that the situation? AG*
>
>
> I am not sure exactly what you are wanting to discover in this discussion. 
>

*One objective was to convince myself whether the wf you have written for 
decoherence makes any sense. Originally I thought one needed mutual 
interference of all components for it to be viable. I doubted whether each 
component interferes with the others in your proposed wf because the |e> 
wave functions have no well defined deBroglie wave lengths (which I thought 
were necessary for a valid quantum superposition). Now I am not sure about 
any of this. Maybe you can assess the status of my confusion. But first 
read what I wrote below about the S Cat. TIA, AG*
 

> From my perspective, interference is due to cross terms between states, so 
> if there are no cross terms, such as if the states are orthogonal, then 
> there is no interference.
>

*OK. AG*
 

> In the cat example, the cat is not both dead and alive simultaneously. If 
> someone wants to claim that, then they are saying that the live and dead 
> states are not actually orthogonal, so the original state can be 
> re-established by the interference between live and dead cats. However, 
> classical states do not interfere because the classical dead state is 
> orthogonal to the alive state.
>
 
*How do you know they're orthogonal? AFAIK, Alive or Dead do NOT have 
mathematical representations, so we can DEFINE them as orthogonal or not, 
depending on how we define their inner products. But regardless of whether 
they're orthogonal or not, the cat is in a superposition involving Alive 
and Dead states in the form I wrote earlier, the standard wf for this 
problem. But any vector which is a sum of other vectors, can be said to 
manifest all vectors in the sum simultaneously. So even without 

Re: Entanglement

2018-06-05 Thread Bruce Kellett

From: mailto:agrayson2...@gmail.com>>


On Tuesday, June 5, 2018 at 7:03:28 AM UTC, Bruce wrote:

From: 


On Tuesday, June 5, 2018 at 1:18:29 AM UTC, Bruce wrote:

From: 


Remember that the analysis I have given above is schematic,
representing the general progression of unitary evolution.
It is not specific to any particular case, or any particular
number of possible outcomes for the experiment.

Bruce

*OK. For economy we can write, **(|+>|e+> + |->|e->),  where
e stands for the entire universe other than the particle
whose spin is being measured. What is the status of the
interference between the terms in this superposition? For a
quantum superposition to make sense, there must be
interference between the terms in the sum. At least that's
my understanding of the quantum principle of superposition.
But the universe excluding the particle being measured seems
to have no definable wave length; hence, I don't see that
this superposition makes any sense in how superposition is
applied. Would appreciate your input on this issue. TIA, AG*


A superposition is just a sum of vectors in Hilbert space. If
these vectors are orthogonal there is no interference between
them.


*As a graduate student, in one of those standard problems,  I
seem to recall solving for the wf of some system using the SWE,
and then expanding the solution using an orthonormal set of
eigenfunctions as the basis (or maybe it was claimed there exists
such an expansion). Are you saying there is no interference
between the basis eigenvectors? TIA, AG*


Basis vectors need not be orthogonal. But if you choose and
orthonormal set then the individual basis vectors do not
interfere, though the superposition made up of such a set may be
thought of as an interference between them. But unless you take
the product of this superposition with something else, you do not
have interference cross terms. This is similar to the basis
problem I referred to earlier. If we have a basis of |dead> and
|alive>, being mutually orthogonal, then the basis vectors
(|dead>+|alive>) and (|dead>-|alive>) are also orthonormal. But
this basis is not stable under decoherence, so the environmental
states corresponding to this basis do interfere to produce the
stable |e_dead> and |e_alive> states.

Thus, if the orthogonal basis you choose is such that the
interaction of each basis vector with the environment leads to
orthogonal environment vectors, then there is no interference
between these environmental states -- this is how classical states
emerge.

Bruce

*
So in the case of the S Cat, the superposition,  ( |alive> |undecayed> 
+ |dead>|decayed> ) , does NOT imply the cat is simultaneously alive 
and dead because the states in this superposition are orthogonal? If 
that's your conclusion, and if I am not misreading the discussions of 
this problem incorrectly, most, if not all of the texts which discuss 
this problem are completely misleading. Is that the situation? AG*


I am not sure exactly what you are wanting to discover in this 
discussion. From my perspective, interference is due to cross terms 
between states, so if there are no cross terms, such as if the states 
are orthogonal, then there is no interference. In the cat example, the 
cat is not both dead and alive simultaneously. If someone wants to claim 
that, then they are saying that the live and dead states are not 
actually orthogonal, so the original state can be re-established by the 
interference between live and dead cats. However, classical states do 
not interfere because the classical dead state is orthogonal to the 
alive state. In MWI, the term FAPP occurs here, because MWIers claim 
that the interference terms never vanish completely. So probably the 
issue comes down to how you react to FAPP orthogonality. If that means 
that the states can still interfere, then they are not actually 
orthogonal. If they can't interfere to reconstruct the original state, 
then decoherence has led to genuinely orthogonal states.


Bruce

--
You received this message because you are subscribed to the Google Groups 
"Everything List" group.
To unsubscribe from this group and stop receiving emails from it, send an email 
to everything-list+unsubscr...@googlegroups.com.
To post to this group, send email to everything-list@googlegroups.com.
Visit this group at https://groups.google.com/group/everything-list.
For more options, visit https://groups.google.com/d/optout.


Re: Entanglement

2018-06-05 Thread agrayson2000


On Tuesday, June 5, 2018 at 7:03:28 AM UTC, Bruce wrote:
>
> From: >
>
>
> On Tuesday, June 5, 2018 at 1:18:29 AM UTC, Bruce wrote: 
>>
>> From: 
>>
>>
>> Remember that the analysis I have given above is schematic, representing 
>> the general progression of unitary evolution. It is not specific to any 
>> particular case, or any particular number of possible outcomes for the 
>> experiment.
>>
>> Bruce
>>
>> *OK. For economy we can write,  ** (|+>|e+> + |->|e->),  where e stands 
>> for the entire universe other than the particle whose spin is being 
>> measured. What is the status of the interference between the terms in this 
>> superposition? For a quantum superposition to make sense, there must be 
>> interference between the terms in the sum. At least that's my understanding 
>> of the quantum principle of superposition. But the universe excluding the 
>> particle being measured seems to have no definable wave length; hence, I 
>> don't see that this superposition makes any sense in how superposition is 
>> applied. Would appreciate your input on this issue. TIA, AG*
>>
>>
>> A superposition is just a sum of vectors in Hilbert space. If these 
>> vectors are orthogonal there is no interference between them. 
>>
>
> *As a graduate student, in one of those standard problems,  I seem to 
> recall solving for the wf of some system using the SWE, and then expanding 
> the solution using an orthonormal set of eigenfunctions as the basis (or 
> maybe it was claimed there exists such an expansion). Are you saying there 
> is no interference between the basis eigenvectors? TIA, AG*
>
>
> Basis vectors need not be orthogonal. But if you choose and orthonormal 
> set then the individual basis vectors do not interfere, though the 
> superposition made up of such a set may be thought of as an interference 
> between them. But unless you take the product of this superposition with 
> something else, you do not have interference cross terms. This is similar 
> to the basis problem I referred to earlier. If we have a basis of |dead> 
> and |alive>, being mutually orthogonal, then the basis vectors 
> (|dead>+|alive>) and (|dead>-|alive>) are also orthonormal. But this basis 
> is not stable under decoherence, so the environmental states corresponding 
> to this basis do interfere to produce the stable |e_dead> and |e_alive> 
> states.
>
> Thus, if the orthogonal basis you choose is such that the interaction of 
> each basis vector with the environment leads to orthogonal environment 
> vectors, then there is no interference between these environmental states 
> -- this is how classical states emerge.
>
> Bruce
>

*So in the case of the S Cat, the superposition,  ( |alive> |undecayed> + 
|dead>|decayed> ) ,  does NOT imply the cat is simultaneously alive and 
dead because the states in this superposition are orthogonal? If that's 
your conclusion, and if I am not misreading the discussions of this problem 
incorrectly, most, if not all of the texts which discuss this problem are 
completely misleading. Is that the situation? AG*

-- 
You received this message because you are subscribed to the Google Groups 
"Everything List" group.
To unsubscribe from this group and stop receiving emails from it, send an email 
to everything-list+unsubscr...@googlegroups.com.
To post to this group, send email to everything-list@googlegroups.com.
Visit this group at https://groups.google.com/group/everything-list.
For more options, visit https://groups.google.com/d/optout.


Re: Entanglement

2018-06-05 Thread Bruce Kellett

From: mailto:agrayson2...@gmail.com>>


On Tuesday, June 5, 2018 at 1:18:29 AM UTC, Bruce wrote:

From: 


Remember that the analysis I have given above is schematic,
representing the general progression of unitary evolution. It is
not specific to any particular case, or any particular number of
possible outcomes for the experiment.

Bruce

*OK. For economy we can write, **(|+>|e+> + |->|e->),  where e
stands for the entire universe other than the particle whose spin
is being measured. What is the status of the interference between
the terms in this superposition? For a quantum superposition to
make sense, there must be interference between the terms in the
sum. At least that's my understanding of the quantum principle of
superposition. But the universe excluding the particle being
measured seems to have no definable wave length; hence, I don't
see that this superposition makes any sense in how superposition
is applied. Would appreciate your input on this issue. TIA, AG*


A superposition is just a sum of vectors in Hilbert space. If
these vectors are orthogonal there is no interference between them.


*As a graduate student, in one of those standard problems,  I seem to 
recall solving for the wf of some system using the SWE, and then 
expanding the solution using an orthonormal set of eigenfunctions as 
the basis (or maybe it was claimed there exists such an expansion). 
Are you saying there is no interference between the basis 
eigenvectors? TIA, AG*


Basis vectors need not be orthogonal. But if you choose and orthonormal 
set then the individual basis vectors do not interfere, though the 
superposition made up of such a set may be thought of as an interference 
between them. But unless you take the product of this superposition with 
something else, you do not have interference cross terms. This is 
similar to the basis problem I referred to earlier. If we have a basis 
of |dead> and |alive>, being mutually orthogonal, then the basis vectors 
(|dead>+|alive>) and (|dead>-|alive>) are also orthonormal. But this 
basis is not stable under decoherence, so the environmental states 
corresponding to this basis do interfere to produce the stable |e_dead> 
and |e_alive> states.


Thus, if the orthogonal basis you choose is such that the interaction of 
each basis vector with the environment leads to orthogonal environment 
vectors, then there is no interference between these environmental 
states -- this is how classical states emerge.


Bruce

--
You received this message because you are subscribed to the Google Groups 
"Everything List" group.
To unsubscribe from this group and stop receiving emails from it, send an email 
to everything-list+unsubscr...@googlegroups.com.
To post to this group, send email to everything-list@googlegroups.com.
Visit this group at https://groups.google.com/group/everything-list.
For more options, visit https://groups.google.com/d/optout.


Re: Entanglement

2018-06-05 Thread agrayson2000


On Tuesday, June 5, 2018 at 1:18:29 AM UTC, Bruce wrote:
>
> From: >
>
>
> Remember that the analysis I have given above is schematic, representing 
> the general progression of unitary evolution. It is not specific to any 
> particular case, or any particular number of possible outcomes for the 
> experiment.
>
> Bruce
>
> *OK. For economy we can write,  ** (|+>|e+> + |->|e->),  where e stands 
> for the entire universe other than the particle whose spin is being 
> measured. What is the status of the interference between the terms in this 
> superposition? For a quantum superposition to make sense, there must be 
> interference between the terms in the sum. At least that's my understanding 
> of the quantum principle of superposition. But the universe excluding the 
> particle being measured seems to have no definable wave length; hence, I 
> don't see that this superposition makes any sense in how superposition is 
> applied. Would appreciate your input on this issue. TIA, AG*
>
>
> A superposition is just a sum of vectors in Hilbert space. If these 
> vectors are orthogonal there is no interference between them. 
>

*As a graduate student, in one of those standard problems,  I seem to 
recall solving for the wf of some system using the SWE, and then expanding 
the solution using an orthonormal set of eigenfunctions as the basis (or 
maybe it was claimed there exists such an expansion). Are you saying there 
is no interference between the basis eigenvectors? TIA, AG*

Your quest for a wavelength in every superposition is the wrong way to look 
> at things. Macroscopic objects have vanishingly small deBroglie 
> wavelengths, but the can still be represented as vectors in a HIlbert 
> space, so can still form superpositions. I think you are looking for 
> absolute classicality in quantum phenomena -- that is impossible, by 
> definition.
>
> Bruce
>

-- 
You received this message because you are subscribed to the Google Groups 
"Everything List" group.
To unsubscribe from this group and stop receiving emails from it, send an email 
to everything-list+unsubscr...@googlegroups.com.
To post to this group, send email to everything-list@googlegroups.com.
Visit this group at https://groups.google.com/group/everything-list.
For more options, visit https://groups.google.com/d/optout.


Re: Entanglement

2018-06-05 Thread 'scerir' via Everything List


> Il 5 giugno 2018 alle 5.05 Bruce Kellett  ha 
> scritto:
> 
> From: mailto:agrayson2...@gmail.com >
> 
> > > 
> > On Tuesday, June 5, 2018 at 1:18:29 AM UTC, Bruce wrote:
> > 
> > > > > From: 
> > > 
> > > > > > > 
> > > > Remember that the analysis I have given above is 
> > > > schematic, representing the general progression of unitary evolution. 
> > > > It is not specific to any particular case, or any particular number of 
> > > > possible outcomes for the experiment.
> > > > 
> > > > Bruce
> > > > 
> > > > OK. For economy we can write,  (|+>|e+> + |->|e->),  
> > > > where e stands for the entire universe other than the particle whose 
> > > > spin is being measured. What is the status of the interference between 
> > > > the terms in this superposition? For a quantum superposition to make 
> > > > sense, there must be interference between the terms in the sum. At 
> > > > least that's my understanding of the quantum principle of 
> > > > superposition. But the universe excluding the particle being measured 
> > > > seems to have no definable wave length; hence, I don't see that this 
> > > > superposition makes any sense in how superposition is applied. Would 
> > > > appreciate your input on this issue. TIA, AG
> > > > 
> > > > > > > A superposition is just a sum of vectors 
> > > > in Hilbert space. If these vectors are orthogonal there is no 
> > > > interference between them. Your quest for a wavelength in every 
> > > > superposition is the wrong way to look at things. Macroscopic objects 
> > > > have vanishingly small deBroglie wavelengths, but the can still be 
> > > > represented as vectors in a HIlbert space, so can still form 
> > > > superpositions. I think you are looking for absolute classicality in 
> > > > quantum phenomena -- that is impossible, by definition.
> > > 
> > > Bruce
> > > 
> > > > > 
> > If that's the case, why all the fuss about Schrodinger's cat? AG
> > 
> > > Is there a fuss about Schrödinger's cat? Whatever fuss there is, 
> > is not about the possibility of a superposition of live and dead cats. It 
> > is about choosing the correct basis in which to describe the physical 
> > situation. The Schrödinger equation does not specify a basis, and that is 
> > its main drawback. In fact, that observation alone is sufficient to sink 
> > the naive many-worlds enthusiast -- he doesn't know in which basis the 
> > multiplication of worlds occurs.
> 
> Bruce
> 
> 
"In this article, we demonstrate that we can measure the de Broglie wavelength 
of a two-photon wave packet (biphoton) with a Young double-slit experiment. The 
incident two-photon wave packet is generated collinearly from a nonlinear 
crystal by the process of spontaneous parametric down-conversion. The photons 
transmitted by the double slit form a fourth-order pattern which
is a superposition of two Young interference patterns with different 
periodicity. One of them results from the interference of the individual 
photons (“the parts of the object” [in J. Jacobson, G. Björk, I. Chuang, and Y. 
Yamamoto, Phys. Rev. Lett. 74, 4835 (1995]) and has an oscillation period of 
lambda_0. The other pattern is due to the interference of the “object as a 
whole with itself,” i.e., the interference of the “biphoton” and shows a 
periodicity of (lambda_0) / 2."

Measurement of the de Broglie Wavelength of a Multiphoton Wave Packet
E. J. S. Fonseca, C. H. Monken, and S. Pádua, PHYSICAL REVIEW LETTERS, 5 APRIL 
1999, VOLUME 82, NUMBER 14

https://tinyurl.com/ya5rxn8a
https://tinyurl.com/yatbb4ku





 
 

-- 
You received this message because you are subscribed to the Google Groups 
"Everything List" group.
To unsubscribe from this group and stop receiving emails from it, send an email 
to everything-list+unsubscr...@googlegroups.com.
To post to this group, send email to everything-list@googlegroups.com.
Visit this group at https://groups.google.com/group/everything-list.
For more options, visit https://groups.google.com/d/optout.


Re: Entanglement

2018-06-04 Thread agrayson2000


On Tuesday, June 5, 2018 at 3:05:40 AM UTC, Bruce wrote:
>
> From: >
>
>
> On Tuesday, June 5, 2018 at 1:18:29 AM UTC, Bruce wrote: 
>>
>> From: 
>>
>>
>> Remember that the analysis I have given above is schematic, representing 
>> the general progression of unitary evolution. It is not specific to any 
>> particular case, or any particular number of possible outcomes for the 
>> experiment.
>>
>> Bruce
>>
>> *OK. For economy we can write,  ** (|+>|e+> + |->|e->),  where e stands 
>> for the entire universe other than the particle whose spin is being 
>> measured. What is the status of the interference between the terms in this 
>> superposition? For a quantum superposition to make sense, there must be 
>> interference between the terms in the sum. At least that's my understanding 
>> of the quantum principle of superposition. But the universe excluding the 
>> particle being measured seems to have no definable wave length; hence, I 
>> don't see that this superposition makes any sense in how superposition is 
>> applied. Would appreciate your input on this issue. TIA, AG*
>>
>>
>> A superposition is just a sum of vectors in Hilbert space. If these 
>> vectors are orthogonal there is no interference between them. Your quest 
>> for a wavelength in every superposition is the wrong way to look at things. 
>> Macroscopic objects have vanishingly small deBroglie wavelengths, but the 
>> can still be represented as vectors in a HIlbert space, so can still form 
>> superpositions. I think you are looking for absolute classicality in 
>> quantum phenomena -- that is impossible, by definition.
>>
>> Bruce
>>
>
> *If that's the case, why all the fuss about Schrodinger's cat? AG*
>
>
> Is there a fuss about Schrödinger's cat? Whatever fuss there is, is not 
> about the possibility of a superposition of live and dead cats. It is about 
> choosing the correct basis in which to describe the physical situation. The 
> Schrödinger equation does not specify a basis, and that is its main 
> drawback. In fact, that observation alone is sufficient to sink the naive 
> many-worlds enthusiast -- he doesn't know in which basis the multiplication 
> of worlds occurs.
>
> Bruce
>

*Every description of the cat paradox I have read involves the assertion 
that since the cat is entangled with the radioactive source, and assuming 
its wf can be written as the sum of two terms involving the possible states 
of the composite entangled system, the cat is simultaneously alive and dead 
before the box is opened. Isn't that description dependent on the claim 
that the superposition implies interference among its constituent wf's, or 
as is generally said -- that the cat is in both alive and dead states 
simultaneously? AG*

-- 
You received this message because you are subscribed to the Google Groups 
"Everything List" group.
To unsubscribe from this group and stop receiving emails from it, send an email 
to everything-list+unsubscr...@googlegroups.com.
To post to this group, send email to everything-list@googlegroups.com.
Visit this group at https://groups.google.com/group/everything-list.
For more options, visit https://groups.google.com/d/optout.


Re: Entanglement

2018-06-04 Thread agrayson2000


On Tuesday, June 5, 2018 at 3:25:25 AM UTC, agrays...@gmail.com wrote:
>
>
>
> On Monday, June 4, 2018 at 4:26:45 PM UTC, Bruno Marchal wrote:
>>
>>
>> On 3 Jun 2018, at 23:37, agrays...@gmail.com wrote:
>>
>>
>>
>> On Sunday, June 3, 2018 at 3:15:13 PM UTC, agrays...@gmail.com wrote:
>>>
>>>
>>>
>>> On Sunday, June 3, 2018 at 1:05:48 PM UTC, Bruno Marchal wrote:


 On 2 Jun 2018, at 00:23, agrays...@gmail.com wrote:

 On Friday, June 1, 2018 at 4:43:29 PM UTC, agrays...@gmail.com wrote:
>
>
>
> On Friday, June 1, 2018 at 3:59:05 PM UTC, Bruno Marchal wrote:
>>
>>
>> On 31 May 2018, at 23:05, agrays...@gmail.com wrote:
>>
>>
>> 
>>>
>>>
>>> *How can you have experimental evidence for many worlds if they are 
>>> disjoint from this world? AG *
>>>
>>> *When mathematics points to things which don't exist, it's usually, 
>>> maybe always, the consequence of some unstated, erroneous assumption in 
>>> its 
>>> application. As I previously explained, it's a fallacy to apply the 
>>> principle of superposition of states to entities that fail to have well 
>>> defined deBroglie wave lengths (and which therefore can manifest 
>>> interference) -- such as cats in a box, or instruments, or 
>>> "environments". 
>>> That's what Schrodinger warned us about, but the lesson has yet to sink 
>>> in. 
>>> AG*
>>>
>>> Then quantum mechanics is false somewhere in between the observed 
>>> and the observer, but there are no evidences to back that claim. One 
>>> history is no less speculative than many one, and one history makes no 
>>> sense with the SWE for which evidences abound. Then, simple independent 
>>> hypothesis leads directly to many histories, so QM as known today do 
>>> confirm those independent hypothesis, like mechanism in the cognitive 
>>> science (not in physics).
>>>
>>>
>>> *CMIIAW,*
>>>
>>> ?
>>>
>>
>>
>> *Correct Me If I Am Wrong. AG *
>>
>>
>>
>> OK.
>>
>>
>>
>>
>> *but I think Everett used superpositions of macro states similar to 
>>> what Bruce wrote earlier, where sums of tensor products are formed 
>>> using 
>>> the apparatus and environment. *
>>>
>>>
>>> OK.
>>>
>>
>>
>> *That's what I surmised. Thanks for the confirmation. AG *
>>
>>> *All I claimed above is NOT that quantum mechanics is false, but 
>>> rather than one cannot form a legitimate superposition with entities 
>>> that 
>>> have no well defined deBroglie wave length -- since the existence of a 
>>> well 
>>> defined wave length is a necessary condition for interference, and 
>>> that's 
>>> the core property of a superposition. So, if you indulge this error you 
>>> will get nonsense, such as a cat which is simultaneously alive and 
>>> dead. AG*
>>>
>>> We cannot measure the “precise wavelength” in practice, but that is 
>>> not needed to get the superposition state. Actually, you make the same 
>>> remark that de Broglie himself, who concluded that superposition 
>>> applies 
>>> only to light atoms, and fade away on atomic distance. 
>>>
>>
>>
>> *Nothing can be measured precisely. Do you have a link to his 
>> comment? AG*
>>
>>
>>
>> It is in one of his many book in French. I think it is in its “La 
>> théorie de la mesure en Mécanique Ondulatoire (Interpretation usuelle et 
>> interpretation causale)”. He wrote this after retirement when he came 
>> back 
>> to what could that theory means.
>>
>
>> But today we can get the interference effects with superposed “big” 
>>> molecules, like the 60 carbon  ball, and cosmology indicated possible 
>>> interference between highly dense and massive object. 
>>>
>>
>> *As I expressly stated in an earlier post, billiard and Bucky balls 
>> have well defined deBroglie wave lengths and thus CAN be included in 
>> superpositions. However, most macro objects do NOT have well defined 
>> deBroglie wave lengths, *
>>
>>
>> Wave length applies to continuous variables, but when you entangle 
>> the cat with a spin state, as in Bohm thought experience, we use only 
>> the 
>> fact that the state of the compound object is O, say, and the state of 
>> the 
>> cat is O * cat *(up +down) = (O *(cat alive * up + cat-dead * down)) = O 
>> * 
>> cat alive *up + O * cat dead * down. The two branch are part of the 
>> “wave”, 
>> or better some state in some Hllbert Space (which is just an infinite 
>> linear space, with some limits, and a scalar product).
>>
>>
>>
>>
>> *such as an instrument in a lab, the lab itself, and the general 
>> environment, and CANNOT be included in a superposition,*
>>

Re: Entanglement

2018-06-04 Thread agrayson2000


On Monday, June 4, 2018 at 4:26:45 PM UTC, Bruno Marchal wrote:
>
>
> On 3 Jun 2018, at 23:37, agrays...@gmail.com  wrote:
>
>
>
> On Sunday, June 3, 2018 at 3:15:13 PM UTC, agrays...@gmail.com wrote:
>>
>>
>>
>> On Sunday, June 3, 2018 at 1:05:48 PM UTC, Bruno Marchal wrote:
>>>
>>>
>>> On 2 Jun 2018, at 00:23, agrays...@gmail.com wrote:
>>>
>>> On Friday, June 1, 2018 at 4:43:29 PM UTC, agrays...@gmail.com wrote:



 On Friday, June 1, 2018 at 3:59:05 PM UTC, Bruno Marchal wrote:
>
>
> On 31 May 2018, at 23:05, agrays...@gmail.com wrote:
>
>
> 
>>
>>
>> *How can you have experimental evidence for many worlds if they are 
>> disjoint from this world? AG *
>>
>> *When mathematics points to things which don't exist, it's usually, 
>> maybe always, the consequence of some unstated, erroneous assumption in 
>> its 
>> application. As I previously explained, it's a fallacy to apply the 
>> principle of superposition of states to entities that fail to have well 
>> defined deBroglie wave lengths (and which therefore can manifest 
>> interference) -- such as cats in a box, or instruments, or 
>> "environments". 
>> That's what Schrodinger warned us about, but the lesson has yet to sink 
>> in. 
>> AG*
>>
>> Then quantum mechanics is false somewhere in between the observed and 
>> the observer, but there are no evidences to back that claim. One history 
>> is 
>> no less speculative than many one, and one history makes no sense with 
>> the 
>> SWE for which evidences abound. Then, simple independent hypothesis 
>> leads 
>> directly to many histories, so QM as known today do confirm those 
>> independent hypothesis, like mechanism in the cognitive science (not in 
>> physics).
>>
>>
>> *CMIIAW,*
>>
>> ?
>>
>
>
> *Correct Me If I Am Wrong. AG *
>
>
>
> OK.
>
>
>
>
> *but I think Everett used superpositions of macro states similar to 
>> what Bruce wrote earlier, where sums of tensor products are formed using 
>> the apparatus and environment. *
>>
>>
>> OK.
>>
>
>
> *That's what I surmised. Thanks for the confirmation. AG *
>
>> *All I claimed above is NOT that quantum mechanics is false, but 
>> rather than one cannot form a legitimate superposition with entities 
>> that 
>> have no well defined deBroglie wave length -- since the existence of a 
>> well 
>> defined wave length is a necessary condition for interference, and 
>> that's 
>> the core property of a superposition. So, if you indulge this error you 
>> will get nonsense, such as a cat which is simultaneously alive and dead. 
>> AG*
>>
>> We cannot measure the “precise wavelength” in practice, but that is 
>> not needed to get the superposition state. Actually, you make the same 
>> remark that de Broglie himself, who concluded that superposition applies 
>> only to light atoms, and fade away on atomic distance. 
>>
>
>
> *Nothing can be measured precisely. Do you have a link to his comment? 
> AG*
>
>
>
> It is in one of his many book in French. I think it is in its “La 
> théorie de la mesure en Mécanique Ondulatoire (Interpretation usuelle et 
> interpretation causale)”. He wrote this after retirement when he came 
> back 
> to what could that theory means.
>

> But today we can get the interference effects with superposed “big” 
>> molecules, like the 60 carbon  ball, and cosmology indicated possible 
>> interference between highly dense and massive object. 
>>
>
> *As I expressly stated in an earlier post, billiard and Bucky balls 
> have well defined deBroglie wave lengths and thus CAN be included in 
> superpositions. However, most macro objects do NOT have well defined 
> deBroglie wave lengths, *
>
>
> Wave length applies to continuous variables, but when you entangle the 
> cat with a spin state, as in Bohm thought experience, we use only the 
> fact 
> that the state of the compound object is O, say, and the state of the cat 
> is O * cat *(up +down) = (O *(cat alive * up + cat-dead * down)) = O * 
> cat 
> alive *up + O * cat dead * down. The two branch are part of the “wave”, 
> or 
> better some state in some Hllbert Space (which is just an infinite linear 
> space, with some limits, and a scalar product).
>
>
>
>
> *such as an instrument in a lab, the lab itself, and the general 
> environment, and CANNOT be included in a superposition,*
>
>
> The problem is that by lack of reasonable isolation any object in our 
> hot environment decoheres at the speed of light (or not far). The 
> “multiplication of the universe” start at each point 

Re: Entanglement

2018-06-04 Thread Bruce Kellett

From: mailto:agrayson2...@gmail.com>>


On Tuesday, June 5, 2018 at 1:18:29 AM UTC, Bruce wrote:

From: 


Remember that the analysis I have given above is schematic,
representing the general progression of unitary evolution. It is
not specific to any particular case, or any particular number of
possible outcomes for the experiment.

Bruce

*OK. For economy we can write, **(|+>|e+> + |->|e->),  where e
stands for the entire universe other than the particle whose spin
is being measured. What is the status of the interference between
the terms in this superposition? For a quantum superposition to
make sense, there must be interference between the terms in the
sum. At least that's my understanding of the quantum principle of
superposition. But the universe excluding the particle being
measured seems to have no definable wave length; hence, I don't
see that this superposition makes any sense in how superposition
is applied. Would appreciate your input on this issue. TIA, AG*


A superposition is just a sum of vectors in Hilbert space. If
these vectors are orthogonal there is no interference between
them. Your quest for a wavelength in every superposition is the
wrong way to look at things. Macroscopic objects have vanishingly
small deBroglie wavelengths, but the can still be represented as
vectors in a HIlbert space, so can still form superpositions. I
think you are looking for absolute classicality in quantum
phenomena -- that is impossible, by definition.

Bruce


*If that's the case, why all the fuss about Schrodinger's cat? AG*


Is there a fuss about Schrödinger's cat? Whatever fuss there is, is not 
about the possibility of a superposition of live and dead cats. It is 
about choosing the correct basis in which to describe the physical 
situation. The Schrödinger equation does not specify a basis, and that 
is its main drawback. In fact, that observation alone is sufficient to 
sink the naive many-worlds enthusiast -- he doesn't know in which basis 
the multiplication of worlds occurs.


Bruce

--
You received this message because you are subscribed to the Google Groups 
"Everything List" group.
To unsubscribe from this group and stop receiving emails from it, send an email 
to everything-list+unsubscr...@googlegroups.com.
To post to this group, send email to everything-list@googlegroups.com.
Visit this group at https://groups.google.com/group/everything-list.
For more options, visit https://groups.google.com/d/optout.


Re: Entanglement

2018-06-04 Thread agrayson2000


On Tuesday, June 5, 2018 at 2:02:49 AM UTC, John Clark wrote:
>
> On Sat, May 26, 2018 at 12:50 AM, > 
> wrote:
>  
>
> *> when decoherence or the MWI implies the creation of full-blown worlds 
> (that we can't observe), there seems to be a large body of opinion that 
> accepts this bizarre result without serious criticism that there's no 
> mechanism or process for creating full-blown worlds.*
>
>
> But there is such a mechanism, the Schrodinger Wave Equation insists those 
> other worlds are there.
>

*And Maxwell's equations insist the existence of advanced waves. Do 
advanced waves exist even though they haven't been detected, because the 
mathematics insists otherwise? AG *

> In fact it could be argued that the Many World's Interpretation is poorly 
> named, it should simply be called Quantum Mechanics and the idea that some 
> unknown mechanism somehow destroys all worlds but one should not be called 
> the Copenhagen Interpretation, it should be called the Disappearing World's 
> Interpretation
> ​.​
>
>  
>
> No. I don't believe in such worlds. I tend to think a large segment of 
> professional physicists have gone mad.
>
>
> You believe the idea is mad because you don't feel like you split, and for 
> thousands o
> ...

-- 
You received this message because you are subscribed to the Google Groups 
"Everything List" group.
To unsubscribe from this group and stop receiving emails from it, send an email 
to everything-list+unsubscr...@googlegroups.com.
To post to this group, send email to everything-list@googlegroups.com.
Visit this group at https://groups.google.com/group/everything-list.
For more options, visit https://groups.google.com/d/optout.


Re: Entanglement

2018-06-04 Thread agrayson2000


On Tuesday, June 5, 2018 at 1:18:29 AM UTC, Bruce wrote:
>
> From: >
>
>
> Remember that the analysis I have given above is schematic, representing 
> the general progression of unitary evolution. It is not specific to any 
> particular case, or any particular number of possible outcomes for the 
> experiment.
>
> Bruce
>
> *OK. For economy we can write,  ** (|+>|e+> + |->|e->),  where e stands 
> for the entire universe other than the particle whose spin is being 
> measured. What is the status of the interference between the terms in this 
> superposition? For a quantum superposition to make sense, there must be 
> interference between the terms in the sum. At least that's my understanding 
> of the quantum principle of superposition. But the universe excluding the 
> particle being measured seems to have no definable wave length; hence, I 
> don't see that this superposition makes any sense in how superposition is 
> applied. Would appreciate your input on this issue. TIA, AG*
>
>
> A superposition is just a sum of vectors in Hilbert space. If these 
> vectors are orthogonal there is no interference between them. Your quest 
> for a wavelength in every superposition is the wrong way to look at things. 
> Macroscopic objects have vanishingly small deBroglie wavelengths, but the 
> can still be represented as vectors in a HIlbert space, so can still form 
> superpositions. I think you are looking for absolute classicality in 
> quantum phenomena -- that is impossible, by definition.
>
> Bruce
>

*If that's the case, why all the fuss about Schrodinger's cat? AG *

-- 
You received this message because you are subscribed to the Google Groups 
"Everything List" group.
To unsubscribe from this group and stop receiving emails from it, send an email 
to everything-list+unsubscr...@googlegroups.com.
To post to this group, send email to everything-list@googlegroups.com.
Visit this group at https://groups.google.com/group/everything-list.
For more options, visit https://groups.google.com/d/optout.


Re: Entanglement

2018-06-04 Thread John Clark
On Sat, May 26, 2018 at 12:50 AM,  wrote:


> *> when decoherence or the MWI implies the creation of full-blown worlds
> (that we can't observe), there seems to be a large body of opinion that
> accepts this bizarre result without serious criticism that there's no
> mechanism or process for creating full-blown worlds.*


But there is such a mechanism, the Schrodinger Wave Equation insists those
other worlds are there. In fact it could be argued that the Many World's
Interpretation is poorly named, it should simply be called Quantum
Mechanics and the idea that some unknown mechanism somehow destroys all
worlds but one should not be called the Copenhagen Interpretation, it
should be called the Disappearing World's Interpretation
​.​



> No. I don't believe in such worlds. I tend to think a large segment of
> professional physicists have gone mad.


You believe the idea is mad because you don't feel like you split, and for
thousands of years people though it was obvious the Earth does not more
because it doesn't feel like its moving, but then Newton showed there is no
reason you should expect to feel it move. And by spiting Everett explained
why the quantum world is so weird and he also explained why you don't feel
yourself split.

​ John K Clark​

-- 
You received this message because you are subscribed to the Google Groups 
"Everything List" group.
To unsubscribe from this group and stop receiving emails from it, send an email 
to everything-list+unsubscr...@googlegroups.com.
To post to this group, send email to everything-list@googlegroups.com.
Visit this group at https://groups.google.com/group/everything-list.
For more options, visit https://groups.google.com/d/optout.


Re: Entanglement

2018-06-04 Thread Bruce Kellett

From: mailto:agrayson2...@gmail.com>>


Remember that the analysis I have given above is schematic, 
representing the general progression of unitary evolution. It is not 
specific to any particular case, or any particular number of possible 
outcomes for the experiment.


Bruce

*OK. For economy we can write, **(|+>|e+> + |->|e->),  where e stands 
for the entire universe other than the particle whose spin is being 
measured. What is the status of the interference between the terms in 
this superposition? For a quantum superposition to make sense, there 
must be interference between the terms in the sum. At least that's my 
understanding of the quantum principle of superposition. But the 
universe excluding the particle being measured seems to have no 
definable wave length; hence, I don't see that this superposition 
makes any sense in how superposition is applied. Would appreciate your 
input on this issue. TIA, AG*


A superposition is just a sum of vectors in Hilbert space. If these 
vectors are orthogonal there is no interference between them. Your quest 
for a wavelength in every superposition is the wrong way to look at 
things. Macroscopic objects have vanishingly small deBroglie 
wavelengths, but the can still be represented as vectors in a HIlbert 
space, so can still form superpositions. I think you are looking for 
absolute classicality in quantum phenomena -- that is impossible, by 
definition.


Bruce

--
You received this message because you are subscribed to the Google Groups 
"Everything List" group.
To unsubscribe from this group and stop receiving emails from it, send an email 
to everything-list+unsubscr...@googlegroups.com.
To post to this group, send email to everything-list@googlegroups.com.
Visit this group at https://groups.google.com/group/everything-list.
For more options, visit https://groups.google.com/d/optout.


Re: Entanglement

2018-06-04 Thread agrayson2000
From: 

On Sunday, June 3, 2018 at 2:23:46 AM UTC, agrays...@gmail.com wrote: 

>
> On Saturday, May 26, 2018 at 12:06:53 AM UTC, Bruce wrote: 
>>
>> From: >
>> On Wednesday, May 23, 2018 at 8:16:07 AM UTC, Bruce wrote: 
>>>
>>> From: >>
>>> OK, but how does one jump to the assumption of other worlds? Doesn't 
>>> each "branch" exist in this world? AG 
>>>
>>>
>>> Initially yes. But decoherence diagonalizes the density matrix FAPP, so 
>>> the other branches become unreachable. That is what one means by separate 
>>> worlds.
>>>
>>
>> *I am tentatively OK with this conclusion (tentatively until I examine 
>> the mathematics and verify it), as long as these separate "worlds" do NOT 
>> contain copies of THIS world. It's the copying that I find hugely 
>> extravagant, ridiculous, and erroneous! Can decoherence theory be 
>> consistent without the "copying" claim?  Is this the view you adopt to keep 
>> your sanity? TIA, AG*
>>
>>
>> The fact that the whole world is copied in each branch of the MWI is a 
>> simple consequence of the mathematics. If one has a state
>>
>> |psi> = (|+> + |->)
>>
>> that one measures, which is a superposition of two possible outcome 
>> states, |+> and |->, then schematically this measurement process looks like
>>
>>  |psi>|A>|O>|e>,
>>
>> where |A> is the apparatus, |O> is the observer, and |e> is everything 
>> else, namely the environment. Unitary evolution takes this to:
>>
>> (|+>|A+>|P+>|e+> + |->|A->|O->|e->)
>>
>> where |A+> means the apparatus register the |+> result, |O+> means the 
>> observer sees the |+> result, and |e+> means that information about the |+> 
>> result leaks into the environment by decoherence and is effectively 
>> recorded there many times. Similarly for the other |-> branch.
>>
>
> *As previously noted, the formula you meant to write is:   
> (|+>|A+>|O+>|e+> + |->|A->|O->|e->). What thought experiment would be 
> appropriate to understand either of the environmental states? AG*
>

*Does |e+> represent the eigenstate of the universe, excluding the particle 
being measured, the apparatus and the observer, when the spin is measured 
UP? That is, what is the physical content and meaning of |e+> within the 
standard formalism of QM? AG*


You are right, I meant |O+> for the observer who sees |+>. But the 
environment states, |e+> and |e->, are not necessarily eigenvalues of any 
particular simple operator. They are, rather, a schematic representation of 
the total environment (excluding apparatus and observer) that, by 
decoherence, contains multiple copies of the result (|+> or |->, 
respectively). By construction, these environmental states are orthogonal.

Remember that the analysis I have given above is schematic, representing 
the general progression of unitary evolution. It is not specific to any 
particular case, or any particular number of possible outcomes for the 
experiment.

Bruce

*OK. For economy we can write,  *
* (|+>|e+> + |->|e->),  where e stands for the entire universe other than 
the particle whose spin is being measured. What is the status of the 
interference between the terms in this superposition? For a quantum 
superposition to make sense, there must be interference between the terms 
in the sum. At least that's my understanding of the quantum principle of 
superposition. But the universe excluding the particle being measured seems 
to have no definable wave length; hence, I don't see that this 
superposition makes any sense in how superposition is applied. Would 
appreciate your input on this issue. TIA, AG*

-- 
You received this message because you are subscribed to the Google Groups 
"Everything List" group.
To unsubscribe from this group and stop receiving emails from it, send an email 
to everything-list+unsubscr...@googlegroups.com.
To post to this group, send email to everything-list@googlegroups.com.
Visit this group at https://groups.google.com/group/everything-list.
For more options, visit https://groups.google.com/d/optout.


Re: Entanglement

2018-06-04 Thread Bruno Marchal

> On 3 Jun 2018, at 23:37, agrayson2...@gmail.com wrote:
> 
> 
> 
> On Sunday, June 3, 2018 at 3:15:13 PM UTC, agrays...@gmail.com wrote:
> 
> 
> On Sunday, June 3, 2018 at 1:05:48 PM UTC, Bruno Marchal wrote:
> 
>> On 2 Jun 2018, at 00:23, agrays...@gmail.com <> wrote:
>> 
>> On Friday, June 1, 2018 at 4:43:29 PM UTC, agrays...@gmail.com 
>>  wrote:
>> 
>> 
>> On Friday, June 1, 2018 at 3:59:05 PM UTC, Bruno Marchal wrote:
>> 
>>> On 31 May 2018, at 23:05, agrays...@gmail.com <> wrote:
>>> 
>>> 
>>> 
>>> 
>>> How can you have experimental evidence for many worlds if they are disjoint 
>>> from this world? AG 
>>> When mathematics points to things which don't exist, it's usually, maybe 
>>> always, the consequence of some unstated, erroneous assumption in its 
>>> application. As I previously explained, it's a fallacy to apply the 
>>> principle of superposition of states to entities that fail to have well 
>>> defined deBroglie wave lengths (and which therefore can manifest 
>>> interference) -- such as cats in a box, or instruments, or "environments". 
>>> That's what Schrodinger warned us about, but the lesson has yet to sink in. 
>>> AG
>>> Then quantum mechanics is false somewhere in between the observed and the 
>>> observer, but there are no evidences to back that claim. One history is no 
>>> less speculative than many one, and one history makes no sense with the SWE 
>>> for which evidences abound. Then, simple independent hypothesis leads 
>>> directly to many histories, so QM as known today do confirm those 
>>> independent hypothesis, like mechanism in the cognitive science (not in 
>>> physics).
>>> 
>>> CMIIAW,
>>> ?
>>> 
>>> Correct Me If I Am Wrong. AG 
>> 
>> 
>> OK.
>> 
>> 
>> 
>> 
>>> but I think Everett used superpositions of macro states similar to what 
>>> Bruce wrote earlier, where sums of tensor products are formed using the 
>>> apparatus and environment.
>>> 
>>> OK.
>>> 
>>> That's what I surmised. Thanks for the confirmation. AG 
>>> All I claimed above is NOT that quantum mechanics is false, but rather than 
>>> one cannot form a legitimate superposition with entities that have no well 
>>> defined deBroglie wave length -- since the existence of a well defined wave 
>>> length is a necessary condition for interference, and that's the core 
>>> property of a superposition. So, if you indulge this error you will get 
>>> nonsense, such as a cat which is simultaneously alive and dead. AG
>>> We cannot measure the “precise wavelength” in practice, but that is not 
>>> needed to get the superposition state. Actually, you make the same remark 
>>> that de Broglie himself, who concluded that superposition applies only to 
>>> light atoms, and fade away on atomic distance.
>>> 
>>> Nothing can be measured precisely. Do you have a link to his comment? AG
>> 
>> 
>> It is in one of his many book in French. I think it is in its “La théorie de 
>> la mesure en Mécanique Ondulatoire (Interpretation usuelle et interpretation 
>> causale)”. He wrote this after retirement when he came back to what could 
>> that theory means.
>> 
>> But today we can get the interference effects with superposed “big” 
>> molecules, like the 60 carbon  ball, and cosmology indicated possible 
>> interference between highly dense and massive object.
>> 
>> As I expressly stated in an earlier post, billiard and Bucky balls have well 
>> defined deBroglie wave lengths and thus CAN be included in superpositions. 
>> However, most macro objects do NOT have well defined deBroglie wave lengths,
>> 
>> 
>> Wave length applies to continuous variables, but when you entangle the cat 
>> with a spin state, as in Bohm thought experience, we use only the fact that 
>> the state of the compound object is O, say, and the state of the cat is O * 
>> cat *(up +down) = (O *(cat alive * up + cat-dead * down)) = O * cat alive 
>> *up + O * cat dead * down. The two branch are part of the “wave”, or better 
>> some state in some Hllbert Space (which is just an infinite linear space, 
>> with some limits, and a scalar product).
>> 
>> 
>> 
>> 
>>> such as an instrument in a lab, the lab itself, and the general 
>>> environment, and CANNOT be included in a superposition,
>> 
>> The problem is that by lack of reasonable isolation any object in our hot 
>> environment decoheres at the speed of light (or not far). The 
>> “multiplication of the universe” start at each point of the front of the 
>> universal wave. The universal wave is a sum of all its “front” wave in the 
>> space-time, structure, except that with GR we have no clue on how to proceed 
>> (well, The M theory is perhaps right, and string theory has found 
>> application in Number Theory, …).
>> 
>> 
>> 
>> 
>> 
>> 
>> 
>>> the main property of which is interference. This is what Schrodinger 
>>> demonstrated in his cat paradox, but the lesson has been lost. AG
>> 
>> The formalism implies the many world, like the periodic chemical properties 
>> of the 

Re: Entanglement

2018-06-03 Thread Bruce Kellett

From: mailto:agrayson2...@gmail.com>>
On Sunday, June 3, 2018 at 2:23:46 AM UTC, agrays...@gmail.com 
 wrote:



On Saturday, May 26, 2018 at 12:06:53 AM UTC, Bruce wrote:

From: 
On Wednesday, May 23, 2018 at 8:16:07 AM UTC, Bruce wrote:

From: 
OK, but how does one jump to the assumption of other
worlds? Doesn't each "branch" exist in this world? AG


Initially yes. But decoherence diagonalizes the density
matrix FAPP, so the other branches become unreachable.
That is what one means by separate worlds.


*I am tentatively OK with this conclusion (tentatively until
I examine the mathematics and verify it), as long as these
separate "worlds" do NOT contain copies of THIS world. It's
the copying that I find hugely extravagant, ridiculous, and
erroneous! Can decoherence theory be consistent without the
"copying" claim?  Is this the view you adopt to keep your
sanity? TIA, AG*


The fact that the whole world is copied in each branch of the
MWI is a simple consequence of the mathematics. If one has a state

|psi> = (|+> + |->)

that one measures, which is a superposition of two possible
outcome states, |+> and |->, then schematically this
measurement process looks like

 |psi>|A>|O>|e>,

where |A> is the apparatus, |O> is the observer, and |e> is
everything else, namely the environment. Unitary evolution
takes this to:

(|+>|A+>|P+>|e+> + |->|A->|O->|e->)

where |A+> means the apparatus register the |+> result, |O+>
means the observer sees the |+> result, and |e+> means that
information about the |+> result leaks into the environment by
decoherence and is effectively recorded there many times.
Similarly for the other |-> branch.


*As previously noted, the formula you meant to write is:
(|+>|A+>|O+>|e+> + |->|A->|O->|e->). What thought experiment would
be appropriate to understand either of the environmental states? AG*


*Does |e+> represent the eigenstate of the universe, excluding the 
particle being measured, the apparatus and the observer, when the spin 
is measured UP? That is, what is the physical content and meaning of 
|e+> within the standard formalism of QM? AG*


You are right, I meant |O+> for the observer who sees |+>. But the 
environment states, |e+> and |e->, are not necessarily eigenvalues of 
any particular simple operator. They are, rather, a schematic 
representation of the total environment (excluding apparatus and 
observer) that, by decoherence, contains multiple copies of the result 
(|+> or |->, respectively). By construction, these environmental states 
are orthogonal.


Remember that the analysis I have given above is schematic, representing 
the general progression of unitary evolution. It is not specific to any 
particular case, or any particular number of possible outcomes for the 
experiment.


Bruce

--
You received this message because you are subscribed to the Google Groups 
"Everything List" group.
To unsubscribe from this group and stop receiving emails from it, send an email 
to everything-list+unsubscr...@googlegroups.com.
To post to this group, send email to everything-list@googlegroups.com.
Visit this group at https://groups.google.com/group/everything-list.
For more options, visit https://groups.google.com/d/optout.


Re: Entanglement

2018-06-03 Thread agrayson2000


On Sunday, June 3, 2018 at 3:15:13 PM UTC, agrays...@gmail.com wrote:
>
>
>
> On Sunday, June 3, 2018 at 1:05:48 PM UTC, Bruno Marchal wrote:
>>
>>
>> On 2 Jun 2018, at 00:23, agrays...@gmail.com wrote:
>>
>> On Friday, June 1, 2018 at 4:43:29 PM UTC, agrays...@gmail.com wrote:
>>>
>>>
>>>
>>> On Friday, June 1, 2018 at 3:59:05 PM UTC, Bruno Marchal wrote:


 On 31 May 2018, at 23:05, agrays...@gmail.com wrote:


 
>
>
> *How can you have experimental evidence for many worlds if they are 
> disjoint from this world? AG *
>
> *When mathematics points to things which don't exist, it's usually, 
> maybe always, the consequence of some unstated, erroneous assumption in 
> its 
> application. As I previously explained, it's a fallacy to apply the 
> principle of superposition of states to entities that fail to have well 
> defined deBroglie wave lengths (and which therefore can manifest 
> interference) -- such as cats in a box, or instruments, or 
> "environments". 
> That's what Schrodinger warned us about, but the lesson has yet to sink 
> in. 
> AG*
>
> Then quantum mechanics is false somewhere in between the observed and 
> the observer, but there are no evidences to back that claim. One history 
> is 
> no less speculative than many one, and one history makes no sense with 
> the 
> SWE for which evidences abound. Then, simple independent hypothesis leads 
> directly to many histories, so QM as known today do confirm those 
> independent hypothesis, like mechanism in the cognitive science (not in 
> physics).
>
>
> *CMIIAW,*
>
> ?
>


 *Correct Me If I Am Wrong. AG *



 OK.




 *but I think Everett used superpositions of macro states similar to 
> what Bruce wrote earlier, where sums of tensor products are formed using 
> the apparatus and environment. *
>
>
> OK.
>


 *That's what I surmised. Thanks for the confirmation. AG *

> *All I claimed above is NOT that quantum mechanics is false, but 
> rather than one cannot form a legitimate superposition with entities that 
> have no well defined deBroglie wave length -- since the existence of a 
> well 
> defined wave length is a necessary condition for interference, and that's 
> the core property of a superposition. So, if you indulge this error you 
> will get nonsense, such as a cat which is simultaneously alive and dead. 
> AG*
>
> We cannot measure the “precise wavelength” in practice, but that is 
> not needed to get the superposition state. Actually, you make the same 
> remark that de Broglie himself, who concluded that superposition applies 
> only to light atoms, and fade away on atomic distance. 
>


 *Nothing can be measured precisely. Do you have a link to his comment? 
 AG*



 It is in one of his many book in French. I think it is in its “La 
 théorie de la mesure en Mécanique Ondulatoire (Interpretation usuelle et 
 interpretation causale)”. He wrote this after retirement when he came back 
 to what could that theory means.

>>>
 But today we can get the interference effects with superposed “big” 
> molecules, like the 60 carbon  ball, and cosmology indicated possible 
> interference between highly dense and massive object. 
>

 *As I expressly stated in an earlier post, billiard and Bucky balls 
 have well defined deBroglie wave lengths and thus CAN be included in 
 superpositions. However, most macro objects do NOT have well defined 
 deBroglie wave lengths, *


 Wave length applies to continuous variables, but when you entangle the 
 cat with a spin state, as in Bohm thought experience, we use only the fact 
 that the state of the compound object is O, say, and the state of the cat 
 is O * cat *(up +down) = (O *(cat alive * up + cat-dead * down)) = O * cat 
 alive *up + O * cat dead * down. The two branch are part of the “wave”, or 
 better some state in some Hllbert Space (which is just an infinite linear 
 space, with some limits, and a scalar product).




 *such as an instrument in a lab, the lab itself, and the general 
 environment, and CANNOT be included in a superposition,*


 The problem is that by lack of reasonable isolation any object in our 
 hot environment decoheres at the speed of light (or not far). The 
 “multiplication of the universe” start at each point of the front of the 
 universal wave. The universal wave is a sum of all its “front” wave in the 
 space-time, structure, except that with GR we have no clue on how to 
 proceed (well, The M theory is perhaps right, and string theory has found 
 application in Number Theory, …).




Re: Entanglement

2018-06-03 Thread agrayson2000


On Sunday, June 3, 2018 at 2:23:46 AM UTC, agrays...@gmail.com wrote:
>
>
>
> On Saturday, May 26, 2018 at 12:06:53 AM UTC, Bruce wrote:
>>
>> From: >
>> On Wednesday, May 23, 2018 at 8:16:07 AM UTC, Bruce wrote: 
>>>
>>> From: >>
>>> OK, but how does one jump to the assumption of other worlds? Doesn't 
>>> each "branch" exist in this world? AG 
>>>
>>>
>>> Initially yes. But decoherence diagonalizes the density matrix FAPP, so 
>>> the other branches become unreachable. That is what one means by separate 
>>> worlds.
>>>
>>
>> *I am tentatively OK with this conclusion (tentatively until I examine 
>> the mathematics and verify it), as long as these separate "worlds" do NOT 
>> contain copies of THIS world. It's the copying that I find hugely 
>> extravagant, ridiculous, and erroneous! Can decoherence theory be 
>> consistent without the "copying" claim?  Is this the view you adopt to keep 
>> your sanity? TIA, AG*
>>
>>
>> The fact that the whole world is copied in each branch of the MWI is a 
>> simple consequence of the mathematics. If one has a state
>>
>> |psi> = (|+> + |->)
>>
>> that one measures, which is a superposition of two possible outcome 
>> states, |+> and |->, then schematically this measurement process looks like
>>
>>  |psi>|A>|O>|e>,
>>
>> where |A> is the apparatus, |O> is the observer, and |e> is everything 
>> else, namely the environment. Unitary evolution takes this to:
>>
>> (|+>|A+>|P+>|e+> + |->|A->|O->|e->)
>>
>> where |A+> means the apparatus register the |+> result, |O+> means the 
>> observer sees the |+> result, and |e+> means that information about the |+> 
>> result leaks into the environment by decoherence and is effectively 
>> recorded there many times. Similarly for the other |-> branch.
>>
>
> *As previously noted, the formula you meant to write is:   
> (|+>|A+>|O+>|e+> + |->|A->|O->|e->). What thought experiment would be 
> appropriate to understand either of the environmental states? AG*
>

*Does |e+> represent the eigenstate of the universe, excluding the particle 
being measured, the apparatus and the observer, when the spin is measured 
UP? That is, what is the physical content and meaning of |e+> within the 
standard formalism of QM? AG*

>
>
>> As one can see immediately, this evolution necessarily means that 
>> everything is duplicated, the apparatus, observer, and the rest of the 
>> world, differing in the two branches only in consequence of the different 
>> measurement results (|+> or |->). Decoherence does not cause the "copying", 
>> the copying is a result of the Schrödinger equation. Decoherence occurs 
>> independently in each branch, as can be seen in the above schematic outline 
>> of the process.
>>
>> Bruce
>>
>

-- 
You received this message because you are subscribed to the Google Groups 
"Everything List" group.
To unsubscribe from this group and stop receiving emails from it, send an email 
to everything-list+unsubscr...@googlegroups.com.
To post to this group, send email to everything-list@googlegroups.com.
Visit this group at https://groups.google.com/group/everything-list.
For more options, visit https://groups.google.com/d/optout.


  1   2   3   4   5   6   7   >